Sunteți pe pagina 1din 150

Q.

1) The incidence of breech at term is


(a) 1%
(b) 3%
(c) 99.5%
(d) 0.5%
Your Response : b
Correct Answer : B
Exp: 3%
Researchers generally site a breech presentation frequency at term of 3-4%. at the onset of la
some claim a frequency as high as 7%.). the incidence is about 20 % at 28 wks which drops to 3

(Q.2) Aromatase enzyme


(a) Converts estrogen to androgen
(b) Converts androgen to estrogen
(c) Helps in maintaining normal placental blood flow
(d) None of the above
Your
a
Response :
Correct
B
Answer :
Exp: Converts androgen to estrogen
Aromatization refers to conversion of androgen to estrogen by
aromatase enzyme.
MNEUMONIC: A converts to A to E.

(Q.3) In a pregnant woman DVT


(a) Is more common in left leg as compare to right leg
(b) Is more common in right leg as compare to left leg
(c) Equal in both the legs
(d) Is not common
Your Response : b
Correct Answer : A
Exp: Is more common in left leg as compare to right leg
A hypercoagulable state is characteristic of pregnancy, and deep venous thrombosis (DVT) oc
in 2000 pregnancies. Pulmonary embolism is one of the most common causes of maternal de
United States. In pregnant women, DVT occurs much more commonly in the left leg than in th
due to the compression of the left iliac vein by the iliac artery and the uterus.
Activated protein C resistance caused by the factor V Leiden mutation increases the risk for D
pulmonary embolism during pregnancy. Approximately 25% of women with DVT during pregn
factor V Leiden allele. The presence of the factor V Leiden mutation also increases the risk for
preeclampsia.

(Q.4) Most common cause of acute cervicitis is:


(a) E. Coli
(b) Chlamydia
(c) Pseudomonas
(d) Gonococcus
Your Response : c
Correct Answer : B
Exp: Chlamydia
Chlamydia trachomatis as being the most common single organism isolated from acute muco
cervicitis patients.
Also JEFFCOATS 7thy Ed chapter 20 just enumerates the causes as being N. gonorrheae, Chla
trachomatis or puerperial origin without actually mentioning the most common cause.

(Q.5) All are indications for post operative radiotherapy in a case of Carcinoma Endometrium EXCEPT?
(a) Myometrial invasion >1/2 thickness
(b) Positive lymph nodes
(c) Endocervical involvement
(d) Tumor positive for estrogen receptors
Your Response : d
Correct Answer : D
Exp: Tumor positive for estrogen receptors
Receptor positivity have no influence on postop radiotherapy.
As far as receptor status is concerned, the best correlation is that of a good prognosis with hig
progesterone receptors rather than a poor prognosis with high estrogen receptors
The best prognostic factor of cancer endometrium, amongst others, is the Histological gradin
disease
Other important predictors are:
Depth of myometrial invasion
Status of pelvic and para-aortic lymph nodes
Malignant cells in peritoneal washings
Lymph vascular invasion
Cervical invasion

(Q.6) One of the following is a definitive indication of caesarean section


(a) Previous LSCS
(b) Placenta previa
(c) Breech
(d) Twins in labour
Your Response : b
Correct Answer : B
Exp: Placenta previa
Twins in labour can be allowed normal delivery . The indication of caesarean specifically relat
include1. First twin in non-vertex presentation
Monochorionic monoamniotic twins
Previous LSCS
Interlocking
TTTS
Conjoint twins
Previous caesarean can be allowed vaginal birth. The success is 60%. Breech can be allowed v
either directly or as cephalic after ECV. Major degree placenta previa is the definitive indicatio
caesarean section.

(Q.7) Investigation of choice in postdated pregnancy is


(a) Ultrasound
(b) NST
(c) BPS
(d) Modified BPS
Your Response : d
Correct Answer : D
Exp: Modified BPS
Best investigation for postdated pregnancy is modified biophysical score that combines two i
-- NST which is an index for acute fetal problem and AFI that is an index of chronic problem an
placental function. The management includes assessment of Bishops score along with above
procedures and decision should be taken based on the combination of the three. Postdatism
indication of termination of pregnancy if NST and AFI are normal.

(Q.8) Drug of choice for trichomonas vaginalis


(a) Fluconazole
(b) Metronidazole
(c) Azithromycin
(d) Nevirapine
Your Response : b
Correct Answer : B
Exp: Metronidazole
Trichomoniasis is treated with an oral antiprotozoal medicine, such as metronidazole or tinida
medicine is taken either as a single dose or as multiple doses. The cure rate in treating trich u
metronidazole is 90% to 95%. The cure rate using tinidazole is 86% to 100%.Sex partner(s) sho
at the same time. This increases the cure rate and reduces the possibility of further transmiss
reinfection. Sexual intercourse should be avoided during treatment until symptoms have gon
until partners have been treated. Male partners may not have symptoms but still need trea

(Q.9) Cryptomenorrhea is seen in:


(a) Asherman's syndrome
(b) Testicular feminizing syndrome
(c) Imperforate hymen
(d) Uterine agenesis
Your Response : c
Correct Answer : C
Exp: Imperforate hymen
Cryptomenorrhea means monthly subjective symptoms of menstruation without flow of bloo
caused by an imperforate hymen. Crypt literally means hidden. Thus cryptmenorrhoea means
menorrhoea.

(Q.10) Management of CIN III


(a) Cervical biopsy
(b) Hysterectomy
(c) Cone biopsy
(d) LEEP conisation
Your Response : d
Correct Answer : D
Exp: LEEP conisation
CINIII , once diagnosed after cervical biopsy ,needs to be further investigated to rule out inv
done by taking a cone of the cervix that includes all the ectocervix and the endocervical glan
specimen can be taken with a scalpel which is the traditional cold knife conization or with LE
The lesser morbidity associated with LEEP is making it a better choice for further manageme
If on cone specimen which could have been obtained by any of the mentioned technique, th
invasion then the procedure is therapeutic . If invasion is demonstrated then hysterectomy i
(Q.11) In a rape victim which of the following is true about HIV
prophylaxis?
(a) Combivir 1BD x 7 days
(b) Combivir 1BD x 14 days
(c) Combivir 1BD x 21 days
(d) Combivir 1BD x 28 days
Your
c
Response :
Correct
D
Answer :
Exp: Combivir 1BD x 28 days
Ref: Nelson, 18th edition, Page 855, Table 118-3

(Q.12) HIV transmission in relation to pregnancy is maximum during


(a) Labour and delivery
(b) First trimester
(c) Second trimester
(d) Breast feeding
Your Response : a
Correct Answer : A
Exp: Labour and delivery
The maximum risk is at labour and delivery. A baby can become infected with HIV in the wom
delivery or while breast-feeding. If the mother does not receive treatment, 25 % of babies b
with HIV will be infected by the virus.
With treatment that percentage can be reduced to less than 2 %. If no preventative steps ar
risk of HIV transmission during childbirth is estimated to be 10-20%.
The chance of transmission is even greater if the baby is exposed to HIV-infected blood or flu
care providers should avoid performing amniotomies (intentionally rupturing the amniotic s
labor), episiotomies and other procedures that expose the baby to the mothers blood.
The risk of transmission increases by 2% for every hour after membranes have been rupture
Cesarean sections performed before labor and/or the rupture of membranes may significan
risk of perinatal transmission of HIV.
About 15% of newborns born to HIV-positive women will become infected if they breastfeed
months or longer.

(Q.13) A pregnant 35-yrs-old patient is at highest risk for the concurrent development of which malignancies?
(a) Cervix
(b) Ovary
(c) Breast
(d) Vagina
Your Response : c
Correct Answer : A
Exp: Cervix
Cervical cancer is a more common gynecologic malignancy in pregnancy than ovarian or br
due to the fact that it is a disease of younger women.
Management of cervical intraepithelial lesions is complicated in pregnancy because of increa
vascularity of the cervix and because of the concern that manipulation and trauma to the ce
compromise continuation of the pregnancy.
A traditional cone biopsy is only indicated in the presence of apparent microinvasive disease
colposcopically directed cervical biopsy Otherwise, more limited procedures such as shallow
are more appropriate.
If invasive cancer is diagnosed, the decision to treat immediately or wait until fetal viability d
on the gestational age at which the diagnosis is made and the severity o disorder.
Survival is decreased for malignancies discovered later in pregnancy Radiation therapy almo
results in spontaneous abortion, in part because the fetus is particularly radiosensitive.
Chemotherapy is associated with higher than expected rates of fetal malformations consiste
antimetabolite effects of agents used. Specific malformations depend on the agent used and
pregnancy at which the exposure occurs.

(Q.14) Most likely affected organ in Meyer-Rokitansky-Kster-Hauser syndrome is:


(a) Ovary agenesis
(b) Fallopian tubes absent
(c) Vaginal atresia
(d) Bicornuate uterus
Your Response : b
Correct Answer : C
Exp: Vaginal atresia
Congenital hypoplasia or absence of vagina in combination with abnormal or absent uterus
MayerRbkitansky-Kuster-Hauser syndrome) is second most common cause of primary amen
patients are ascertained after the time of expected puberty because they fail to menstruate
normal, and the breasts, axillary and pubic hair, and habitus are feminine in character. The u
from almost normal, lacking only a conduit to the introitus, to the characteristic rudimentary
cords with or without a lumen. The karyotype is 46, XX.

(Q.15) EMA-CO regime of chemotherapy is used in the treatment of?


(a) Gestational trophoblastic neoplasia
(b) Malignant Ovarian germ cell tumor
(c) Endometrial carcinoma
(d) CA cervix
Your Response : c
Correct Answer : A
Exp: Gestational trophoblastic neoplasia Ref: Dutta, 6th Edition (Revised 2009), Page no 193-20
Patients with high-risk tumors (high beta-hCG levels, disease presenting 4 months after ant
pregnancy, brain or liver metastasis, or failure of single-agent methotrexate) are initially trea
combination chemotherapy. EMA-CO (a cyclic non-cross-resistant combination of etoposide
methotrexate, and dactinomycin alternating with cyclophosphamide and vincristine); cisplat
and vinblastine; and cisplatin, etoposide, and bleomycin are effective regimens. EMA-CO is n
regimen of choice for patients with high-risk disease because of excellent survival rates (>8
toxicity.

(Q.16) Highest rate of transmission of toxoplasmosis in pregnancy is:


(a) Puerperium
(b) 3rd trimester
(c) 2nd trimester
(d) 1 st trimester.
Your Response : c
Correct Answer : B
Exp: 3rd trimester
Ref: Dutta, 6th Edition (Revised 2009), Page no 296
About 1/3rd of all women infected with Toxoplasma gondii during pregnancy transmit the p
fetus; the remainder will give birth to normal, uninfected babies. Of the various factors that
outcome, gestational age at the time of infection is the most critical. In pregnancy, if the mo
infected during the first trimester, the incidence of transplacental infection is lowest (abou
the disease in the neonate is most severe. If maternal infection occurs during the third trim
incidence of transplacental infection is greatest (65 %), but the infant is usually asymptoma

(Q.17) Ru 486 can be used in all of the following conditions except


(a) Endometriosis
(b) Emergency contraception
(c) To cause abortion
(d) Endometrial carcinoma
Your Response : d
Correct Answer : D
Exp: Endometrial carcinoma
Ref: Dutta, 6th Edition (Revised 2009), Page no 175, 550
Mifepristone (RU 486) is a recently developed 19-nonsteroid with potent competitive anti
progestationaland significant antiglucocorticoid activity. Its uses are as under:
Termination of pregnancy of up to 9 weeks: 600 mg as single oral dose causes complete abo
85%'1 cases.
Mifepristone administered within 10 days of a missed period results in an apparent late hea
dislodged blastocyst) in up to 90% of cases.
Ontra gestational: Given once a month on the expected date of menstruation it can dislodge
present) and thus ensure menstruation irrespective of contraception or otherwise.
Administered as a single dose within 72 hours of intercourse, it can serve as a postcoital con
fewer side effects than high dose estrogen. Induction of labor: By blocking the relaxant actio
progesterone on uterus of late pregnancy, mifepristone can induce labor.
Cushing's syndrome: For inoperative cases.
Other uses under evaluation: Endometriosis, uterine fibroid, certain breast cancers and men

(Q.18) Contraceptive of choice during lactation:


(a) POP
(b) Combined oral contraceptive pills
(c) Barrier method
(d) IUCD
Your Response : a
Correct Answer : A
Exp: POP
Women should be advised that the use of progestogen-only methods in the first 6 weeks po
not appear to have an adverse effect on breast milk volume (Grade B).
Women should be advised that the use of progestogen-only methods when breastfeeding p
99% efficacy (Grade B). Women should be advised that the problematic bleeding associated
progestogen-only methods appears to be more acceptable than that experienced by women
breast. The majority of studies show no adverse effects of POPs or DMPA on breastfeeding
infant growth or development .
Lactational amenorrhoea method (LAM) -started immediately postpartum to provide effect
contraception.
Remember that the LAM is an interim method effective for the first 6 months postpartum o
she would need alternate contraceptive method.

(Q.19) Cause of infection in TB of fallopian tube is:


(a) Hematogenous spread from a primary focus elsewhere
(b) Lymphatic spread
(c) Ascending infection
(d) Primary infection
Your Response : a
Correct Answer : A
Exp: Hematogenous spread from a primary focus elsewhere
Genital tuberculosis is almost always secondary to a focus elsewhere in the body and the pri
most often situated in the lungs, lymph nodes, urinary tract, bones and joints in that order. T
hematogenous spread is the most common method of spread.

(Q.20) Regarding vaginal candidiasis which is false:


(a) Cottage cheese like secretions are seen
(b) Intense pruritus
(c) Most common in non-pregnant women
(d) Buds and hyphae seen in KOH preparation
Your Response : b
Correct Answer : C
Exp: Most common in non-pregnant women
Candidiasis is caused by Candida albinism which thrives in an acidic medium with an abunda
carbohydrates. It is found commonly in pregnancy, and in patients on oral contraceptives,
and. corticosteroids. It is also seen in patients with diabetes. It causes a profuse discharge a
pruritus with soreness of vagina and dysuria. Vagina and vulva are edematous and excoriate
white patches of cheesy material on the vagina and vulva can be easily identified. A 10% KOH
the buds and hyphae of the Candida organism. Nystatin or ketoconazole vaginal tablets use
are curative when used along with a cream of the same medication for External use.

(Q.21) Premenopausal peripheral conversion of estrogen precursors in the obese patient results in formation of:
(a) Estriol
(b) Estradiol
(c) Estrone
(d) Androstenedione
Your Response : a
Correct Answer : C
Exp: Most common in non-pregnant women
In premenopausal adult women, most of the estrogen in the body is derived from ovarian se
estradiol, but a significant portion comes also from the extraglandular conversion of andros
estrone. To a lesser extent, testosterone conversion to estradiol also contributes to the estr
Muscle and adipose tissue are the major sites of aromatization. When there is an increase in
obese persons, estrogen levels will be higher, because adipose tissue exhibits a greater aro
androstenedione to estrone than does muscle.

(Q.22) A 59-yrs-old woman undergoes vaginal hysterectomy and anteroposterior repair for uterine prolapse. Which of the fo
complication of this procedure that often develops within 2 weeks of surgery?
(a) Dyspareunia
(b) Stress urinary incontinence
(c) Nonfistulous fecal incontinence
(d) Enterocele
Your Response : b
Correct Answer : B
Exp: Stress urinary incontinence
Many patients who have uterine prolapse or a large protuberant cystocele will be continent
urethra obstruction caused by the cystocele or prolapse.
In fact, at times these patients may need to reduce the prolapse in order to void. Following s
if the urethrovesical junction is not properly elevated, urinary incontinence may result.
This incontinence may present within the first few days following surgery.
Dyspareunia can be caused by shortening of the vagina or constriction at the introitus after h
complete.
If the vaginal vault is not properly suspended and the uterosacral ligaments plicated, vaginal
or enterocele may occur at a later date. Fecal incontinence is not a complication of vaginal h
with repair. It may occur, however, if a fistula is formed through unrecognized damage to th
mucosa.

(Q.23) Velamentous insertion of the cord is associated with an increased risk for:
(a) Premature rupture of the membranes
(b) Fetal exsanguinations before labor
(c) Torsion of the umbilical cord
(d) Fetal malformations
Your Response : b
Correct Answer : B
Exp: Fetal exsanguinations before labor
With velamentous insertion of the cord, the umbilical vessels separate in the membranes at
from the placental margin, which they reach surrounded only by amnion. It occurs in about 1
gestations but is quite common in multiple pregnancies. Feta malformations are more comm
mentous insertion the umbilical cord. When fetal vessels cross the internal us (vasa previa),
membranes may be accompanied by rupture of a fetal vessel, leading to fetal exsanguinatio
risk of premature rupture of membranes and of torsion of the umbilical cord has not been d
association with velamentous insertion of the cord.

(Q.24) What is the name of following operation?

(a) Cho suture


(b) Internal iliac ligation
(c) Operation for uterine inversion
(d) B-lynch Suture
Your
Respons d
e:
Correct
D
Answer :
Exp: B-lynch Suture

(Q.25) A 65- year old woman complains of leakage of urine. The most common cause of this condition in such patient is
(a) Anatomic stress urinary incontinence
(b) Urethral diverticula
(c) Overflow incontinence
(d) Unstable bladder
Your Response : a
Correct Answer : D
Exp: Unstable bladder
As patients age, the incidence of vesicle instability or unstable bladder increases dramaticall
estrogen has been reported to decrease urgency, frequency, and nocturia in menopausal wo
on correction of stress urinary incontinence or vesicle instability is unclear. In the elderly pop
are also many transient causes of incontinence that the physician should consider. These inc
dementia, medications (especially a-adrenergic blockers), decreased patient mobility, endoc
abnormalities (hypercalcemia, hypothyroidism), stool impaction, and urinary tract infections

(Q.26) A 26-yrs-old patient has had three consecutive spontaneous abortions early in the second trimester. As part of an eva
this problem, the least useful test would be:
(a) Hysterosalpinogram
(b) Chromosomal analysis of the couple
(c) Endometrial biopsy in the luteal phase
(d) Postcoital test
Your Response : d
Correct Answer : D
Exp: Postcoital test
A major cause of spontaneous abortions in the first trimester is chromosomal abnormalitie
the causes of losses in the second trimester are more likely to be uterine or environmental in
Patients should be screened for thyroid function, diabetes mellitus, and collagen vascular dis
is also a correlation between patients with a positive lupus anticoagulant and recurrent misc
hysterosalpingogram should be performed to rule out uterine structural abnormalities, such
uterus, septate uterus, unicornuate uterus, submucous fibroids, or intrauterine adhesions. E
biopsy is performed to rule out an insufficiency of the uterus. If no abnormalities are found,
husband and wife should be karyotyped to see if a balanced translocation or 45,X mosaicism
A postcoital test is useful for couples who can not conceive, but does not address postconc

(Q.27) Not a feature of HELLP syndrome:


(a) Raised liver enzymes
(b) Eosinophilia
(c) Thrombocytopenia
(d) Hemolytic anemia
Your Response : b
Correct Answer : B
Exp: Eosinophilia
Ref: Dutta, 6th Edition (Revised 2009), Page no 224
HELLP is an acronym derived from the first letters of the terms that describe the following la
findings: Hemolytic anemia, Elevated Liver enzymes, and Low Platelet count. HELLP syndrom
severe pre-eclampsia.

(Q.28) Highest rate of ectopic pregnancy is seen with:


(a) Progestasert
(b) Cu T
(c) Lippes loop
(d) All have same rate
Your Response : a
Correct Answer : A
Exp: Progestasert
The presence of an inert copper-containing or progesterone intrauterine device (IUD) traditi
been thought to be a risk factor for ectopic pregnancy.
However, only the progesterone IUD has a rate of ectopic pregnancy higher than that for wo
any form of contraception. The modem copper IUD does not increase the risk of ectopic pre
Nevertheless, if a woman ultimately conceives with an IUD in place, it is more likely to be an
pregnancy as compared to women not on any IUCDS. The actual incidence of ectopic pregna
use is 3-4%.
Progestasert is a third generation IU(D).
It is a T shaped device filled with progesterone. The hormone is released slowly in the uter
of 65 mcg daily. It has a direct local effect on the uterine lining, on the cervical mucus and
the sperms.

(Q.29) Which of the following is NOT an assisted reproduction technique


(a) ZIFT
(b) GIFT
(c) IVF and embryonal transfer
(d) Artificial insemination into the uterus
Your Response : d
Correct Answer : D
Exp: Artificial insemination into the uterus
Assisted reproductive technologies
Couples who have failed to respond to traditional infertility treatments, including those with
severe endometriosis, oligospermia, immunologic or unexplained infertility, may benefit from
fertilization (IVF), gamete intrafallopian transfer (GIFT) and zygote intrafallopian transfer (ZIF
All of the procedures involve ovarian stimulation to produce multiple oocytes
GIFT - involves the placement of sperm and eggs in the uterine tube by laparoscopy or minila
is more invasive than IVF
GIFT is not appropriate for women with severe tubal disease and is less successful than IVF Z
fertilization occurs in vitro, and the early development of the embryo occurs in the uterine t
transfer by laparoscopy or minilaparotomy
A recent development is intracytoplasmic sperm injection (ICSI) which allow fertilization with
sperm, this provides the opportunity for men with severe oligospermia or obstructive azoos
father children.
Artificial insemination in Azoospermia - if azoospermia is present, artificial insemination by
usually result in pregnancy, assuming female function is normal.
Artificial insemination isnt an assisted reproductive technique as it doesnt require any sti
ovaries or testicles . It is simply transfer of sperms to female reproductive tract artificially
state.

(Q.30) Sperms are released during ejaculation from


(a) Seminal vesicles
(b) Epididymis
(c) Testes
(d) Prostate
Your Response : b
Correct Answer : B
Exp: Epididymis
Spermatozoa formed in the testis enter the caput epididymis, progress to the corpus, and fin
thecaudal region, where they are stored. Sperm entering the caput epididymis are incomple
the ability to swim forward (motility) and to fertilize an egg.
It stores the sperm for 23 months. During their transit in the epididymis, sperm undergo m
processes necessary for them to acquire these functions. Final maturation is completed in
reproductive tract (capacitation).
During ejaculation, sperm flow from the lower portion of the epididymis (which functions as
reservoir).
They have not been activated by products from the prostate gland, and they are unable to s
transported via the peristaltic action of muscle layers within the vas deferens, and are mixed
diluting fluids of the seminal vesicles and other accessory glands prior to ejaculation (formin

(Q.31) All are indications of intra uterine insemination except


(a) Vicid cervical mucous
(b) Oligospermia
(c) Tubal blockade
(d) Immune factor of sperms
Your Response : d
Correct Answer : C
Exp: Tubal blockade
IUI is specifically used for male factor infertility and for cervical factor infertility in females. In
the IUI crosses the cervix and the semen sample is directly placed in the uterus Viscid cervica
,oligospermia and immune factor i.e antisperm antibodies in the cervical mucus are all indic
Tubal factor infertility is treated with IVF.

(Q.32) Specific treatment of severe pre eclampsia is


(a) Magnesium sulphate
(b) Termination of pregnancy
(c) Clonidine
(d) Hydralazine
Your Response : b
Correct Answer : B
Exp: Termination of pregnancy
Management includes termination of pregnancy as only this followed by placental delivery w
maternal physiology to normal. Magnesium sulphate is a very toxic drug that is not to be giv
with severe preeclampsia prophylactically as it not an antihypertensive. Magnesium sulpha
for impending or established eclampsia.
Clonidine and hydralazine are antihypertensives to be used in mild preeclampsia.

(Q.33) Pure gonadal dysgenesis will be diagnosed in the presence of


(a) Bilateral streak gonads
(b) Bilateral dysgenetic gonads
(c) One side streak and other dysgenetic gonads
(d) One side streak and other normal looking gonads
Your Response : a
Correct Answer : A
Exp: Pure gonadal dysgenesis- "Phenotypic female"
Clinical features
Bilateral streaks gonads, infantile uterus and fallopian tubes and sexual infantilism
Normal height
Normal 46, XX or 46 XY karyotype
Axillary and pubic hair is scanty, and the internal genitalia consist of mullerian derivative
only
Tumor may develop in the streak gonads, particularly dysgerminoma or gynandroblastoma
in the 46, XY, disorder.

(Q.34) A 20 year old woman gives a history of sharp pain in the lower abdomen for 2-3 days every month approximately 2 w
the menses: The most probable etiology for her pain is
(a) Endometriosis
(b) Dysmenorrhea
(c) Pelvic tuberculosis
(d) Mittelschmerz
Your Response : a
Correct Answer : D
Exp: Mittelschmerz - Midcycle ovulatory pain occurs during oocyte release from an ovarian folicle
produces lateral pain that migrates from one side to the other from month to month.
Cyclic causes - include primary dysmenorrhea in which periods are painful but no pelvic path
Secondary dysmenorrhea due to endometriosis, adenomyosis, or perhaps chronic salpingitis
Gynecological causes of continuous pain include - later stages of endometriosis, adenomyos
prolapse, and chronic salpingitis or pelvic adhesions
Ovarian remment syndrome - occurs after oophorectomy when residual ovarian tissue is tra
retroperitoneal location.

(Q.35) Stain used for maturity assessment of amniotic fluid cells is:
(a) Congo red
(b) Lehman's stain
(c) Sudan red stain
(d) Nile blue sulphate
Your Response : d
Correct Answer : D
Exp: new question
With advancing maturity increasing numbers of cornified skin cells are shed into the amnioti
fetus. Because some come from sebaceous glands they contain lipid which when stained wit
sulphate become orange and when 10% of cells from an amniocentesis sample of fluid stain
pregnancy has reached at least 38 weeks and it has reached term or beyond when 50% reac

(Q.36) Complete or incomplete uterine inversion depends on whether fundus has passed through:-
(a) Cervix
(b) Vagina
(c) Introitus
(d) Hymen
Your Response : a
Correct Answer : A
Exp: Cervix
Types of inversion of uterus:
a. ONE: Complete. Visible outside thecervix.
b. TWO: Incomplete. Visible only at the cervix
Other way to describe inversion is first, second, third, or fourth degree. Acute inversion occu
hrs of delivery and is most common type. Subacute inversion presents from 24 hrs to 4 week
or chronic inversion which presents 4 weeks after delivery.

(Q.37) Single pelvic ala is present in


(a) Robert's pelvis
(b) Naegele's pelvis
(c) Osteomalacia pelvis
(d) Rickets pelvis
Your
b
Response :
Correct
B
Answer :
Exp: Ref: Dutta, 6th Edition (Revised 2009), Page no 347
Naegele's pelvis - Ala on one side is absent
Robert's pelvis - Ala on both sides are absent
Osteomalacic pelvis - the shape of inlet is triradiate
Rachitic pelvis - shape of inlet is reniform
The expected date of delivery can be calculated by
Naegele's rule

(Q.38) Maximum increase in cardiac output during pregnancy is seen in:


(a) 32 weeks
(b) 36 weeks
(c) During labour
(d) Just after delivery
Your Response : a
Correct Answer : D
Exp: Just after delivery
Ref: Dutta, 6th Edition (Revised 2009), Page no 53
The cardiac output starts to increase from 5th wk of pregnancy and reaches its peak ( 40-50%
34 wks. Thereafter it remains static till labour (+ 50%) and immediately following delivery(+
pre-labour values. This increase in cardiac output is due to squeezing out of blood from the u
maternal circulation (auto transfusion) during labour and in the immediate post partum.
Cardiac output returns to the pre labour values by 1hr following delivery and to the pre preg
4 weeks time.

(Q.39) Chorionic villus biopsy is done in all of the following EXCEPT


(a) Neural tube defects
(b) Sickle cell disease
(c) Myotonic dystrophy
(d) Down syndrome
Your Response : a
Correct Answer : A
Exp: Neural tube defects
Ref: Dutta, 6th Edition (Revised 2009), Page no 107, 492
a. Indications of Chorionic villus biopsy :
Karyotyping is the most common indicationDown syndrome
Hemoglobinopathies Sickle cell disease, Thalassemia, etc.
Biochemical studiesGauchers disease, Niemann-Pick disease, Tay-Sachs disease, etc.
Single gene defectsMyotonic dystrophy, Cystic fibrosis, Huntingtons disease, Phenylketonur
etc.
Neural tube defects are diagnosed by ultrasound and estimation of alpha-fetoprotein in mater
serum/ amniotic fluid.
b. Chorionic villous biopsy can be done after 10th week (earlier than amniocentesis)
Earlier results reduce maternal stress by going for a possible therapeutic abortion (if indicate
at an early stage.
c. Initially a trans-cervical route is acceptable to take the sample.
A trans-abdominal route is taken if the pregnancy exceeds 12th week.

(Q.40) Technique used for aspiration of sperm directly from testes for in vitro fertilization is:
(a) TESA
(b) MESA
(c) GIFT
(d) IVF
Your Response : b
Correct Answer : A
Exp: TES(A).
Sperm aspiration technique involves the use of minor surgical procedures to collect sperms f
within the genital tract. In men in whom transport of sperms is not possible.
This may involve aspiration of sperms from the vas deferens, epididymis, or testicles.
In vitro fertilization is required to achieve pregnancy with the majority of these extraction pr
Procedure Source IVF
Vasal aspiration Vas deferens May be
Epididymal aspiration Epididymis Yes
Testicular aspiration Testicle Yes
Epididymal aspiration: is of two types:
- MESA: Microsurgical Epididymal Sperm
Aspiration
- PESA: Percutaneous Epididymal Sperm Aspiration
This is performed in situations in which vas is either not present or is scarred from prior surg
or infection.
Testicular sperm extraction (TESE, TESA)
The newest of the three aspiration techniques, testicular sperm retrieval is a breakthrough a
demonstrates that sperms donot have to mature and pass through epididymis in order to fe
Because of their immaturity, however, testicular sperms need to be injected directly into the
(intra cytoplasmic sperm injection) for fertilization to occur. Testicular sperm extraction is in
patients in whom there is blockade in the epididymis very close to where it attaches to the t
blockade with in ducts that conduct sperm out of the testis.
IVF( in vitro fertilization)
In this induction of ovulation is done followed by aspiration of mature oocytes under ultraso
50000 selected sperms are used for insemination and at 2 to 4 cell stage, three embryo tran
uterine cavity 1 cm below the fundus is performed.
GIFT(gamete intra fallopian transfer technique)
It involves aspiration of oocytes following ovulation induction. These are mixed with 50,000
sperms and transferred to ampullary portions of both the fallopian tubes 4 cm from the fimb
MESA and TESA are both techniques of sperm aspiration but since the question specifically a
aspiration directly from the testes, TESA is the most appropriate ans here.

(Q.41) All of the following are ultrasonographic fetal growth parameters except
(a) Head circumference
(b) Trans cerebellar diameter
(c) Femur length
(d) Biparietal diameter
Your Response : a
Correct Answer : B
Exp: Trans cerebellar diameter
Ultrasound imaging: Standard morphometric measurements are as under: Fetal crown-rump
trimester); Biparietal diameter; Head circumference;
Abdominal circumference; Femur length; and Total intrauterine volume.
(Q.42) The lesion listed below that would most likely pursue a benign course is :-
(a) Vaginal adenosis
(b) Adenomyosis
(c) Extramammary Pagets disease
(d) Endometrial hyperplasia
Your Response : a
Correct Answer : B
Exp: Adenomyosis
Adenomyosis is the presence of benign glands and stroma within the myometrium. It is a co
of diffuse uterine enlargement and menstrual irregularities; however, it is not a precancerou
Vaginal adenosis is a DES-related precursor lesion for clear cell adenocarcinoma- of the va
Extramammary Pagets disease is an intraepithelial adenocarcinoma. Endometrial hyperplas
precursor lesion of endometrial carcinoma. Anadenoacanthoma is an endometrial adenocar
benign component of mature squamous cells.

(Q.43) All are risk factors of pre eclampsia except


(a) Diabetes in pregnancy
(b) Hydatidiform mole
(c) Primipara
(d) Previous history of pre eclampsia
Your Response : a
Correct Answer : A
Exp: Diabetes in pregnancy
Primiparous is at a higher risk than multiparous lady. H.mole due to abnormal trophoblastic
and uterine distension is at higher risk. Infact h.mole is associated with early onset preeclam
h/o preeclampsia also increases risk .

(Q.44) Fibroid causes all the following except


(a) Infertility
(b) Amenorrhea
(c) Pelvic mass
(d) Menorrhagia
Your Response : b
Correct Answer : B
Exp: Amenorrhea
Symptoms of fibroid are:
Menorrhagia, polymenorrhea and metrorrhagia
Infertility, recurrent abortions
Pain
Abdominal lump
Pressure symptoms
Vaginal discharge
Around 50% women are asymptomatic.
Important points:
These are benign neoplasms occurring in 5-20% of women in reproductive age group.
Oestrogen dependence on growth is most imp etiological factor, so rare before puberty and
menopause.
Calcification begins at periphery where as degeneration begins in central portion.
Distribution of myoma in body of uterus is intramural (75%), submucous (15%) and subserou
Red degeneration is seen in 2nd half of pregnancy and needs conservative management.
Sarcomatous change is seen in 0.5% of all myomas. It occurs in post menopausal women an
characterized by sudden growth, pain and post menopausal bleeding.
Endometrial ca is associated with fibromyomas in women over 40 yrs of age in 3% cases.

(Q.45) Minimum dose of estrogen in combined OCP's


(a) 20mcg
(b) 30cg
(c) 35mcg
(d) 50mcg
Your Response : b
Correct Answer : A
Exp: 20mcg
A low-dose OCP contains < 50 mcg Ethinyl estradiol .
30 to 35 mcg preferred.
20 mcg pill(ultra low dose) associated with increased bleeding, perhaps less bone mineral d
and no health benefits 50 mcg indicated only if medication interactions may increase hepati
metabolism (DMPA may be a better choice in these patients) .

(Q.46) Nulliparity and late menopause are risk factors for which of the following group of diseases
(a) Cervical cancer and endometrial hyperplasia
(b) Ovarian cancer and endometrial polyp
(c) Vulvar cancer and endometrial cancer
(d) Endometrial cancer and breast cancer
Your Response : d
Correct Answer : D
Exp: Endometrial cancer and breast cancer
Nulliparity and late menopause are factors associated with increased exposure to estrogen,
are operative in endometrial hyperplasia, Unopposed estrogen stimulation is not operative i
polyps, cervical cancer, or vulvar cancer

(Q.47) Tumor marker in dysgerminoma


(a) AFP
(b) HCG
(c) LDH
(d) Ca-125
Your Response : a
Correct Answer : C
Exp: LDH
AFP------Yolk Sac tumor or Endodermal sinus tumor
HCG-----Choriocarcinoma
CA-125---Epithelial ovarian cancer
LDH-----Dysgerminoma (Also raised in HELLP syndrome).Additional tumor marker for
Dysgerminoma is PLAP

(Q.48) Which of the following is correct for the calculation of Pearl index?
(No. of accidental pregnancies x 1200) / (No. of patients observed x months
(a) of use)
(b) (No. of accidental pregnancies x 1200) / (No. of patients observed x 2400)
(c) (No. of patients observed x months of use) / (No. of accidental pregnancies)
(d) (No. of patients observed) / (No. of accidental pregnancies x 2400)
Your Response
a
:
Correct Answer
A
:
Exp: (No. of accidental pregnancies x 1200) / (No. of patients observed x months
of use)
Pearl Index: Failure rate of any contraceptive is calculate din terms of
pregnancy rate per hundred women years of use. It is calculated by
following formula:
Number of accidental pregnancy x 1200
Pearl index = --------------------------------------------------
Number of patients observed x months of use
Here 1200 is the number of months in 100 years

(Q.49) There is a mid-cycle shift in the basal body tempt. after ovulation in women. This is caused by:
(a) FSH-peak
(b) LH-peak
(c) Estradiol
(d) Progesterone
Your Response : d
Correct Answer : D
Exp: Progesterone
Biphasic changes in basal body temperature are characteristic of the ovulatory cycle and are
alterations in progesterone levels. An increase in basal body temperature by 0.3 to 0.5C beg
ovulation, persists during the luteal phase, and returns to the normal baseline (36.2 to 36.4
onset of the subsequent menstruation.

(Q.50) A 24-year-old patient presents to OPD at 18 weeks of pregnancy with a previous history of a child with Down syndrom
appropriate is
(a) Triple test
(b) Chorionic villous biopsy
(c) Amniocentesis
(d) Cordocentesis
Your Response : b
Correct Answer : C
Exp: Amniocentesis
"Tripple test gives risk ratio and for further confirmation amniocenteses has to be done" Trip
(MSAFP,hCG, UE3) it is performed at 15-18 weeks while amniocenteses in performed at 16 w
pregnancy
The ultrasound nuchal translucency (NT) is now appreciated as a sensitive marker for Down
screening and other aneuploidies between 10 and 13 weeks.
Outside that range, the NT disappears. Although some centers have had superb results, othe
done well.
Blood free -hCG and PAPP-A in the first trimester, and double (AFP and hCG) or triple (AFP, h
at 15 to 20 weeks) evaluations are statistically comparable. The combination of NT and first-
biochemistry will likely be the optimal approach.
Biochemistry does not work well for multiple gestations. Ultrasound can also detect structur
but often high-quality ultrasound services require patients to travel long distances, whereas
shipped from essentially anywhere to a competent lab.
Quadruple test is use o Inhibin along with the standard three parameters
Cordocentesis (Percutaneous umbilical cord sampling) usually performed after 18 weeks ges
(Q.51) Most common serotype of HPV associated with invasive cervical carcinoma is?
(a) HPV 16
(b) HPV 18
(c) HPV 32
(d) HPV 36
Your Response : a
Correct Answer : A
Exp: HPV 16
HPV is associated with 80% of CIN and over 90% of invasive carcinoma
'HPV -16 is most common HPV associated with squamous cell carcinoma. HPV -18 is most co
associated with Aden carcinoma.' -CGDT 9th ed pg 904
As it is well known that histologically 95% of cervical ca are SCC and only 5% adenoca. Hence
most common serotype of HPV associated with cervical ca.
On the basis of strength of association to cacervix, HPV has been classified into:
Low risk type-6, 11, 42, 43
Causes CIN and condyloma acuminate
Intermediate risk-33, 35, 55, 52
Causes CINI and CIN that don't progress.
High risk- 16, 18,31,39,45,56,48,59
Causes high grade CIN that progresses to invasive ca.

(Q.52) "Turtle neck sign" is associated with;


(a) Anencephaly.
(b) Extended breech.
(c) Shoulder dystocia.
(d) Congenital goiter.
Your Response : d
Correct Answer : C
Exp: Shoulder dystocia
Ref: Dutta, 6th Edition (Revised 2009), Page no 406
Inability to feel the neck of the baby, when fetal head retracts/recoils against the maternal p
external rotation is not accomplished as such there is inability to deliver the shoulders after
been delivered. This is an important sign to diagnose 'shoulder dystocia' after full dilatation
this sign is known as turtle neck sign.

(Q.53) A 32-week-pregnant woman with a history of two previous mid trimester abortions presents with regular contraction
The cervix is 2 cm dilated. All the following are done EXCEPT
(a) Tocolytics
(b) Dexamethasone
(c) Antibiotics
(d) Emergency encirclage
Your Response : d
Correct Answer : D
Exp: Emergency encirclage
Glucocorticoids (Dexamethasone) is advocated where the pregnancy is less than 34 weeks, t
fetal lung maturation so that the incidence of respiratory distress can be minimized
Antibiotics - Prophylactive antibiotics are given to minimize maternal and perinatal risk of in
ampicillin
Tocolytics - arrest the preterm labour and inhibit uterine contractions
Circlage operation - the operation should be done around 14 weeks of pregnancy or at leas
earlier than the lowest period of previous wastage, as early as the 10th weeks
Second trimester abortion - mostly due to cervical incompetence.

(Q.54) A 16-year-old girl presents with blind vaginal pouch with absence of uterus. Investigation to be done is
(a) Prolactin levels
(b) Karyotyping
(c) IVP
(d) FSH levels
Your Response : b
Correct Answer : B
Exp: Karyotyping - Indications
Patients with malformations suggestive of one of the recognized syndromes associated with a
chromosome aberration
Patients of any age who are grossly retarded physically or mentally especially if there are asso
anomalies
Any patients with ambiguous internal or external genitalia or suspected hermaphroditisms
Girl with primary amenorrhoea and boys with delayed pubertal development.
Males with learning or behavioral disorders who are taller than expected (Based on parental
Certain malignant & pre malignant disease
Parents of a patients with chromosomal translocations
Couples with history of multiple spontaneous abortion of unknown cause
Couples who are infertile after more common obstetric and urologic causes have been exclud
Prenatal diagnosis - advanced maternal age, previous child with chromosome aberration, intr
growth delay

(Q.55) DMPA causes all the following EXCEPT


(a) Weight gain
(b) Irregular cycles
(c) Amenorrhoea
(d) Thromboembolism
Your Response : c
Correct Answer : D
Exp: Thromboembolism
Draw backs of DMP A is - frequent irregular menstrual bleeding, spotting and amenorrhoea are
common, weight gain.

(Q.56) True regarding changes in respiratory system in pregnancy:


(a) Respiratory rate increase.
(b) Vital capacity remains unchanged.
(c) Tidal volume increase.
(d) Total lung capacity increases.
Your Response : a
Correct Answer : C
Exp: Tidal volume increase.
Ref: Dutta, 6th Edition (Revised 2009), Page no 55
With the enlargement of the uterus, especially in the later months, there elevation of the dia
and breathing becomes diaphragmatic. The subcostal angel increases from 68-103 degrees;
diameter of the chest expands by 2 cm and the chest circumference increase by 5-7 cm. A st
hyperventilation occurs during pregnancy leading to increase in tidal volume and therefore i
respiratory minute volume by 40%.
Features Non-pregnant Pregnancy at term Chang
Respiratory rate/min 15 15 Unaffec
Vital capacity 3200 3300 Almost unc
Tidal volume (ml) 475 675 +40%
Residual volume (ml) 965 765 -20%
Total lung capacity 5000 4750 -8%

(Q.57) A 17 year old unmarried girl presents with fever, nausea, vomiting, pallor and pain abdomen. She is having foul smell
stained discharge per vaginum. Your most likely diagnosis of this case IS
(a) Twisted ovarian cyst
(b) Typhoid with enteric perforation
(c) Pelvic abscess with septic peritonitis
(d) Ruptured ectopic pregnancy
Your Response : d
Correct Answer : C
Exp: Pelvic abscess with septic peritonitis
It is a case of septic abortion complicated by septic peritonitis and pelvic abscess. As the pat
unmarried she had some procedure as criminal abortion to terminate unwanted pregnancy.
twisted ovarian cyst, fever and vaginal discharge are absent. In ruptured ectopic pregnancy,
are there but fever is usually absent and vaginal discharge is not foul smelling, there is only s
per vaginam.

(Q.58) All the conditions mentioned below are associated with decreased fetal heart rate except
(a) Fetal movement
(b) Chronic hypoxia
(c) Cord compression
(d) Head compression
Your Response : a
Correct Answer : A
Exp: Fetal movement
Fetal movement is associated with increase in fetal heart rate .This forms the basis of non st
compression leads to vagal stimulation and thus bradycardia. Hypoxia and cord compression
hypoxia) will also lead to bradycardia.

(Q.59) With reference to fetal heart rate, a non-stress test is considered reactive when:
(a) Two fetal heart rate accelerations are noted in 20 mins
(b) One fetal heart rate acceleration is noted in 20 mins
(c) Two fetal heart rate accelerations are noted in 10 mins
(d) Three fetal heart rate accelerations are noted in 30 mins
Your Response : a
Correct Answer : A
Exp: Two fetal heart rate accelerations are noted in 20 mins
Ref: Dutta, 6th Edition (Revised 2009), Page no 609
Non-Stress Test: It is a non-invasive test designed to monitor fetal responses to movements
fetal heart rate with perceived fetal movements.
With movement, the fetus will increase its heart rate and should maintain this increase for s
seconds. If the movement stresses the fetus, the heart rate will drop quickly.
This test monitors the acceleration of heart rate and whether the fetus becomes stressed or
mother is asked to lay down for while the fetal heart rate is recorded with a disc-shaped Dop
ultrasound strapped around her abdomen, similar to the Doppler equipment used to listen t
heart rate.
The Doppler is attached to a monitor that graphs the fetal heart rate. She will report each tim
the baby move by pressing a button that makes an arrow on the graph. A normal fetus will r
rate at least 15 beats whenever it moves. If the baby shows two accelerations in heart rate l
15 seconds during the 20-minute test, it is read as normal or "reactive."
A reactive non-stress test (NST) is a reassuring sign of fetal well being. If the heart rate does
with movement or another 20 minutes of observation fails to meet the criteria, and then the
test is considered "non-reactive."

(Q.60) Gold standard for diagnosis of PID?


(a) Clinical triad of Pain, Fever and Cervical tenderness
(b) Histologic confirmation of Endometritis
(c) Diagnostic Laparoscopy
(d) USG
Your Response : b
Correct Answer : C
Exp: Diagnostic Laparoscopy
Visualization of purulent exudate coming through fimbrial end of fallopian tube is diagnostic
The best material for culture is this exudate. Thus laparoscopy is the gold standard though d
mostly clinical. Laparoscopy is also the gold standard for ectopic pregnancy, ovulation, endo

(Q.61) The following statements are true about intrauterine devices (IUD) except:
(a) Levonorgestrel releasing IUD has an effective life of 5 years
(b) IUD can be used for emergency contraception within 5 days
(c) The pregnancy rate of Lippes loop and Cu- T 200 are similar
(d) Multiload Cu-375 is a third generation IUD
Your Response : b
Correct Answer : D
Exp: USG
Ref: Dutta, 6th Edition (Revised 2009), Page no 549
Generation Device Example
First generation Non-medicated or inert Lippes Loop
IUDs
Second generation Copper IUDs Cu-7, Cu T 200; T Cu-220 C, T Cu-380 A
or
Ag; Nova T; Multiload devices (ML - Cu -
250 & ML- Cu -375)
Third generation Hormone releasing IUDs Progestasert

(Q.62) Posterior colpotomy is done in


(a) Pelvic abscess
(b) Ovarian abscess
(c) Hematocolpos
(d) Tubo-ovarian mass
Your Response : b
Correct Answer : A
Exp: Pelvic abscess
Pelvic abcess that is dissecting through rectovaginal septum is treated with posterior colpoto
procedure is done vaginally and the incision is made at the point of maximum bulge. This he
of the pus or it can burst into the rectum or into abdomen resulting in generalised peritoniti
Hematocolpos is seen with imperforate hymen and is treated with incision and drainage. Th
incision is typically cruciate.

(Q.63) An ectopic pregnancy is shed as


(a) Decidua vera
(b) Decidua basalis
(c) Decidua capsularis
(d) Decidua rubra
Your Response : c
Correct Answer : A
Exp: Decidua vera
Ref: Dutta, 6th Edition (Revised 2009), Page no 24
The endometrial lining of the uterus is called decidua during pregnancy, If there is fertilizatio
pregnancy occurs, the decidua differentiated into
Decidua basalis - becomes the maternal portion of the placenta
Decidua - capsularis - the thin layer covering the ovum
Decidua vera or parietalis - which is the rest of the decidua lining the uterine cavity outside t
implantation
*The decidua develops all the characteristic of intra uterine pregnancy except that it contain
of chorionic villl
When the ovum is dead it is either disintegrated and comes out piecemeal or comes in a sing
(decidual cast)
Functions of decidua
Provide the nidus for implantation of the fertilized ovum
Provide nutrition (glycogen, fat) to the growing ovum
The growing substance of the decidua consisting of mucopolysaccharides limits deeper pene
trophoblastic cells, a protective action
Provide the basal plate of the placenta.

(Q.64) Clomiphene citrate is indicated in


(a) Stein-Leventhal syndrome
(b) Ashermans syndrome
(c) Carcinoma endometrium
(d) All of the above
Your Response : d
Correct Answer : A
Exp: Stein-Leventhal syndrome
If the patient wishes to become pregnant, clomiphene or other drugs can be employed for in
ovulation for female infertility
Clomiphene - can sometimes stimulate a man's own pituitary gonadotropins (when his pitui
there by increasing testosterone and sperm production for male infertility
Clomiphene citrate is non steroidal estrogen antagonist it induces ouulation in 60% of wome
Ieventhal syndrome (PCOS) and is the initial treatment of choice. Adverse effects - Poly cysti
multiple pregnancy hot flushes, gastric upset, vertigo, allergic dermatitis, risk of ovarian tum
increased

(Q.65) Endometrial hyperplasia is seen in


(a) Endodermal sinus tumour
(b) Dysgerminoma
(c) Polycystic ovarian disease
(d) Carcinoma of cervix
Your Response :
Correct Answer : C
Exp: Polycystic ovarian disease
Polycystic ovarian disease - endometrial hyperplasia due to high content of estrogen, largely
extraglandular aromatization of circulating androstenedione.
Endometrial hyperplasia is common in carcinoma endometrium
Treatment - simple endometrial hyperplasia calls for cyclic progestin therapy
(medroxyprogesteroneor norethindrone)
If endometrial hyperplasia with atypical cells or carcinoma of the endometrium is found, hys
necessary

(Q.66) All the following are indications for termination of pregnancy in an APH patient EXCEPT
(a) 37 weeks
(b) IUD
(c) Transverse lie
(d) Significant bleeding
Your Response : d
Correct Answer : C
Exp: Transverse lie
The definite management - comprises prompt delivery, this is considered when ever
The patient has her first bout of bleeding after 37 completed weeks. Successful conservative
brings the patients upto 37 weeks. If the initial or a subsequent bout of bleeding is very seve
Patient is in labour
Evidence of maternal or fetal jeopardy
Intrauterine fetal death

(Q.67) All the following are TRUE about HCG EXCEPT


(a) Has luteotrophic action
(b) Acts on the same receptors as LH
(c) Secreted by cytotrophoblast
(d) It is a glycoprotein
Your Response : c
Correct Answer : C
Exp: Secreted by cytotrophoblast
Human chorionic gonadotrophin (hCG) is a glycoprotein
It consists of a hormone non specific a subunit is biochemically similar to LH, FSH and TSH
Hormone specific ~ subunit is relatively unique to hCG
hCG is produced by syncytiotrophoblast of the placenta and secreted into the blood of
both mother and fetus

(Q.68) Fetal hydrops is most commonly associated with:


(a) Cardiac anomalies.
(b) Renal anomalies.
(c) GI anomalies.
(d) Skeletal anomalies.
Your Response : a
Correct Answer : A
Exp: Cardiac anomalies
Ref: Dutta, 6th Edition (Revised 2009), Page no 333, 497
Cardiac anomaly is the most common anatomical abnormality associated with fetal hydrops
hydrops associated with a structurally abnormal heart carries a poor prognosis, as the anom
severe. Those hydropic fetuses with arrhythmia but with structurally normal heart usually re
either transplacental or direct fetal therapy. In these cases the prognosis is good.
(Q.69) Paramesonephric duct develop into:
(a) Vas deferens
(b) Seminal vesicle
(c) Ureter
(d) Uterus
Your Response : a
Correct Answer : D
Exp: Uterus
Male Female
Mesonephric duct Duct of epididymis Duct of epoophoron
(Wolffian duct) Ductus deferens Part of bladder and ure
Ejaculatory duct
Part of bladder and prostatic
urethra
(Paramesonephric or Mullerian duct) Appendix of testis Uterine tube
Prostatic utricle Uterus
Vagina (?)

(Q.70) In puberty menorrhagia the incorrect statement is


(a) Associated with anovulatory bleeding
(b) Endometrial biopsy confirms diagnosis
(c) Routine screening for bleeding disorder
(d) Hormone and Hematonics are given
Your Response : d
Correct Answer : B
Exp: Endometrial biopsy confirms diagnosis
Puberty menorrhagia is mostly due to anovular cycles resulting in deficiency of progesteron
poorly supported and structured endometrium that bleeds. Invasive procedure is strictly con
. If the teenager gives h/o sexual activity then a urine pregnancy test is a must even in the ab
preceding amenorrhoea to r/o pregnancy and its complications. It is also a part of the protoc
bleeding disorders especially if menorrhagia follows an episode of viral illness or if it occurs a
puberty. The treatment is with hormones and hematinics to prevent anaemia. Irregular ripen
shedding of endometrium are types of ovulatory DUB which can also lead to puberty menor

(Q.71) Following suction evacuation for 10 weeks old missed abortion the lady presented with bleeding PV and pain after 4 d
probable cause would be?
(a) Uterine atony
(b) Cervical trauma
(c) Retained placental bits
(d) Uterine rupture
Your Response : c
Correct Answer : C
Exp: Retained placental bits
Incomplete abortion or retained products will present in this manner. Uterine atony and cer
would present preoperatively . Uterine perforation should also be identified at the time of p
sudden loss of resistance/ pressure. If missed and presenting later the patient would presen
abdominal symptoms as the risk of bowel injury is higher with suction and evacuation.

(Q.72) The best investigation to diagnose carcinoma cervix in early stages


(a) Colposcopic biopsy
(b) Acetowhite over lesion
(c) Pap smear
(d) Hysteroscopy
Your Response : c
Correct Answer : C
Exp: Pap smear
Pap smear is a screening test. It is the initial test in all patients unless a visible growth is pres
to be biopsied. Colposcopy is also a screening test that is done in well equipped centres . It i
needs more expertise. Acetowhite lesion has many differentials ranging from leukoplakia, SI
infection to metaplasia. This technique is used in periphery were there is no facility of Pap sm
colposcopy to identify the high risk cases. Hysteroscopy is for diagnosis of uterine lesion. Of
listed here, Pap smear is the best answer.

(Q.73) Which of the following findings characterizes a normal semen sample?


(a) Agglutination
(b) Sperm concentration of 35 million per mL
(c) 5% normal sperm morphology
(d) 10% progressive sperm motility
Your Response : b
Correct Answer : B
Exp: Sperm concentration of 35 million per mL
Because of the variability in semen specimens from the same person, preferably three speci
be evaluated over the course of an investigation for infertility.
Semen Parameters
volume >2.0 mL, pH 7.2 7.8,
Concentration > 20x106/ml, Motility > 50%, Morphology > 30% with normal morphology

(Q.74) "INFIBULATION" refers to


(a) Double vagina
(b) Elephantiasis vulvae
(c) Female circumcision
(d) Rudimentary cervix.
Your Response : c
Correct Answer : C
Exp: Female circumcision
Infibulation refers to extreme form of genital mutilation. It includes removal of clitoris, labia
most of labia majora only a small opening is left for menstrual blood and urine to pass. The m
immediate danger is severe infection and urinary incontinence.

(Q.75) Rubin's diagnostic criteria is used for:


(a) Overian pregnancy.
(b) Hydatiform mole.
(c) Cervical pregnancy.
(d) Abdominal pregnancy.
Your
b
Response :
Correct
C
Answer :
Exp: Cervical pregnancy
Ref: Dutta, 6th Edition (Revised 2009), Page no 193
Rubin's criteria, given by Rubin in 1983 for diagnosis of cervical
pregnancy are as follows
Soft enlarged cervix to or equal to or larger than fundus.
Uterine bleeding following amenorrhea without uterine cramping.
Products of conception entirely within or firmly attached to endocervix.
Closed internal os, partially opens external os.

(Q.76) Inguinal hernia in male infants is caused by which congenital infection:


(a) CMV.
(b) Toxoplasma.
(c) Rubella.
(d) Congenital syphilis.
Your Response : d
Correct Answer : A
Exp: CMV
Manifestations of CMV in the neonates include focal or generalized organ involvement. Thes
petechiae, hepatosplenomegaly, jaundice, microcephaly, IUGR, Chorioretinitis and inguinal h
infants.

(Q.77) Endometrial carcinoma is caused by all the following EXCEPT


(a) Unopposed oestrogen
(b) Tamoxifen
(c) Oral contraceptives
(d) Irradiation
Your Response : d
Correct Answer : C
Exp: Oral contraceptives
Anetiological factors of endometrial carcinoma
Peri menopausal (Age 55-60 years]
Common in nulliparous
The prolonged use of exogenous estrogen (and Tamoxifen) in post menopausal women mult
of endometrial cancer about seven times
Obesity hypertension, DM
Family history positive
DUB
Feminizing ovarian tumors, fibroids and PCOD
Endometrial hyperplasia
Continuous irradiation may damage the endometrium and leads to endometrium carcinoma

(Q.78) A patient with positive antiphospholipid antibodies would have all the following EXCEPT
(a) Recurrent foetal loss
(b) Venous thrombosis
(c) Thrombocytosis
(d) Neurological complications
Your Response : c
Correct Answer : C
Exp: Thrombocytosis
Ref: Dutta, 6th Edition (Revised 2009), Page no 169, 343, 636
Common obstetric complications associated with antiphosphoid syndrome
Recurrent fetal loss
IUGR
PIH venous thrombosis)
Placental abruption (2nd trimester
Recurrent thrombotic events (Arterial
Thrombocytopenia
Auto immune or connective tissue disease
Many of these patients have SLE like symptoms, but do not meet specific diagnostic
criteria for that disease
Livedo reticularis, skin ulcers, mental status changes and mitral regurgitation are also
noted

(Q.79) In Cardiotocography, the first sign of fetal hypoxia is:


(a) Loss of baseline variability.
(b) Persistent deceleration.
(c) Variable deceleration.
(d) Absence of acceleration.
Your Response : c
Correct Answer : D
Exp: Absence of acceleration
Ref: Dutta, 6th Edition (Revised 2009), Page no 609
Absence of acceleration is the first indication of onset of gradual hypoxia; Decreased beat to
variability also denotes fetal hypoxia, fetal sleep, medications and infections.

(Q.80) All are the criteria for medical management of ectopic pregnancy
except:
(a) Asymptomatic patient.
(b) Serum r3-hCG level < 2000 IU/L.
(c) Pregnancy diameter <5 cm.
(d) Unruptured tube.
Your
b
Response :
Correct
C
Answer :
Exp: Pregnancy diameter <5 cm
Criteria are:
Asymptomatic patient.
Serum -hCG level < 2000 IU/L.
Pregnancy diameter <2 cm.
Unruptured tube.
Non-active bleeding.
No fetal cardiac activity on USG.
< 100 ml blood in the pouch of Douglas.

(Q.81) The most common cause of female pseudohermaphroditism is


(a) Congenital adrenal hyperplasia
(b) Ovarian dysgenesis
(c) Virilizing ovarian syndrome
(d) Exogenous androgen
Your Response : a
Correct Answer : A
Exp: Congenital adrenal hyperplasia
Female pseudo hermaphroditism - in female, congenital adrenal hyperplasia due to 21 hydro
deficiency results in female pseudohermaphroditism because the disorder of steroidogenesi
in fetal life, there is almost always evidence of some degree of masculinization at birth.
It is manifested by enlargement of the clitoris, and varying degrees of labial fusion's, the vag
a common opening with the urethra (urogenital sinus)

(Q.82) Reproductive Risk index includes all except:


(a) Maternal mortality rate.
(b) Birth rate.
(c) Prevalence of anemia among pregnant population.
(d) Post partum complications.
Your Response : c
Correct Answer : D
Exp: Post partum complications.
Reproductive Risk index was introduced by population assessment International (PAI) as a re
assess the risk in 133 countries comprising of world population. It includes(a) Birth rate; (b) M
fertility rate; (d) Prenatal Care; (e)% of birth attended by skilled attendants;(f)% of HIV infect
partners;(g) anemia prevalence;(h) prevalence of contraception;(i) MTP policies.

(Q.83) Unsatisfactory colposcopy means?


(a) Unable to visualize cervix
(b) Unable to visualize transformation zone
(c) Unable to visualize ecto cervix
(d) Unable to visualize endo cervix
Your Response : b
Correct Answer : B
Exp: Unable to visualize transformation zone
Colposcopy is said to be satisfactory if the entire transformation zone is visualized. It is said t
unsatisfactory if this does not happen . The reason could be difficulty in visualisation of cer
continuous bleeding from the cervix which precludes its assessment, transformation zone h
into endocervical canal as in postmenopausal women.

(Q.84) The level of which hormone is unchanged in pregnancy:


(a) Prolactin.
(b) Growth hormone.
(c) ACTH.
(d) Plasma vasopressin.
Your Response : b
Correct Answer : D
Exp: Plasma vasopressin
The levels of ACTH, CRR, and Prolactin increase; the level of GH increases during pregnancy d
variant syncytiotrophoblast; Plasma vasopressin levels remain unchanged.

(Q.85) Least important in the Treatment of menorrhagia?


(a) NSAIDS
(b) Tranexamic acid
(c) Norethisterone
(d) Clomiphene
Your Response : a
Correct Answer : D
Exp: Clomiphene
Conservative - Rest, sedatives, reassurance, oral iron, blood transfusions
Hormones therapy - oestrogen, progestational steroids (Norethynodrel, norethisterone)
Danazol, OCP, Testosterone
NSAIDS, - Mefenamic acid
Antifibrinolytic agents - Tranexamic acid
GnRH agonist, ethamsylate, radiotherapy
Clomiphene - may be advocated if pregnancy is desire and if cycles are anovulatory

(Q.86) Regarding Hyperemesis gravidarum false is :


(a) Allergic basis is suggested.
(b) Hormonal imbalance may be a cause.
(c) Small heart is an incidental finding.
(d) May show manifestations of Wernickes encephalopathy.
Your Response : a
Correct Answer : C
Exp: Small heart is an incidental finding
Small heart is a constant finding; there may be subendocardial hemorrhage; small hemorrha
hypothalamic region giving the manifestations of Wernickes encephalopathy. Etiology inclu
psychogenic, dietetic and allergic basis.

(Q.87) Pressure from the fibroid may also cause?


(a) Leg ulcers
(b) Peau de orange
(c) Superficial thrombophlebitis
(d) Varicose veins
Your Response : d
Correct Answer : D
Exp: Varicose veins
Compression of pelvic vasculature by a markedly enlarged uterus may cause varicosities or e
(swelling) of the lower extremities. Peau' de orange is a skin edema that occurs in breast can
are common in patients with a history of advanced diabetes or peripheral vascular disease. S
thrombophlebitis may result from stasis of blood in this patient, but it is not as common as v
varicosities. Deep venous thrombosis is not associated with leiomyomas

(Q.88) Which of the following statement is true concerning Galactorrhea:-


(a) Serum prolactin (PRL) is above 20 ng/ml
(b) Pituitary adenoma worsens the prognosis
(c) Associated with menstrual disturbances & infertility
(d) All of the above
Your Response : d
Correct Answer : D
Exp: All of the above
A number of drugs are known to cause galactorrhea, example: Methyldopa, reserpine, amph
metoclopramide, phenothiazine, tricyclic antidepressant, butyrophenones, and Isoncx.

(Q.89) A 20 year old primi at 32 weeks of gestation has polyhydraminos. She should be advised:-
(a) Bed Rest
(b) Artificial rupture of membranes
(c) Oral indomethacin
(d) Restriction of oral fluid
Your Response : d
Correct Answer : C
Exp: Oral indomethacin
Ref: Dutta, 6th Edition (Revised 2009), Page no 213
Medical treatment of hydramnios is indomethacin in dose of 2mgmlkg/day orally. It reduces
by foetal kidney. Such therapy may cause premature closure of ductus arteriosus. If the pat
respiratory distress due to hydramnios, then amniotic fluid is drained by amniocentesis. The
drainage is maintained at about 500 ml/hr, and a total of 1500 to 2000 ml is withdrawn such
drainage docs not increase the risk of placental abruption. Amniocentesis may cause entry o
organism into the amniotic cavity & development of chorioamnionitis which should be watc

(Q.90) A 28 year old female present with acute abdominal pain, vaginal bleeding, severe pallor, hypotension with amenorrh
weeks. The provisional diagnosis is:
(a) Uterine fibroid
(b) Ovarian cyst
(c) Ruptured ectopic pregnancy
(d) inevitable abortion
Your Response : c
Correct Answer : C
Exp:
Ref: Dutta, 6th Edition (Revised 2009), Page no 179 - 189
Diagnosis is a classical case of ruptured ectopic pregnant)'. Uterine fibroid usually present w
menorrhagia, amenorrhea is absent. Ovarian' cyst until twisted does not cause acute abdom

(Q.91) A 34-year-old female present with copious vaginal discharge with odour. Discharge is gray in color microscopic exami
clue cells. The treatment of choice is:-
(a) Azithromycin
(b) Fluconazole
(c) Metronidazole
(d) Clotrimazole
Your Response : c
Correct Answer : C
Exp: Ruptured ectopic pregnancy
Diagnosis is bacterial vaginosis. Clue cells are vaginal epithelium cells studded with coccobac
Metronidazole is drug of choice, organism is Gardnerella vaginalis. Discharge gives fishy sme
whiff test) on treating with KO(B).

(Q.92) Which of the following test is used to estimate the amount of feto-maternal hemorrhage?
(a) Coombs test
(b) Kleihauer betke test
(c) Lielys spectrophotometer
(d) Schrills test
Your Response : d
Correct Answer : B
Exp: Kleihauer betke test
Kleihauer-Betke test is used to estimate amount of fetal maternal hemorrhage. A maternal b
treated with an acid-dilution procedure allowing identification of fetal blood cells. Combs te
detection of antibodies (Rh) in Rh negative mother as a screening test. Liley spectrophotome
plotting amniotic fluid bilirubin in case of Rh sensitized mother after Amniocentesis. Shrill te
biopsy test in suspected Ca cervix. Gram iodine is used as a Stainer. Healthy' tissue stains bro
carcinomatous are a remains unstained.
(Q.93) In obstretics ultra sound commonly employed range of frequency is:-
(a) 3.5-5 MHZ (Million Cycle per second)
(b) 5 7.5 MHZ
(c) 1 2.5 MHZ
(d) 8 10 MHZ
Your Response : b
Correct Answer : A
Exp: 3.5-5 MHZ (Million Cycle per second)
Ref: Dutta, 6th Edition (Revised 2009), Page no 643
3.5- 5 MHZ is commonly used frequency of ultra sonic waves for obstetric ultra sound. A pie
crystal is used to generate the sound waves.

(Q.94) Point of distinction between partial mole to complete mole is:


(a) Partial mole show trophoblastic proliferation with absent villi
(b) Typical of partial mole is cellular atypia
(c) Partial mole is more prone to tumor malignancy
(d) Partial mole is triploid
Your Response : a
Correct Answer : D
Exp: Partial mole is triploid
Complete mole
A complete mole contains no fetal tissue 90% are 46, XX, and 10% are 46, XV.
All chromosomes are of paternal origin
An enucleate egg is fertilized by a haploid sperm (which then duplicates its chromosomes), o
fertilized by 2 sperm
Chorionic villi have grapelike (hydatidiform) swelling
There is trophoblastic hyperplasia
After a complete mole develops, uterine invasion occurs in 15% of patients, and metastasis o
4%.
Partial mole
Fetal tissue is often present in a partial mole 69, XXX or 69, XXY
This results from fertilization of a haploid ovum and duplication of the paternal haploid chro
from dispermy.
Swelling of the chorionic villi
There is hyperplastic trophoblastic tissue
No cases of choriocarcinoma have been reported after a partial mole, although 4% of patien
moles develop persistent non-metastatic trophoblastic disease

(Q.95) What is the absolute contraindication of using this


instrument?

(a) Before D&C for DUB


(b) Before D&C for Missed abortion
(c) Before placing IUCD
(d) In Manchester operation
Your
Response b
:
Correct
B
Answer :
Exp: Before D&C for Missed abortion

(Q.96) A 25 year old infertile lady is having menorrhagia. On examination an abdominal mass is palpable which is immobile.
examination- uterus and appendages are fixed. The most appropriate first line treatment in this case is
(a) Exploratory laparotomy
(b) Tubal microsurgery
(c) Anti tubercular drugs
(d) Hysterectomy
Your Response : c
Correct Answer : C
Exp: Anti tubercular drugs
Immobile abdominal mass with fixed uterus and appendages, when associated with menstru
an infertile woman, strongly favours the diagnosis of genital tuberculosis. The first line treat
tubercular drugs. Surgery in from of TAH with BSO required as last option. Tubal microsurge
contraindicated in genital tuberculosis. Exploratory laparotomy not done as first line measur
required when there is confusion in diagnosis.

(Q.97) Bilateral ovarian carcinoma with breach in


capsular wall with ascites, peritoneal metastasis & positive cytology belongs to stage?
(a) Ic
(b) IIc
(c) IIIc
(d) IVc
Your Response : d
Correct Answer : A
Exp: Ic
Stage I limited to one or both ovaries IA involves one ovary; capsule intact; no tumor o
surface; no malignant cells in ascites or peritoneal washings
IB involves both ovaries; capsule intact; no tumor on ovarian surface; negative washings
IC tumor limited to ovaries with any of the following: capsule ruptured, tumor on ovarian
positive washings
Stage II pelvic extension or implants IIA extension or implants onto uterus or fallopian
tube; negative washings
IIB extension or implants onto other pelvic structures; negative washings
IIC pelvic extension or implants with positive peritoneal washings
Stage III peritoneal implants outside of the pelvis; or limited to the pelvis with extension t
bowel or omentum IIIA microscopic peritoneal metastases beyond pelvis
IIIB macroscopic peritoneal metastases beyond pelvis less than 2 cm in size
IIIC peritoneal metastases beyond pelvis > 2 cm or lymph node metastases
Stage IV distant metastases to the liver or outside the peritoneal cavity
Para-aortic lymph node metastases are considered regional lymph nodes (Stage IIIC). As ther
para-aortic lymph node intervening before the thoracic duct on the right side of the body, th
cancer can rapidly spread to distant sites such as the lung.

(Q.98) Best ultrasound index SGA fetus is:-


(a) Femur length
(b) Head circumference
(c) Abdominal circumference
(d) CRL
Your Response
d
:
Correct
C
Answer :
Exp: Abdominal circumference
Head circumference is last to be affected. Liver is first organ to be
affected in IUGR babies.

(Q.99) A young woman presented with infertility. Hysterosalpingogram shows beaded fallopian tubes with clubbing of the a
Most likely diagnosis is
(a) Gonococcal infection
(b) Chlamydial infection
(c) Tuberculosis
(d) Herpes genitalis
Your Response : c
Correct Answer : C
Exp: Tuberculosis
Following findings strongly suggests tuberculosis salpingitis in Hysterosalpingo gram
(a) A rigid non peristaltic pipe-like tube, called lead pipe appearance
(b) Beading an variation in filling density
(c) Calcification of the tube
(d) Cornual block
(e) A jagged fluffiness of the tubal outline
(f) Vascular or lymphatic intravasation of the dye
(g) Tobacco-pouch seen at naked eye examination
**In a proven case of genital tuberculosis, hysterosalpingography is contraindicated, as it ma
infection

(Q.100) A 45-year-old female presents with polymenorrhoea for 6 months. On examination nothing abnormal was found. W
following is the next best line of management
(a) Give progesterone for 3 cycles
(b) Give OCP for 3 cycles
(c) Dilation and curettage
(d) Hysterectomy
Your Response : b
Correct Answer : C
Exp: Dilation and curettage
Poly menorrhea - or epimenorrhea, the menstrual cycle is reduced from the normal of 28 d
of two to three weeks and remain constant at that frequency.
Treatment - Dilation and curettage is a most effective short term method of controlling ute
longer term treatments will generally be directed towards the cause of the abnormal uterin
DUB due to chronic anovulation is treated with cyclic progestin therapy with OCP
Severe and intractable bleeding of a dysfunctional nature may rarely require hysterectomy

(Q.101) All the following are associated with breech presentation at full term EXCEPT?
(a) Cornual implantation of the placenta
(b) Congenital malformation
(c) Uterine anomaly
(d) Placenta accreta
Your Response : b
Correct Answer : D
Exp: Placenta Accreta.
Ref: Dutta, 6th Edition (Revised 2009), Page no 375
The following are the known factors responsible for breech presentation. In a significant n
cases, the cause remains obscure.
Prematurity It is the commonest cause of breech presentation.
Factors preventing spontaneous version: (a) Breech with extended legs (b) Twins (c) Oligoh
Congenital malformation of the uterus such as septate or bicornuate uterus (e) short cord,
absolute (e) intrauterine death of the fetus.
Favourable adaptation : (a) Hydrocephalus big head can be well accomdated in the wide f
Placenta praevia (c) Contracted pelvis (d) Cornufundal attachment of the placenta minim
of the fundus where the smaller head can be placed comfortably.
Undue mobility of the fetus: (a) Hydramnios (b) Multiparae with lax abdominal wall.
Fetal abnormality : Trisomies 13, 18, 21 and myotonic dystrophy due to alteration of fetal m
and mobility.

(Q.102) Hart's rule is related to;


(a) Mechanism of Internal rotation.
(b) Mechanism of Crowning.
(c) Mechanism of Engagement.
(d) Mechanism of Restitution.
Your Response : a
Correct Answer : A
Exp: Mechanism of internal rotation
Elastic recoil of Levator Ani occurs bringing the occiput forwards in the midline. The proces
until the occiput is placed anteriorly. This is called as rotation by law of pelvic floor- Hart's r

(Q.103) At what stage of delivery antibiotic prophylaxis should be given in caesarian section?
(a) At the onset of labour pain
(b) At rupture of membrane
(c) When umbilical cord has been clamped
(d) After placenta delivery
Your Response : b
Correct Answer : C
Exp: When umbilical cord has been clamped
Ref: Dutta, 6th Edition (Revised 2009), Page no 640
Infective agents are mostly polymicrobial, including gram positive, gram negative aerobes a
Generally antibiotics with broad spectrum activity are better. Use of ceftriaxone (1 gm), Ce
gm) or amoxicillin clavulanic acid (1.2 gm), by intravenous route is a reasonable choice. Sh
of 1-3 doses are to be given. This can reduce the cost compared to a full seven day course.
the mother is given after the umbilical cord has been clamped. This avoids antibiotic expos
baby. Bacteriology pattern and antibiotic sensitivity need to be monitored regularly by the
laboratory.

(Q.104) Sarcoidosis in pregnancy:


(a) Worsens.
(b) Improves then worsens.
(c) No change.
(d) Worsens then improves.
Your
Response b
:
Correct
C
Answer :
Exp: No change
Pregnancy does not influence the natural course of
the disease.

(Q.105) Provisional short antiretroviral regime given in peripartum period reduces risk of vertical transmission by
(a) 35%
(b) 50%
(c) 65%
(d) 75%
Your Response : b
Correct Answer : B
Exp: 50%
Ref: Dutta, 6th Edition (Revised 2009), Page no 300
Most transmission of HIV occurs during pregnancy and birth, breast feeding may account fo
infants becoming infected after delivery.
Studies have demonstrated that truncated regimens of zidovudine alone or with Lamivudin
mother during last few weeks of pregnancy or even during labour & delivery & to the infan
less reduced transmission to infant by 50% compared to placebo.

(Q.106) Most Effective Contraceptive Method in presence of Cardiac Diseases is


(a) Barrier Method
(b) IUCD
(c) NFPM
(d) COCP
Your Response : a
Correct Answer : A
Exp: Barrier Method
Barrier method is safest. With persistent use the risk of failure associated with this techniq
drastically. NFPM are associated with a high failure rate. IUCD increases the risk of infective
at the time of insertion and later also in case PID occurs. OCP increase the risk of
venous thromboembolism .

(Q.107) Atosiban is used for:


(a) Pregnancy inducted hypertension
(b) Eclampsia
(c) Preterm labour
(d) Antiphospholipid antibody syndrome
Your Response : d
Correct Answer : C
Exp: Preterm labour
Ref: Dutta, 6th Edition (Revised 2009), Page no 508
Atosibian is an oxytocin analogue (antagonist) that blocks myometrial oxytocin receptors. I
intracellular calcium release, release of prostaglandins and thereby inhibits myometrial con

(Q.108) A 55 year old lady presenting to OPD with postmenopausal bleeding for 3 months has a 1 x1 an nodule on the anter
cervix, The most appropriate investigation to be done subsequently is
(a) Pap smear
(b) Punch biopsy
(c) Endocervical curettage.
(d) Colposcopy
Your Response : a
Correct Answer : B
Exp: Punch biopsy
Punch biopsy is required in all cases in which there is visible area of redness, an iodine nega
area of colposcopic abnormality.
A cone biopsy and curettage should be done when cytologic examination reveals moderate
dysplasia, carcinoma in situ, or perhaps invasive carcinoma.

(Q.109) The sternomastoid tumor appears usually:


(a) Just after birth
(b) 24 hours after birth
(c) 7 -10 days after birth
(d) 31-45 days after birth
Your Response : b
Correct Answer : C
Exp: 7-10 days after birth
Sternomastoid haematoma ( tumor) appears about 7-10 days after birth & is usually situate
junction of upper and middle third of the muscle. The swelling disappears by 6 month of ag

(Q.110) A case of Gestational trophoblastic neoplasia belongs to high risk group if disease develop
after
(a) Hydatidiform mole
(b) Full term pregnancy
(c) Spontaneous
(d) Ectopic pregnancy abortion
Your Response
a
:
Correct
B
Answer :
Exp: Full term pregnancy
Ref: Dutta, 6th Edition (Revised 2009), Page no 193-202
Classification of gestational trophoblastic disease
H. mole - 1. Complete 2. Partial
Gestational trophoblastic tumors
Non metastatic
Metastatic - Low risk - no risk factors
High risk - any risk factors
Pyretherapy hCG level> 40,000 miu/mL
Duration> 4 months
Brain or liver metastasis
Prior chemotherapy failure
Antecedent term pregnancy

(Q.111) (B) Lynch suture is applied on


(a) Cervix
(b) Uterus
(c) Fallopian tubes
(d) Ovaries
Your Response : b
Correct Answer : B
Exp: Uterus
Ref: Dutta, 6th Edition (Revised 2009), Page no 413-419
B - Lynch suture is a fundus compression suture applied to prevent PPH, It is placed at the t
laparotomy after hysterotomy and delivery of fetus, Advantage of this suture is that hyster
avoided.

(Q.112) Which of the following hepatitis virus have significant perinatal transmission
(a) Hepatitis-E virus
(b) Hepatitis-C virus
(c) Hepatitis-B virus
(d) Hepatitis-A virus
Your Response : a
Correct Answer : C
Exp: Hepatitis-B virus
Ref: Dutta, 6th Edition (Revised 2009), Page no 291
Perinatal transmission
Hepatitis -B
The risk of chronic hepatitis B infection in neonate who does not receive immunoprophylax
vaccination is 40%
Vertical transmission is 10% in mother with negative HBe Ag
When the women are infected during the 2nd and 3rd trimester the neonatal infection is 6
respectively and virtually all are infected in the perinatal period
Hepatitis - C
A marked variation is noted in vertical transmission rates of HCV is 0- 36%
In hepatitis C VIR positive, HIV negative mothers, without history of IV drugs abuse or trans
exposure, the risk of HCV vertical transmission is 0-18%
In patients who are HIV negative with ongoing IV drug abuse or blood transfusion during pr
- 23% HIV positive mothers vertical transmission rates - 6 - 36%

(Q.113) Risk of rupture in LSCS is


(a) 4%
(b) 10%
(c) 1.5%
(d) 15%
Your Response : d
Correct Answer : C
Exp: 1.5%
Risk of rupture in LSCS is 0.2 -1.5%. The risk of rupture in classical caesarean is 4-9%.The ris
when a trial of labour is given in Previous LSCS is 1 in 1000. The low rate of rupture of lowe
because of better healing that occurs as the lower segment is quiescent in puerperium and
scar to heal well. The two edges in the lower segment are of same caliber and when appro
well without hemorrhages that weaken the scar.

(Q.114) Haultain's operation is done for:


(a) Inversion of uterus
(b) Morbid adherent placenta
(c) Pelvic haematoma
(d) Rupture of the uterus
Your Response : b
Correct Answer : A
Exp: Inversion of uterus
Ref: Dutta, 6th Edition (Revised 2009), Page no 421
In sub acute stage of inversion of uterus if reposition could not be achieved manually or by
method, reposition may be done by abdominal operation (Haultain's operation).

(Q.115) Most common Cause of First Trimester Abortion is


(a) Chromosomal Abnormalities
(b) Maternal infections
(c) Hypothyroidism
(d) Progesterone deficiency
Your Response : d
Correct Answer : A
Exp: Chromosomal Abnormalities
Chromosomal abnormalities are responsible for 50% of first trimester abortions. Other cau
vascular disease like lupus, diabetes, hormonal problems, infections, uterine
abnormalities, trauma. Maternal age and previous h/o spontaneous abortion are leading r
abortion.

(Q.116) Most Common Side Effect of IUCD Insertion is


(a) Bleeding
(b) Cramping pain
(c) Syncope
(d) Uterine perforation
Your Response : a
Correct Answer : B
Exp: Cramping pain
Cramping: Many women feel cramping or pain during the IUD insertion process and immed
a result of cervix dilation during insertion. Taking NSAIDS before the procedure can reduce
can the use of a local anaesthetic. Misoprostol 6 to 12 hrs before insertion can help with ce
dilatation. Some women may have cramps for 1 to 2 weeks following insertion. The copper
increase cramps during a womans period. This symptom will clear up for some women in 3
but may not for others.
Perforation: Very rarely, the IUD can move through the wall of the uterus. The risk of perfo
1,000 insertions or less.
Bleeding : it is another side effect that is very common with insertion though not as commo
pain.

(Q.117) Most common variety of compound presentation is:


(a) Head with hand
(b) Head with foot
(c) Head with both foots
(d) Head, hand & a foot
Your Response : a
Correct Answer : A
Exp: Head with hand
Ref: Dutta, 6th Edition (Revised 2009), Page no 386
When a cephalic presentation is complicated by the presence of a hand or a foot or both al
head or presence of one or both hands by the side of the breech, it is called compound pre
commonest one being the head with hand and the rarest one being the presence of head, h
foot.
(Q.118) Which of the following type of anesthesia is frequently associated with fetal bradycardia:
(a) Paracervical block
(b) Pudendal block
(c) Spinal block
(d) Epidural block
Your Response : c
Correct Answer : A
Exp: Paracervical block
Pudendal block is perhaps the most common form of anesthesia used for vaginal delivery.
adequate pain relief for episiotomy, spontaneous delivery forceps delivery, or vacuum extr
success of a pudendal block depends on a clear understanding of the anatomy of the puden
its surroundings. Complications (vaginal hematomas, retropsoas, or pelvic abscesses) are q
Paracervical block was a popular form of anesthesia for the first stage of labor until it was i
several fetal deaths. It has been shown that paracervical block was associated with fetal bra
to 35% of cases, probably the response to rapid uptake of the drug from the highly vascular
space with a resultant reduction of uteroplacental blood flow. Death in some cases was rel
injection of the local anesthetic into the fetus.

(Q.119) The suggested explanation for fetal passage of meconium is


(a) In response to hypoxia
(b) Normal gastrointestinal tract maturation under neural control
Vagal stimulation from common but transient umbilical cord entrapment and resultant in
(c) peristalsis
(d) All of the above
Your Response : a
Correct Answer : D
Exp: All of the above
Three theories have been suggested to explain fetal passage of meconium and may, in part
tenuous connection between the detection of meconium and infant mortality. The patholo
explanation proposes that fetuses pass meconium in response to hypoxia, and that meconi
signals fetal compromise. Alternatively, in utero passage of meconium may represent norm
gastrointestinal tract maturation under neural control. Third, meconium passage could also
stimulation from common but transient umbilical cord entrapment and resultant increased
Thus, fetal release of meconium could also represent physiological processes.

(Q.120) A patient is receiving external beam radiation for treatment of metastatic endometrial cancer. The treatment field i
entire pelvis. Which of the following tissues within this radiation field is the most radiosensitive?
(a) Vagina
(b) Ovary
(c) Rectovaginal septum
(d) Bladder
Your Response : b
Correct Answer : B
Exp: Ovary
Different tissues tolerate different doses of radiation, but the ovaries are by far the most ra
They tolerate up to 2,500 rads, while the other tissues listed tolerate between 5,000 and 20
Acute evidence of excessive radiation exposure includes tissue necrosis and inflammation,
enteritis, cystitis, vulvitis, proctosigmoiditis, and possible bone marrow suppression.
Chronic effects of excessive radiation exposure are manifest months to years after therapy
vasculitis, fibrosis, and deficient cellular regrowth; these can result in proctitis, cystitis, fistu
and stenosis.
Successful radiation depends on (1) the greater sensitivity of the cancer cell compared with
and (2) the greater ability of normal tissue to repair itself after irradiation.
The maximal resistance to ionizing radiation depends on an intact circulation and adequate
oxygenation. Resistance also depends on total dose, number of portions, and time intervals
The relative resistance of normal tissue (cervix and vagina) in cervical cancer allows high su
approaching 15,000 to 20,000 rads to be delivered to the tumor with intracavitary devices,
of the inverse square law, significantly lower doses of radiation reach the bladder and rectu
The greater the fractionalization (number of portions the total dose is broken into), the bet
tissue tolerance of that radiation dose; hence 5,000 rads of pelvic radiation is usually given
fractions over 5 weeks, with approximately 200 rads being administered each day

(Q.121) Most Common cause of Macrosomic Baby is


(a) Gestational Diabetes
(b) Maternal obesity
(c) Prolonged pregnancy
(d) PIH
Your Response : a
Correct Answer : A
Exp: Gestational Diabetes
Though not given in the options here overt diabetes is the commonest cause of macrosomi
pregnancy. Following it is gestational diabetes, obesity, increased maternal weight gain, pro
pregnancy. PIH if at all will cause IUGR not macrosomia.

(Q.122) Which of the following statements concerning placenta previa is true?


(a) Its incidence decreases with maternal age
(b) Its incidence is unaffected by parity
(c) The initial hemorrhage is usually painless and rarely fatal
(d) Management no longer includes a double setup
Your Response : c
Correct Answer : C
Exp: The initial hemorrhage in placenta previa is usually painless and rarely fatal. If the fetus is
and if hemorrhaging is not severe, vaginal examination of a woman suspected of having pla
frequently can be delayed until 37 weeks gestation; this delay in the potentially hazardous
reduces the risk of prematurity which is often associated with placenta previa. Vaginal exam
needed to determine whether a low- lying placenta is covering the internal os of the cervix
performed in an operating room fully prepared for an emergency cesarean section (i.e., a d
Increasing maternal age and multiparity are associated with a higher incidence of placenta

(Q.123) The most important reason pelvic inflammatory disease (PID) must be recognized and treated promptly is to preven
(a) Pelvic pain syndrome
(b) Infertility
(c) Ectopic pregnancy
(d) Sepsis
Your Response : b
Correct Answer : D
Exp: Sepsis
Untreated PID can lead to formation of tubo-ovarian abscess (TOA). Mortali
rare, does occur, particularly in neglected cases in 'which a ruptured TOA ca
septic shock and death. In the United States, more than 150 deaths annually
attributed to PI(D).
(Q.124) Spiegelberg criteria is used for the diagnosis of
(a) Tubal pregnancy
(b) Ovarian pregnancy
(c) Cervical pregnancy
(d) Abdominal pregnancy
Your Response : b
Correct Answer : B
Exp: Ovarian pregnancy
The most common site of ectopic pregnancy is fallopian tube. Ectopic at other sites is seco
ruptured ectopic from fallopian tube unless proved otherwise. The criteria defining primary
these uncommon sites are as follows
Rubins----primary cervical, Speigelberg-----primary ovarian, Studdiford---------primary ab
pregnancy

(Q.125) At 20th week of pregnancy foetus weight is


(a) 250gm
(b) 350gm
(c) 300gm
(d) 400gm
Your Response : a
Correct Answer : D
Exp: 400gm
There is no formula for this. The answer is there in standard books. It weighs 300 gms at 20
Remember the weight is 500 gms at 22weeks which by now is considered as the period of v
WHO.

(Q.126) A woman is found to have a unilateral invasive carcinoma of vulva that is 2 cm in diameter but not associated with e
lymph node spread. Initial management should consists of :
(a) Chemotherapy
(b) Radiation therapy
(c) Radical vulvectomy
(d) Radical vulvectomy and bilateral inguinal lymphadenectomy
Your Response : d
Correct Answer : D
Exp: Radical vulvectomy and bilateral inguinal lymphadenectomy
Women who have invasive vulvar carcinoma usually are treated surgically. If the lesion is u
associated with fixed or ulcerated inguinal lymph nodes, and does not involve the urethra,
or rectum, then treatment usually consists of radical vulvectomy and bilateral inguinal lymp
If inguinal lymph nodes show evidence of metastatic disease, bilateral pelvic lymphadenect
performed. Radiation therapy, though not a routine part of the management of women wh
vulvar carcinoma, is employed (as an alternative to pelvic exenteration with radical vulvect
treatment of women who have local, advanced carcinoma.

(Q.127) The ovarian tumor which is most likely to be associated with virilization
(a) Granulosa tumor
(b) Sertoli-Leydig cell tumor
(c) Immature teratoma
(d) Gonadoblastoma
Your Response : d
Correct Answer : B
Exp: Sertoli-Leydig cell tumor
Sertoli-Leydig cell tumors, which represent less than 1% of ovarian tumors, may produce s
virilization. Histologically, they resemble fetal testes; clinically, they must be distinguished f
functioning ovarian neoplasms as well as from tumors of the adrenal glands, since both adr
and Sertoli-Leydig tumors produce androgens. The androgen production can result in sebor
menstrual irregularity, hirsutism, breast atrophy, alopecia, deepening of the voice, and clito
Recurrences of Sertoli-Leydig cell tumors, which seem to have a low malignant potential, u
within 3 years of the original diagnosis. Granulosa and theca cell tumors are often associate
excessive estrogen production, which may cause pseudoprecocious puberty, post- menopa
or menorrhagia. These tumors are associated with endometrial carcinoma in 15% of patien
these tumors are quite friable, affected women frequently present with symptoms caused
rupture and intraperitoneal bleeding. Granulosa tumors are low-grade malignancies that te
more than 5 years after the initial diagnosis. Because their malignant potential is impossible
histologically, long- term follow-up is mandatory.

(Q.128) A 30-yrs-old diabetic female is concerned about pregnancy. She can be assured that which of the following risks is th
as for the general population.
(a) Preeclampsia and eclampsia
(b) Infection
(c) Fetal cystic fibrosis
(d) Postpartum hemorrhage after vaginal delivery
Your Response : d
Correct Answer : C
Exp: Cystic fibrosis
Maternal diabetes mellitus can affect a pregnant woman and her fetus in many ways. The d
of preeclampsia or eclampsia is about 4 times as likely as among nondiabetic women. Inf
more likely not only to occur but to be severe. The incidences of fetal macrosomia or death
dystocia are increased, and hydramnios is common. The likelihood of postpartum hemorrh
vaginal delivery and the frequency of cesarean section are both increased in diabetic wome
incidence of fetal genetic disorders such as cystic fibrosis is unaffected by diabetes.

(Q.129) Which of the following statements concerning appendicitis in pregnancy is true?


(a) Diagnosis is similar to that in the non pregnant patient
(b) The maternal death rate is highest in the first trimester
(c) Surgical treatment should be delayed until the diagnosis is firmly established
(d) The incidence is unchanged by pregnancy
Your Response : c
Correct Answer : D
Exp: The incidence is unchanged by pregnancy
The incidence of appendicitis in pregnancy is 1 in 2000, the same as that in the nonpregna
population.The diagnosis is very difficult in pregnancy because leukocytosis, nausea, and v
common in pregnancy and the upward displacement of the appendix by the uterus may cau
appendicitis to simulate cholecystitis, pyelonephritis, gastritis, or degenerating myomas. Su
necessary even if the diagnosis is not certain. Delays in surgery due to difficulty in diagnosis
appendix moves up are probably the cause of increasing maternal mortality with increasing
age. Pre mature birth and abortion account for a rate of fetal loss close to 15%.

(Q.130) Strawberry cervix is caused by


(a) Trichomonas vaginalis
(b) Neisseria
(c) Candidiasis
(d) Chlamydia
Your Response : a
Correct Answer : A
Exp: Trichomonas vaginalis
Strawberry cervix is caused by Trichomonas vaginalis. Severe inflammation of the cervix w
haemorrhages gives it a strawberry appearance and hence the name. Similar inflammation
haemorrhages on the vagina is called colpitis macularis.

(Q.131) Hypertonic dysfunctional labor generally can be expected to


(a) Be associated with rapid cervical dilation
(b) Cause little pain
(c) Occur in active phase of labor
(d) Response to sedation
Your Response : b
Correct Answer : D
Exp: Response to sedation
Hypertonic uterine dysfunction is characterized by a lack of coordination of uterine con tra
caused by disorganization of the contraction gradient, which normally is greatest at the fun
the ceryix. This type of dysfunction usually appears during the latent phase of labor and i
sedation, not oxytocin stimulation. The disorder is accompanied by a great deal of discom
cervical dilation (the familiar and painful false labor). After being sedated for a few hours, a
women usually awaken in active labor.

(Q.132) A 21-yrs-old has difficulty voiding 6 hrs postpartum. The least likely cause is?
(a) Preeclampsia
(b) Infusion of oxytocin after delivery
(c) Vulvar hematoma
(d) Urethral trauma
Your Response : b
Correct Answer : A
Exp: Preeclampsia
An inability to void often leads to the diagnosis of a vulvar hematoma. Such hematomas are
enough to apply pressure on the urethra. Pain from urethral laceration is another reason w
difficulty voiding after delivery. Both general anesthesia, which temporarily disturbs neural
bladder, and oxytocin, which has an antidiuretic effect, can lead to an over distended bladd
inability to void. In this case an indwelling catheter should be inserted and left in for at leas
recovery of normal bladder tone and sensation. Preeclampsia often leads to edema, which
to diuresis postpartum.

(Q.133) HCG is produced from


(a) Foetus
(b) Amnion
(c) Cytotrophoblast
(d) Syncytiotrophoblast
Your Response : d
Correct Answer : D
Exp: Syncytiotrophoblast
HCG or corpus luteum rescue factor is produced from syncytiotrophoblast. Its production
trophoblast differentiates after implantation into syncytiotrophoblast and cytotrophoblast.
1.2 days and doubling time is 2 days. . It has 2 subunits & . The subunit is similar to tha
and TSH. The specific actions are due to subunit. Its role is in early pregnancy maintenan
supports corpus luteum so that it produces adequate progesterone and maintains pregnan
time that placenta takes over. If implantation does not occur and HCG is not produced corp
degenerates by 8th day.

(Q.134) Dizygotic twins are always


(a) Monochorionic monoamniotic
(b) Monochorionic diamniotic
(c) Dichorionic diamniotic
(d) Conjoint twins
Your Response : c
Correct Answer : C
Exp: Dichorionic diamniotic
Dizygotic twins are always dichorionic and diamniotic . Whereas the reverse is not true . T
twins can be seen in monozygotic twins if the zygote divides within 72
hrs. Monochorionic monoamniotictwins are always monozygotic.

(Q.135) A 27years old G2P2 comes to your clinic to take advice about contraceptive measure. She has delivered a male child
and presently lactating. She has no contraindication of any hormone use. She can be given-
(a) DMPA
(b) Minipill
(c) Levonorgestrol implants
(d) All of the above
Your Response : b
Correct Answer : B
Exp: Minipill
'Progesterone only contraceptives' including minipills, DMPA and LNG implants don't affe
of milk, therefore, they are the contraceptive of choice for breast feeding women.
ACDG recommendations for hormonal contraception fused by breast feeding women proge
contraceptives prescribed or dispended at discharge from the hospital to be starte2-3 week
(e.g. the first Sunday after the newborn is 2 weeks old).
Depot medroxyprogesterone acetate initiated at 6 weeks postpartum
Hormonal implants inserted at 6 weeks postpartum.
Combined estrogen-contraceptive, if prescribed, should not be started before weeks postp
when lactation is well established and the infant's nutritional status is well-monitored.

(Q.136) Most common symptom of ectopic pregnancy


(a) Bleeding
(b) Pain
(c) Abortion
(d) Infection
Your Response : b
Correct Answer : B
Exp: Pain
Ectopic pregnancy is characterised by the triad of amenorrhoea, pain and BPV of which the
common symptom is pain. The diagnosis is based on clinical features and ultrasound, thou
is the gold standard. When diagnosis is in doubt serial estimation of HCG is done over 48 hr
is s/o ectropic pregnancy.

(Q.137) Boundaries of ovarian fossa are


(a) Posterior obliterated umbilical artery
(b) Ureter posteriorly
(c) Uterine tube anteriorly
(d) Internal iliac artery laterally
Your Response
d
:
Correct Answer
B
:
Exp: Ureter posteriorly
The ovary lies in a shallow depression, named the ovarian fossa, on the lateral
wall of the pelvis.
This fossa has the following boundaries:
Above: by the external iliac vessels
In front: by the obliterated umbilical artery
Behind: by the ureter

(Q.138) Gartners duct is present in


(a) Broad ligament
(b) Ovarian ligament
(c) Perineal membrane
Separates medial part of epididymis
(d) from testis
Your
Respons a
e:
Correct
A
Answer :
Exp: Broad ligament

(Q.139) Bartholins gland is located at


(a) Vestibule
(b) Labia majora
(c) Superficial perineal pouch
(d) Deep perineal pouch
Your Response : a
Correct Answer : C
Exp: Superficial perineal pouch
Bartholins glands or greater vestibular glands are paired structures that are located in the
perineal pouch. The glands are drains by a duct that opens into the vestibule at the juncti
2/3rds and posterior 1/3rd in the groove between labia minora and hymen. The lining epit
columnar which becomes stratified squamous at the tip due to recurrent infections. The tre
Bartholins abscess is incision and drainage while treatment for recurrent Bartholins cyst i
marsupialization.

(Q.140) A 23-year old patient diagnosed with stage IA malignant ovarian germ cell tumor. Which of the following would be t
treatment-
(a) Unilateral oophorectomy
(b) Bilateral oophorectomy and hysterectomy
(c) Unilateral oophorectomy followed by chemotherapy
(d) Ovarian biopsy followed by chemotherapy
Your Response : b
Correct Answer : C
Exp: Unilateral oophorectomy followed by chemotherapy
Approximately 50% to 70% of germ cell malignancies are stage 1. Except for dysgenuinoma
incidence of bilaterality is 10-15%, these tumors are very rarely b/L. Therefore, U/L salpinn
oophorectomy, preserving the contra lateral and uterus, combined with surgical staging c
performed in most patients (particularly in young patient where fertility-sparing surgery is
few exceptions, all germ cell malignancies require postoperative chemotherapy.

(Q.141) Space of Retzius is found


(a) Behind cervix
(b) In front of rectum
(c) In front of bladder
(d) Behind rectum
Your Response : c
Correct Answer : C
Exp: In front of bladder
It is retropubic or prevesical space that is extraperitoneal and lies between urinary bladder
and pubic symphysis.

(Q.142) 25 years old 2nd gravida with B negative blood group reaches in labour room in full dilatation. All the following norm
followed except:
(a) Cord blood to be saved in 2 vial plain & EDTA vial
(b) Early clamping of cord
(c) Avoidance of manual removal of placenta
(d) Liberal prophylactic use of ergometrine
Your Response : c
Correct Answer : D
Exp: Liberal prophylactic use of ergometrine
Use of ergot derivatives is contraindicated in Rh negative mother to avoid chances of feto
micro transfusions.

(Q.143) True regarding foetal trunk movements in third trimester


(a) Increases towards term
(b) Decreases towards term
(c) Remain unchanged
(d) None of the above
Your Response : a
Correct Answer : A
Exp: Increases towards term
By term, the normal number of generalized movements is reduced as a result of cerebral m
processes, rather than as a consequence of the decrease in the amniotic fluid
volume.Simultaneously with the decrease in the number of generalized movements, an i
facial movements, including opening/closing of the jaw, swallowing, and chewing and of
movements is observed.

(Q.144) Which of the following methods of treating eclampsia has the LEAST effect on neonates
(a) Lytic cocktail regime
(b) MgSO4 regime
(c) Diazepam therapy
(d) Phenobarbitone therapy
Your Response : b
Correct Answer : B
Exp: MgSO4 regime
Magnesium sulfate to control convulsions
In more severe cases of preeclampsia as well as eclampsia, magnesium sulfate administere
is the effective anti convulsant agent without producing CNS depression in either the moth
infants
Because labour and delivery is a more likely time for convulsions to develop, women with p
Eclampsia usually are given magnesium sulfate during labour and for 24 hours postpartum
Magnesium sulfate is not given to treat hypertension
Monitoring with (I) Knee jerk (II) Urine out put (III) Respiratory rate

(Q.145) All of the followings are true regarding the pudendal nerve, except
(a) Sensory and motor
(b) Derived from S2,3,4
(c) Comes out through the lesser sciatic foramen
(d) Main nerve supply of pelvic organs
Your Response : b
Correct Answer : C
Exp: Comes out through the lesser sciatic foramen
The pudendal nerve is a mixed nerve in the pelvic region that innervates the skin over exte
of both sexes, as well as sphincters for the bladder and the rectum
The pudendal nerve originates in the sacral plexus; it derives its fibers from the ventral bra
second, third, and fourth sacral nerves (S2, S3, and S4)
It passes between the piriformis and coccygeus muscles and leaves the pelvis through the l
thegreater sciatic foramen
Remember: PIN structures leaves greater sciatic foramen and re-enters lesser sciatic fora
It crosses the spine of the ischium (lies posterior to it), and re-enters the pelvis through the
foramen.
It accompanies the internal pudendal vessels upward and forward along the lateral wall of
fossa, being contained in a sheath of the obturator fascia termed the pudendal canal
The pudendal nerve gives off the inferior rectal nerves. It soon divides into two terminal br
theperineal nerve, and the dorsal nerve of the penis (males) or the dorsal nerve of the clito
females).
The pudendal nerve innervates the penis and clitoris, bulbospongiosus and ischiocavernosu
areas around the scrotum, perineum, and anus.
At sexual climax, the spasms in the bulbospongiosus and ischiocavernous results in ejaculat
male and most of the feelings of orgasm in both sexes.

(Q.146) Abruptio placentae can be classified on the basis of


(a) Anatomic locations of placenta
(b) Pathological features including blood coagulation profile
(c) Clinical findings
(d) Investigations using ultrasound
Your Response : d
Correct Answer : C
Exp: Clinical findings
Abruptio placentae is classified in 4 grades - grade 0 to grade 3 according to case classificat
upon the degree of placental abruption and its clinical effects including bleeding per vaginu
tenderness, condition of fetus, features of shock, associated coagulation defect and kidney
cases are classified into four grades.

(Q.147) Oral anticoagulants given to pregnant women cause


(a) Long bones limb defect
(b) Craniofacial malformation
(c) CVS malformation
(d) Costochondro dysplasia
Your Response : d
Correct Answer : B
Exp: Oral anticoagulants (Warfarin) causes
Contradi's syndrome (Skeletal and facial anomalies optic atrophy, microcephaly, chondrody
punctata
Warfarin given in early pregnancy increases birth defects, specially skeletal abnormality foe
syndrome - hypoplasia of nose, eye socket hand bones, growth retardation
Given later in pregnancy - it can cause CNS defects, foetal haemorrhage, foetal death and a
neonatal hypoprothrombinemia.

(Q.148) Condyloma accuminata in pregnancy is treated by


(a) Podophyllin
(b) Podophyllin toxin
(c) Trichloroacetic acid
(d) 5-FU cream
Your Response
a
:
Correct Answer
C
:
Exp: Treatment of condyloma -acuminata - (verruca-genital is)
Podophyllum is effective only on the genital warts
Podophyllum is potentially toxic and must be avoided in during pregnancy
Chemical cauterization is usually done with liquified phenol or concentrated
trichloroacetic acid

(Q.149) Using histological criteria to date endometrial development, which of the following is the earliest sign of ovulation?
(a) Gland coiling
(b) Stromal edema
(c) Neovascularization
(d) Glycogen accumulation
Your Response : d
Correct Answer : D
Exp: Glycogen accumulation
After ovulation the estrogen-primed endometrium responds to rising levels of progesteron
predictable manner. By day 17, glycogen accumulates in the basal portion of the glandula
creating subnuclear vacuoles and pseudo stratification. This is the first sign of ovulation th
in histological changes, and it is likely the result of direct progesterone action through prog
receptors expressed in the glandular cells.

(Q.150) Gender of the fetus is first evident by what gestational age (weeks) ?
(a) 6
(b) 8
(c) 12
(d) 16
Your Response : b
Correct Answer : C
Exp: 12
By the end of the 12th week of pregnancy, when the uterus usually is just palpable above t
pubis, the crown-rump length of the fetus is 6 to 7 cm. Centers of ossification have appeare
the fetal bones, and the fingers and toes have become differentiated. Skin and nails have d
scattered rudiments of hair appear. The external genitalia are beginning to show definitive
or female gender. The fetus begins to make spontaneous movements.

(Q.151) All are true statements about puerperal venous thrombosis except.
(a) It is most common (10 times) with caesarean section
(b) Heparin is contraindicated
(c) Dehydration correction during delivery is a preventive measure
(d) Advancing parity is a high risk factor
Your Response : c
Correct Answer : B
Exp: Heparin is contraindicated
The 3 important predisposing factors for puerperal venous thrombosis include - trauma, se
anemia.
Dehydration during delivery should be promptly corrected. Other risk factors are - advancin
parity, caesarean section (10 times more than vaginal birth), use of oestrogen in suppressio
obesity heart disease, antiphospholipid antibody syndrome.
Management include bed rest with foot end elevated, analgesics and anticoagulants - Hepa
N followed by 10000 U 5-6 hourly for 4-6 injections, to be continued for at least 7-10 days.
commonly used orally and used for maintenance of therapy.

(Q.152) In presence of secondary sexual characters cause screening for primary amenorrhea is done at
(a) 12years
(b) 14 years
(c) 16 years
(d) 18 years
Your Response : b
Correct Answer : C
Exp: 16 years
Primary amenorrhoea is defined as the absence of secondary sexual characters by 14 yrs
absence of menarche in the presence of secondary sexual characters till 16 yrs of age, In
of pubertal development the changes are as follows thelarche, increased growth spurt,
pubarche, menarche. As menarche is the last stage ,in case secondary sexual characters ha
to account for constitutional delay investigation is begun at 16 years.

(Q.153) Largest tumour of the ovary is


(a) Mucinous cystadenoma
(b) Dermoid
(c) Serous cystadenoma
(d) All of the above
Your Response : b
Correct Answer : A
Exp: Mucinous cystadenoma
Ovarian mucinous cystadenoma is a benign tumour that arises from the surface epithelium
It is a multilocular cyst with smooth outer and inner surfaces. It tends to be huge in size. Th
become very large and can extend up into the abdomen. Of all ovarian tumours, mucinous
comprise 15%.As it contains mucinous fluid, its rupture leads to mucinous deposits on the p
(pseudo-myxoma peritonei).
(Q.154) A patient with a positive pregnancy test and a serum progesterone level of less than 5 ng/mL strongly suggests:
(a) Corpus luteum decline (physiological)
(b) Inconclusive level
(c) A nonviable pregnancy
(d) A viable intrauterine pregnancy
Your Response : a
Correct Answer : C
Exp: A nonviable pregnancy
A single progesterone measurement can be used to establish that there is a normally devel
pregnancy with high reliability. A value exceeding 25 ng/mL excludes ectopic pregnancy wi
percent sensitivity. Values below 5 ng/mL occur in only 0.3 percent of normal pregnancies.
this low suggest either an intrauterine pregnancy with a dead fetus or an ectopic pregnancy
cases, however, progesterone levels range from 5 to 25 ng/mL and are inconclusive.

(Q.155) Mothers with phenylketonuria (PKU) are at increased risk of having children with :
(a) Hydrocephaly
(b) Spina bifida
(c) Skeletal dysplasia
(d) Mental retardation
Your Response : d
Correct Answer : D
Exp: Mental retardation
Women with PKU who plan to conceive and are not already on a phenylalanine-restricted d
counseled to adhere to the diet before conception and throughout pregnancy. Phenylalanin
crosses the placenta, and high maternal serum levels can result in damage to the fetus or p
Hyperphenylalaninemia causes microcephaly with mental retardation and cardiac defects i
heterozygote fetus who otherwise would not be affected. Importantly, these defects can b
with maternal dietary treatment.

(Q.156) What is the diagnosis of this condition?

(a) Red Degeneration in fibroid in pregnancy


(b) Uterus in placenta pravia
(c) Couvelaire uterus
(d) None of the above
Your Response : a
Correct Answer : C
Exp: Couvelaire uterus
Couvelaire uterus (also known as uteroplacental apoplexy) is a life-threatening condition in
loosening of the placenta (abruptio placentae) causes bleeding that penetrates into the
uterine myometriumforcing its way into theperitoneal cavity.

(Q.157) Fetal blood loss occurs in


(a) Vasa previa
(b) Placenta previa
(c) Cord prolapse
(d) Abruption
Your Response : d
Correct Answer : A
Exp: Vasa previa
Vasa previa is an aberration that is seen with velamentous insertion of cord. If the cord im
membrane and vessels start dividing before entering the placenta they are unsupported by
the vessels lie ahead of the presenting part over internal os (vasa previa) ,during labour wh
dilates ,due to shear stress membrane with vessels rupture which results in quick fetal exsa
fetal vessels have no valves . The fetal mortality is 75-100%

(Q.158) Meconium-stained amnionic fluid is closely associated with following neonatal outcome?
(a) Death
(b) Cerebral palsy
(c) Necrotizing enterocolitis
(d) None of the above
Your Response : c
Correct Answer : D
Exp: None of the above
Obstetrical teaching throughout the past century has included the concept that meconium
potential warning of fetal asphyxia.
In an investigation from Parkland Hospital, meconium was found to be a "low-risk" obstetri
because the perinatal mortality attributable to meconium was 1 death per 1000 live births.

(Q.159) Indication to end expectant management of placenta previa.


(a) IUD
(b) 34 weeks
(c) Transverse lie
(d) Hemodynamically stable with no blood loss
Your Response : c
Correct Answer : A
Exp: IUD
Expectant management of palcenta previa is called Mac Afee Johnsons regime. The indic
preterm and hemodynamically stable
- Fetal well being assured i.e IUD, GCA, severe IUGR should be ruled out.
If a patient is on expectant management, the indications for termination are
Patient reaches term
Has a bout of bleeding that leads to alteration in the vitals
IUD occurs
Fetal distress
Patient goes in labour.
In the present question 34 weeks, transverse lie are also indications for conservative mana
bleeding occurs.
(Q.160) Which is not an indication for a classical cesarean incision?
(a) Cannot visualize the lower uterine segment
(b) Transverse lie
(c) Premature breech
(d) Term breech (frank)
Your Response : c
Correct Answer : D
Exp: Term breech (frank)
Occasionally it is necessary to use a classical incision for delivery. Some indications are:
Difficulty in exposing or safely entering the lower uterine segment because the bladder is
densely adherentfrom previous surgery, a myoma occupies the lower uterine segment, or t
been invaded by cancer.
Transverse lie of a large fetus, especially if the membranes are ruptured and the shoulder is
the birth canal. A fetus presenting as a back-down transverse lie may be particularly difficu
through a transverse incision.
Some cases of placenta previa with anterior implantation, especially in the case where the
grown through a prior uterine incision (placenta percreta).
Certain cases in which the fetus is very small, especially if breech, and the lower uterine seg
thinned out.
Massive maternal obesity precluding safe to the lower uterine segment.

(Q.161) Which is a true statement regarding the psychological symptoms of the climacteric?
(a) They are considerably less important than hormone levels
(b) They commonly include insomnia, irritability, frustration, and malaise
(c) They are related to a drop in gonadotropin levels
(d) They are not affected by environmental factors
Your Response : b
Correct Answer : B
Exp: They commonly include insomnia, irritability, frustration, and malaise.
Psychological symptoms during the climacteric occur at a time when much is changing in a
Steroid hormone levels are dropping, and the menses is stop ping. However, studies show t
factors to be unrelated to emotional symptoms in most women. Many factors, such as horm
environmental, and intrapsychic elements, combine to cause the symptoms of the climacte
insomnia; vasomotor instability (hot flushes, hot flashes); emotional lability; and genital tra
with vulvar, vaginal, and urinary symptoms.

(Q.162) Which contraindication is specific to the progestin intrauterine device?


(a) Dysmenorrhea
(b) History of ectopic pregnancy
(c) Prior thromboembolism
(d) Wilson disease
Your Response : b
Correct Answer : B
Exp: History of ectopic pregnancy
Because the progestin released by the Mirena device may inhibit tubal mobility, a previous
pregnancy or predisposing risk factors are considered contraindications.

(Q.163) Which of the following fetal factors are NOT associated with postterm pregnancy?
(a) Renal agenesis
(b) X-linked placental sulfatase deficiency
(c) Adrenal hypoplasia
(d) Anencephaly
Your Response : a
Correct Answer : A
Exp: Renal agenesis
Fetal-placental factors that have been reported as predisposing to postterm pregnancy incl
anencephaly, adrenal hypoplasia, and X-linked placental sulfatase deficiency.
These cause a lack of the usually high estrogen levels of normal pregnancy.

(Q.164) Which ultrasound measurements is the most reliable index of fetal size?
(a) Biparietal diameter
(b) Abdominal circumference
(c) Femur length
(d) Intrathoracic ratio
Your Response : c
Correct Answer : B
Exp: Abdominal circumference
The optimal ultrasonic method of estimating fetal size, and therefore growth restriction, w
Manning. Combining head, abdomen, and femur dimensions should in theory enhance the
predictions of fetal size. Unfortunately, any potential improvement is apparently lost by the
error inherent in measurement of each individual fetal dimension. As a result, abdominal ci
measurements have been accepted by most experts as the most reliable index of fetal size.

(Q.165) Amniotic fluid at term


(a) 800ml
(b) 600ml
(c) 200ml
(d) 1000ml
Your Response : d
Correct Answer : B
Exp: 600ml
Amniotic fluid at 8 weeks------50 ml
12weeks-----200ml
36 weeks-----1 ltr
38 weeks---- 800ml
40 weeks-----600 ml
42 weeks-----200 ml
Risk of postmaturity is due to oligoamnios. Turnover is every 3hrs. Amniotic fluid is rich in e
hence can demonstrate ferning like cervical mucus

(Q.166) Which one of the following tumors involving the female genital tract has the WORST prognosis?
(a) Dysgerminoma in a 35-year-old
(b) Uterine choriocarcinoma in a 25-year-old with a recent history of molar pregnancy
(c) Granulosa cell tumor in a 40-year-old
(d) Serous cystadenocarcinoma of the ovary in a 45-year-old
Your Response : b
Correct Answer : D
Exp: Serous cystadenocarcinoma of the ovary in' a 45-year-old
Around 75% of serous cystadenocarcinoma of the ovary are in an advanced stage at
the time of diagnosis.
These tumors have a worse prognosis than any of the other tumors listed.
(Q.167) Treatment option in 34 weeks with uterine contraction
(a) Tocolytics for 3 more weeks
(b) Dexamethasone should be given
(c) Vacuum assisted delivery
(d) Caesarean section
Your Response : c
Correct Answer : A
Exp: Tocolytics for 3 more weeks
At 34 weeks there is no indication of caesarean section or vacuum assisted delivery. There
of dexamethasone at or more 34weeks. As no other details are mentioned the best option
3 weeks. If the question had specified 34 weeks with placenta previa with contractions then
option would be caesarean section.

(Q.168) The WORST prognosis in patients with endometrial carcinoma is found in patients with tumor involvement of the:
(a) Uterine cervix
(b) Upper vagina
(c) Bladder mucosa
(d) Parametrium
Your Response : c
Correct Answer : C
Exp: Bladder mucosa
Involvement of the bladder mucosa constitutes a stage IV tumor. The other examples give
stage II or III. The more advanced the stage of a tumor, the worse the prognosis.

(Q.169) All are renal changes during normal pregnancy except


(a) Dilatation of pelvis, calyces and ureters which is more marked on right side
(b) Renal bicarbonate threshold increases
(c) GFR increases by 50%
(d) Threshold for AVP release and thirst decrease
Your Response : d
Correct Answer : B
Exp: Renal bicarbonate threshold increases
Various renal changes during normal pregnancy are increase in renal size (renal length appr
greater on X-ray), dilatation of pelvis calyces and ureters which resembles hydronephrosis o
(more marked on right side), GFR and renal plasma flow increases by 50%, renal bicarbonat
decreases, progesterone stimulate respiratory center and there is alteration of osmoreglati
threshold for AVP release and thirst decreases.

(Q.170) Pregnant female with history of previous LSCS comes in labor with hematuria. The diagnosis is?
(a) Impending scar rupture
(b) Prolonged labor
(c) Urethral trauma
(d) Cystitis
Your Response
b
:
Correct Answer
A
:
Exp: Impending scar rupture
Rupture Uterus
Definition: Dissolution in the continuity of uterine wall anytime beyond 28
weeksQ of pregnancy.
Features of Uterine rupture (during labour)
a) Failure in progress of labour. b) Suprapubic pain.
c) Slight bleeding PV and Hematuria. d) Alteration in fetal heart rate.
e) Unexplained maternal tachycardia. f) Hypotension.
g) Tenderness over scar. h) Bladder tenesmus.
i) Ballooning of lower segment.

(Q.171) The most commonly used in emergency contraception?


(a) Levonorgestrel
(b) Micronized progesterone
(c) Medroxyprogesterone
(d) Norethisterone
Your Response : a
Correct Answer : A
Exp: Levonorgestrel
EMERGENCY CONTRACEPTION
Emergency Contraceptive Pills (ECP)
Emergency contraception was once called the best kept secret of family planning.
In the early 1970s, Yuzpe and colleagues tested combinations of ethinyl estradiol and norg
postcoital contraception, based on a single dose of 50 microgm ethinyl estradiol and 500 m
norgestrel . Subsequent clinical trials led them to conclude that the most successful regime
two doses of 100 microgm ethinyl estradiol and 500 microgm levonorgestrel, with the first
within 72 hours of unprotected intercourse, and the second dose taken 12 hours later.
When started within 72 hours of unprotected intercourse, combined ECPs prevent about 7
pregnancies.
Various doses of levonorgestrel have also been tested since the 1970s for emergency contr
large multinational, randomized, double-blind clinical trial was conducted comparing the Yu
and levonorgestrel treatment (one 750 microgm dose followed by a second 750 microgm d
later) when started up to 72 hours after unprotected intercourse. Results showed that levo
not only better tolerated but was also more effective than the Yuzpe regimen, preventing 8
expected pregnancies among women who correctly used the treatment. It also showed tha
either treatment was taken after the act of unprotected intercourse, the more effective it w
then, it has been demonstrated that both 750 microgm pills can be taken together in a sing
mg levonorgestrel) with the same effectiveness and side effects as the two dose regimen.
A primary mechanism of action of ECPs is to inhibit, delay or otherwise interfere with norm
thereby preventing fertilization. ECPs may also alter the endometrium and impair implanta
one study found that use of levonorgestrel ECPs did not impair endometrial morphology.
ECP use does not disrupt an established pregnancy.
Emergency Use of Copper-Bearing IUDs
Copper-bearing IUDs are extremely effective when used as emergency contraception (99%
The IUD must be inserted within 5 days of unprotected intercourse. This method is most ap
women who want long-term, highly effective contraception, and have no contraindications

(Q.172) On follow up study, elevated CA 125 in a case of epithelial ovarian tumor should be further evaluated with?
(a) MRI
(b) CT Scan
(c) Yearly follow up
(d) Clinical examination and serial CA 125 monitoring
Your Response : d
Correct Answer : B
Exp: CT Scan
Evaluation of patients with suspected ovarian cancer should include measurement of serum
tumor marker CA-125.
CA-125 determinants are glycoproteins with molecular masses from 220 to 1000 kDa, and
radioimmunoassay is used to determine circulating CA-125 antigen levels. Between 80 an
patients with epithelial ovarian cancer have levels of CA-125 _ 35 U/mL.
Other malignant tumors can also elevate CA-125 levels, including cancers of the endometri
fallopian tubes, pancreas, breast, lung, and colon.
Certain nonmalignant conditions that can produce moderate elevations of CA-125 levels in
pregnancy, endometriosis, pelvic inflammatory disease, and uterine fibroids. About 1% of n
have serum CA-125 levels _35 U/mL.

(Q.173) Which procedure has the highest incidence of ureteric injury:


(a) Abdominal hysterectomy
(b) Vaginal hysterectomy
(c) Wertheim's hysterectomy
(d) Adnexectomy
Your Response : b
Correct Answer : C
Exp: Wertheim's hysterectomy
The incidence of ureteral injury has increased during the past four to five decades since rem
uterine cervix has become a standard procedure with abdominal hysterectomy.
Removal of the cervix is the riskiest part of the total abdominal hysterectomy procedure, b
injuries occur in the lowest 3 cm of ureter just above the bladder.
Gynecologic Procedures Associated with Ureteral Injury
Operative Procedure Incidence of Ureteral Injury (%)
Abdominal hysterectomy 0.5 - 1.0
Vaginal hysterectomy 0.1
Extensive Wertheim 1.0 - 2.0
Hysterectomy
Adnexectomy 0.1
Marshall-Marchetti-Krantz <0.1
procedure

(Q.174) Which is raised in polycystic ovarian syndrome:


(a) 17-a1pha-OH-progesterone
(b) FSH
(c) LH
(d) TSH
Your Response : b
Correct Answer : C
Exp: Luteinizing hormone
Polycystic ovarian syndrome (PCOS) OR disease (PCOD)/ stein Leventhal syndrome
PCOS includes chronic non-ovulation and hyperandrogenemia associated with normal or ra
(E2), raised LH, and low FSH/LH ratio.
Age - 15-25 years
Features
The raised E2 level causes negative feedback to pituitary resulting in diminished FSH, but ra
The involvement of adrenal glands is seen in raised androstenedione, dehydroepiandroster
testosterone and 17-alpha-Hydroxy-progesterone.
Much of the testosterone is secreted by the ovarian stroma.
Macroscopically
Ovaries
Bilateral Enlarged Lobulated
Free of Adhesion
"Necklace" appearance on ultrasound
Clinical Features
The women is young who complains of oligomenorrhea and often amenorrhea.
Infertility occurs in 30%.
Obesity and hirsutism.
Diagnosis
Laparoscopic evaluation is not only diagnostic, but also therapeutic.
Treatment of PCOS
Oestrogen suppresses androgen and adrenal production. It is best given with progestogen w
androgenic properties, cyclically as in oral contraceptives.
Dexamethasone 0.5 mg or prednisone 5 mg at bedtime also reduces androgen production.
Hirsutism is treated with cyproterone acetate or spironolactone.
Infertility is treated with clomiphene.
In clomiphene failed group, ovulation can be induced with FSH or GnRH analogues.
Surgery is reserved for those in whom:
Medical therapy fails
Hyperstimulation occurs
Use of GnRH analogues is a cost constraint.
Surgery comprises laparoscopic multiple punctures of the cysts with electrocautery or laser

(Q.175) The Investigation of choice in a 55-year-old postmenopausal women who has presented with postmenopausal bleed
is:
(a) Pap smear
(b) Fractional curettage
(c) Transvaginal ultrasound
(d) CA-125 estimation
Your Response : a
Correct Answer : B
Exp: Fractional curettage
Common causes of postmenopausal bleeding
Genital malignancy (Me)
Decubitus ulcer
DUB .
Urethral caruncle .
Senile endometritis
DIAGNOSIS
Speculum Examination
If a growth is found, punch biopsy is to be taken.
If no growth is visible and the cervix looks apparently healthy
. Cervical smear
Aspiration cytology
Fractional curettage, if the cervical cytology becomes negative.
Hysteroscopic evaluation and directed biopsy.
Laparoscopy in suspected cases of ovarian or adnexal mass.
Detection of a benign lesion should not prevent further detailed investigations to rule out
malignancy.

(Q.176) True about puerperal sepsis is


(a) Vagina is the most common site of puerperal infection
(b) May be complicated by salpingitis and pelvic cellulitis but peritonitis never occur
(c) The time limit for defining puerperal pyrexia is 2nd day - 10th day following delivery
(d) Endosalpingitis is commonly caused by direct spread
Your Response : c
Correct Answer : C
Exp: The time limit for defining puerperal pyrexia is 2nd day 10th day following delivery
Puerperal pyrexia is a rise of temperature reaching 100.4F or more (measured orally) on 2
occasion of 24 hours apart (excluding first 24 hours) within first 10 days following delivery.
most common site of puerperal infection in the form of endomyometritis. Salpingitis by dir
not common even with puerperal gonococcal infection. The tube is commonly affected by l
through parametritis. Peritonitis may occur by - spread of infection directly through the tub
of parametric abscess or microabscess in the myometrium or lymphatic spread from myom

(Q.177) If the twins are formed by fertilization of two separate ova, which of the following condition could not occur?
(a) Fetus papyraceous
(b) Fetus acardiacus
(c) Vanishing twin
(d) Pre-eclampsia
Your Response : a
Correct Answer : B
Exp: Fetus acardiacus
Fetus acardiacus occurs only in monozygotic twins. Parts of the fetus remains amorphous a
parasitic without a heart.

(Q.178) Hingorani sign is associated with which disease in pregnancy:


(a) Ovarian tumor
(b) Fibroid
(c) Syphilis
(d) Lymphogranuloma venereum
Your Response : b
Correct Answer : A
Exp: Ovarian tumor
In a case of ovarian tumor abdominal exanimation reveals the cystic swelling felt separated
uterus. In later month of pregnancy confusion may arise. The patient is examined vaginally
Trendelenburg position to elicit the groove between the two swelling i.e. gravid uterus and
tumor (Hingorani sign)

(Q.179) All are true about non-immune fetal hydrops (NIFH) Except
(a) May be related to toxoplasma infection
(b) Accumulation of extracellular fluid in tissues and serous cavities
(c) In presence of structural abnormality prognosis is usually good
(d) Doppler's flow study is useful in prenatal diagnosis
Your Response : b
Correct Answer : C
Exp: In presence of structural abnormality prognosis is usually good
NIHF is defined as accumulation of extra-cellular fluid in tissues and serous cavities in cond
than Rh incompatibility. The various etiologies are chromosomal, congenital, structural ano
hematological infections, placental factors, maternal diseases and idiopathic. Parvovirus, to
syphilis and rubella may cause NIHF. Prenatal diagnosis is possible with high resolution ultr
Doppler's flow study and cordocentesis. Perinatal mortality is high in 50-100%. Prognosis is
worse with presence of structural abnormalities.

(Q.180) FL/AC ratio, which strongly suggest fetal malnutrition:


(a) > 21.5
(b) > 23.5
(c) > 22.5
(d) > 29.5
Your Response : c
Correct Answer : B
Exp: > 23.5
The various parameters used for fetal growth assessment by USG including head circumfer
abdominal circumference, femur length to abdomen ratio and ponderal index. Ponderal ind
determined by dividing the estimated fetal weight by the third power of femur length. Fem
not affected in asymmetric IDGR. The FL/AC ratio is 22 at all gestational ages from 21 week
FL/AC ratio greater than 23.5 suggests IDGR.

(Q.181) BFHI (baby friendly hospital initiative) is launched by


(a) WHO
(b) WHO & UNICEF
(c) India Government
(d) None of the above
Your Response : b
Correct Answer : B
Exp: WHO & UNICEF
In 1992, UNICEF and WHO launched the baby friendly hospital initiative amongst doctors, n
workers and parents, in hospitals and maternity centers to promote, protect and support b
The object is to reestablish the superiority of breast feeding in order to protect the newbor
becoming baby friendly. To fulfill the initiative, UNICEF and WHO laid down ten steps to cre
friendly environment.

(Q.182) Leash sign is found in:


(a) Ectopic Pregnancy
(b) Vulvovaginitis
(c) Polyhydramnios
(d) Tubercular Salpingitis
Your Response : c
Correct Answer : A
Exp: Ectopic Pregnancy
All cases of ectopic pregnancies had a typical eccentric leash of vessels on color Doppler t
low resistance placental type of flow on spectral Doppler. This is called leash sign. This sig
sensitivity of 100% and specifity of 99% a positive predictive value of95% and negative pred
100% thus helping in the diagnosis of early ectopic pregnancy, and resulting in earlier treat
reduced morbidity and mortality.

(Q.183) Intrapartum automated fetal ECG has the advantage over conventional cardiotocography of :
(a) Improving neonatal outcome
(b) Decreasing caesarean section rate
(c) Decreasing need for fetal blood sampling during labour.
(d) All of the above
Your Response : b
Correct Answer : C
Exp: Decreasing the need for fetal blood sampling during labour.
Intrapartum automated fetal electrocardiography decreases the need for fetal blood sampl
labour in comparison with conventional cardiotocography, however it has no impact on ne
come and it does not decreases caesarean section rate.

(Q.184) Rohr's stria found in the placenta are due to:


(a) Deposition of fibrin
(b) Blood vessels
(c) Langhan's cells accumulation
(d) Hofbauer cells accumulation
Your Response : c
Correct Answer : A
Exp: Deposition of fibrin
There may be inconsistent deposition of fibrin called Rohr's stria at the bottom of the int
spaceand surrounding the fastening villi.

(Q.185) The half-life (minutes) of oxytocin is


(a) 1-2
(b) 3-4
(c) 5-6
(d) 8-10
Your Response : b
Correct Answer : B
Exp: 3-4 minutes
Oxytocin is an octapeptide, synthesized in supraoptic and paraventricular nuclei of the hyp
carrier proteins, it is transported from hypothalamus to posterior pituitary where it is even
It has a half-life 00-4 minutes and duration of action of approximately 20 minutes. It is rapid
metabolized and degraded by oxytocinase.

(Q.186) Teratospermia refers to


(a) Increased abnormal forms
(b) Decreased count
(c) Decreased motility
(d) Absent semen
Your Response : a
Correct Answer : A
Exp: Increased abnormal forms
Teratospermia -----increased number of abnormal sperms. Regarding sperm morphology, t
criteria as described in 2010 state that a sample is normal (samples from men whose partn
pregnancy in the last 12 months) if 4% (or 5th centile) or more of the observed sperm have
morphology.
Aspermia-------------absence of semen
Azoospermia--------absence of sperms in semen. Total sperm count, or total sperm number
number of spermatozoa in the entire ejaculate. By WHO, lower reference limit (2.5th perce
million per ejaculate
Oligospermia-------decreased sperm count
Asthenospermia----decreased motility of sperms. The World Health Organization has a valu
this must be measured within 60 minutes of collection
Necrospermia-------increased number of dead sperms.

(Q.187) The main source of production of relaxin is:


(a) Ovary
(b) Placenta
(c) Decidua
(d) Adrenals
Your Response : a
Correct Answer : A
Exp: Ovary
The main source of production of relaxin is the corpus luteum of the ovary but part of it ma
produced by the placenta and decidua.

(Q.188) Fetal biophysical profile used for the assessment of fetal well being does not
includes:
(a) Fetal breathing movements
(b) Fetal tone
(c) Amount fluid volume
(d) Fetal blood PH
Your
d
Response :
Correct
D
Answer :
Exp: Fetal blood PH
Fetal biophysical profile considers several parameters, which
includes:
. Non Stress test . Fetal breathing movements
. Gross body movements . Fetal muscle tone . Amniotic
fluid volume

(Q.189) The correct formula for estimation of iron requirement is


(a) 4.4 x weight (pounds) [100 - Hb%]
(b) 4.4 x weight (kg) [100 - Hb%]
(c) 0.3 x weight (pounds) [100 - Hb%]
(d) 0.3 x weight (kg) [100 - Hb%]
Your Response
b
:
Correct
C
Answer :
Exp: 0.3 x weight (pounds) [100 - Hb%]
For estimation of total iron requirement in iron deficiency anemia, 2
formulae can be used:
(1) 0.3 x W (pounds) [100 - Hb%]
Additional 50% is to be added for partial replenishment of the body store
iron.
(2) 4.4 x W (kg) x Hb deficit (g/dL)
This formula included iron needed for replenishment of iron stores.

(Q.190) Most common cause of hematocolpos


(a) Cervical atresia
(b) Vaginal atresia
(c) Transverse vaginal septum
(d) Imperforate hymen
Your Response : d
Correct Answer : D
Exp: Imperforate hymen
Hematocolpos is collection of blood in the vagina. Which means the level of obstruction sh
orbelow the hymen. Cervical atresia will result In hematometra. The commonest cause is i
hymen followed by low transverse vaginal septum. The complications include
hematometra, hematosalpinx ,endometriosis and infertility.

(Q.191) GnRh analogues are used in


(a) Ovulation induction
(b) Endometriosis
(c) Fibroid
(d) All of the above
Your Response : d
Correct Answer : D
Exp: All of the above
GnRh includes both agonists and antagonists. Used for ovulation induction, fibroid
and endometriosis . It induces a state of pseudomenopause were there is a deficiency of b
and progesterone so that further growth of fibroid and endometriosis does not occur. To o
distressing s/e of menopause E+P are added which is called as Add Back regime. Other co
which they are used is true precocious puberty in which there is activation of HPO axis .

(Q.192) Following delivery, uterine fundus lie about


(a) 101/2 above pubic symphysis
(b) 81/2 above pubic symphysis
(c) 51/2 above pubic symphysis
(d) 21/2" above pubic symphysis
Your Response : b
Correct Answer : C
Exp: 51/2 above pubic symphysis
Following delivery, the fundus lies about 13.5 cm (51/2) above the pubic symphysis. Durin
hrs the level remain constant, thereafter there is steady decrease in height by 1.25 cm in
that by the end of 2nd week the uterus becomes a pelvic organ. These measurements are
significant to know the process of involution. If the height is more than the period of puerp
be because of subinvolution.

(Q.193) Most common site of endometriosis is


(a) Fallopian tube
(b) Ovary
(c) Uterus
(d) Pouch of Douglas
Your Response : b
Correct Answer : B
Exp: Ovary
Ovary is the most common site of endometriosis (75%). Following this are POD (70%), ute
of peritoneum (35%), posterior aspect of pelvic wall and uterosacral ligament (35%), ureter
(10%).
The lowest incidence is seen in small intestine <1% .
This is due to increased activity and peristalsis.

(Q.194) Smooth dilatation of cervix is due to?


(a) Breaking down of collagen fibrils
(b) Accumulation of fluid in between collagen fibres
(c) Fibro-musculo-glandular hypertrophy
(d) All of the above
Your Response : d
Correct Answer : D
Exp: All of the above
Various predisposing factors which favour smooth dilation of cervix are-softening of cervix,
musculoglandular hypertrophy, increased vascularity, accumulation of fluid in between col
breaking down of collagen fibrils by collagenase and elastase, alteration in proteoglycan an
glycosaminoglycan, increased hyaluronic acid and decreased dermatan sulphate.

(Q.195) Diagnostic procedure for postmenopausal bleeding


(a) Endometrial aspiration
(b) Hysteroscopic guided biopsy
(c) Dilatation and curettage
(d) Ultrasound
Your Response : b
Correct Answer : B
Exp: Hysteroscopic guided biopsy
Gold standard is hysteroscopic directed endometrial sampling. Next best is endometrial a
gives a global endometrial sample by virtue of suction. D and C and EB should never be don
the cause of postmenopausal bleeding. The most common cause overall is senile endome
most common malignant cause is endometrial cancer

(Q.196) Most common cause of UTI in pregnancy


(a) Proteus mirabilis
(b) Group B haemolytic streptococcus
(c) E. coli
(d) Klebsiella
Your Response : a
Correct Answer : C
Exp: E. coli
Urinary tract infections are more concerning in pregnancy due to the increased risk of kidn
During pregnancy, high progesterone levels elevate the risk of decreased muscle tone of th
bladder, which leads to a greater likelihood of reflux, where urine flows back up the ureter
the kidneys.
While pregnant women do not have an increased risk of asymptomatic bacteriuria, if bact
present they do have a 25-40% risk of a kidney infection.
Thus if urine testing shows signs of an infectioneven in the absence of symptomstrea
recommended. Cephalexin or nitrofurantoin are typically used because they are generall
safe in pregnancy. A kidney infection during pregnancy may result in premature birth or pr
state of high blood pressure and kidney dysfunction during pregnancy that can lead to seizu
E coli is the most common cause of urinary tract infection (UTI), accounting for approximat
cases. It originates from fecal flora colonizing the periurethral area, causing an ascending in

(Q.197) False about complications of twin pregnancy is?


(a) Malpresentation is more common in 2nd baby
(b) Placental abruption may occur because of associated PIH
(c) Hydramnios is more common in binovular twins
(d) Twin to twin transfusion syndrome exclusively met with uniovular twins
Your Response : b
Correct Answer : C
Exp: Hydramnios is more common in binovular twins
Twin pregnancy may be associated with a number of complications: anemia, preeclampsia,
malpresentation, preterm labour, early rupture of membrane, cord prolapse, PPH, growth
baby, fetal anomalies, fetal death and twin to twin transfusion syndrome.
Amongst these some complications are exclusively seen in uniovular twins like hydramnios
twin transfusion syndrome. Malpresentation is more common in 2nd baby.
Placental abruption may occur because of increased incidence of PIH, sudden escape of liqu
rupture of membrane of hydramnios sac and following delivery of first baby due to sudden
decompression.

(Q.198) Vaccination that is contraindicated in pregnancy is


(a) Yellow fever
(b) Rubella
(c) Tetanus toxoid
(d) Rabies
Your Response
b
:
Correct Answer
B
:
Exp: Rubella
Live viral vaccines that are contraindicated include measles, mumps,
rubella, varicella, polio
Live vaccines that are to be used with caution---- Rabies, Yellow fever.
Killed vaccines with caution ---Japanese encephalitis, cholera, plague,
typhoid.

(Q.199) Most common cause of MMR in India


(a) Anaemia
(b) Hemorrhage
(c) Sepsis
(d) PIH
Your Response : b
Correct Answer : B
Exp: Hemorrhage
The maternal mortality rate (MMR) is the annual number of female deaths per 100,000 live
any cause related to or aggravated by pregnancy or its management (excluding accidental o
causes). The MMR includes deaths during pregnancy, childbirth, or within 42 days of
termination of pregnancy, irrespective of the duration and site of the pregnancy, for a spec
Maternal mortality rate: 200 deaths/100,000 live births (2010)
Causes of maternal deaths
Direct Causes: 81%
Severe Bleeding 25%
Sepsis 15%
Unsafe abortions 13%
Eclampsia 12%
Obstructed Labour 8%
Other direct causes * 8%
Indirect Causes ** 19%
* Other direct causes: Ectopic pregnancy, embolism, anaesthesia related.
** Indirect Causes: Malaria, Anaemia, heart diseases.
(Q.200) Secondary amenorrhea is one of the clinical features of all the following conditions except?
(a) Genital tuberculosis
(b) Endometriosis
(c) Premature ovarian failure
(d) Hyperprolactinemia
Your Response : c
Correct Answer : B
Exp: Endometriosis
Secondary amenorrhea is not the clinical feature of endometriosis.
The various clinical features of endometriosis are:
Dysmenorrhea: Commonest symptom.
Lower abdominal pain.
Dyspareunia.
Infertility.
Menorrhagia and irregular bleeding.
Chronic pelvic pain.
Frequency of micturition and dysuria.
Genital tuberculosis, premature ovarian failure and hyperprolactinemia are important causes
secondary amenorrhea.
Premature ovarian failure means ovarian failure before the age of 40.
Hyperprolactinemia leads to amenorrhea with galactorrhea.

(Q.1) Ulipristal acetate is used for


(a) Breast cancer
(b) Endometriosis
(c) Emergency contraception
(d) AUB
Your Response :
Correct Answer : c
Exp: This drug is a SPRM (selective progesterone receptor modulator). It is used for
contraception and is the only hormonal contraception that is effective for 120
contraindicated in hepatic disorder and in lactational mothers.
For emergency contraception. a 30 mg tablet is used within 120 hours (5 days
unprotected intercourse or contraceptive failure. It has been shown to preven
of expected pregnancies, and prevents more pregnancies than emergency con
with levonorgestrel.
Ulipristal acetate is used for pre-operative treatment of moderate to severe sy
uterine fibroids in adult women of reproductive age in a daily dose of a 5 mg t
(Q.2) Risk of endometrial cancer increases with
(a) Nulliparity
(b) Prolonged lactation
(c) PCOD
(d) Both a and c
Your Response :
Correct Answer : d
Exp: Endometrial cancer is associated with hyperestrogenic states
. The preneoplastic statesincludes hyperplasias( simple typical, simple atypica
complex atypical).
The risk factors are
a. Nulliparity
b. Early age at menarche
c. Late age at menopause
d. PCOD
e. Exogenous administration of estrogen
f. Obesity
g. Non lactational
The Protective factors include
a. Multiparity
b. Prolonged lactation
c. OCPs
d. Smoking

(Q.3) Best timing of transabdominal chorionic villous biopsy


(a) 9 11 weeks
(b) 11-13 weeks
(c) 12 14 weeks
(d) 5 7 weeks
Your Response :
Correct Answer : a
Exp: Chorionic villus sampling (CVS) is a form ofprenatal diagnosisto
determinechromosomalorgenetic disordersin thefetus.
It entails getting a sample of thechorionic villus(placentaltissue) and testing it.
CVS can be carried out 9-11 weeks after the last period, earlier than amniocen
is carried out at 16-20 weeks).
CVS can be performed via trans abdominal or trans cervical route.
Indications
Mother's age of 35 years or greater
Abnormal first trimester screen results
Increasednuchal translucencyor other abnormalultrasoundfindings
Past history of achromosomal abnormalityor othergenetic disorder
Parents are known carriers for agenetic disorder
Risk
There is a risk of miscarriage(0.8-1%)
There is a risk of infection
There is a risk of amniotic fluidleakage.
The resulting amniotic fluid leak can cause.
If the resultingoligohydramniosis not treated and the amniotic fluid continues
can result in the baby developing hypoplastic lungs (underdeveloped lungs).
Additionally, there is a risk of CVS causing digit/limb-reduction defects & oro m
defects if performed before 11 weeks.
Therefore the recommendation to perform CVS is after 10 completed weeks.
Summary table of all tests used for screening / Diagnosing congenital anomali
Investigation Period of Gestation Material of study
Maternal serum 15 20 weeks Maternal serum alfa fe
Alpha feto protein
Triple test 15 18 wks MSAFP ()
Unconjugated Estriol (
Free beta hCG ()
Integrated test 10 wks Nuchal translucency (
14 20 wks PAPP A ()
MSAFP ()
Unconjugated estriol (
Beta HCG ()
Inhibin ()
USG 10 14 wks Fetal Anatomy to see n
16 22 wks translucency
Fetal anatomy
Amniocentesis 14 16 wks Fibroblast
Early 12 14 wks Amniotic
Amniocentesis (can be done at 10 wks) Disorders
Chorionic villi sampling 10 12 weeks Trophoblast cells of ch
Cordocentesis 18 20 weeks Blood and fetal white b
(Percutaneous umbilical
blood sampling)
Fetoscopy 16 20 weeks Visualization of anatom
Note:
This question has been asked in an earlier all India as well. Then the choices w
12, 12-14,14-16. Williams obstetrics write the best time as 10-13 weeks. Now
question the choice are odd choices, i.e 9-11, 11-13. The best time here is 9-1
11 weeks and later is infact too late. The time chosen by almost all centers aro
world to do CVS is in fact 9-11 weeks.
(Q.4) The most reliable fetal parameter forestimation of gestational age in first trimester is
(a) Femur length
(b) G sac diameter
(c) CRL
(d) BPD
Your Response :
Correct Answer : c
Exp: Gestational age (GA) in the first trimester is usually calculated from the fetal c
length (CRL). This is the longest demonstrable length of the embryo or fetus, e
thelimbs and the yolk sac. The correlation between CRL and GA is excellent un
approximately 12 weeks. No sex or race differences are appreciable, but mate
characteristics, such as age and smoking, may have a significant effect beyond
gestation. The GA estimate has a 95% confidence interval of plus or minus 6 d
most accurate between 7 and 10 weeks of pregnancy. Transabdominal ultraso
may underestimate gestational age by an average of 1.6 days compared with
ultrasonography.

(Q.5) Most common serotype of HPV associated with invasive cervical carcinoma is?
(a) HPV 16
(b) HPV 18
(c) HPV 32
(d) HPV 36
Your Response :
Correct Answer : a
Exp: HPV is associated with 80% of CIN and over 90% of invasive carcinoma
'HPV -16 is most common HPV associated with squamous cell carcinoma. HPV
common HPV associated with Aden carcinoma.' -CGDT 9th ed pg 904
As it is well known that histologically 95% of cervical ca are SCC and only 5% a
Hence HPV is the most common serotype of HPV associated with cervical ca.
On the basis of strength of association to cacervix, HPV has been classified int
Low risk type-6, 11, 42, 43
Causes CIN and condyloma acuminate
Intermediate risk-33, 35, 55, 52
Causes CINI and CIN that don't progress.
High risk- 16, 18,31,39,45,56,48,59
Causes high grade CIN that progresses to invasive ca.
(Q.6) Shape of nulliparous cervix is:
(a) Transverse
(b) Longitudinal
(c) Circular
(d) T shaped
Your Response :
Correct Answer : c
Exp: Shape of nulliparous cervix is circular the cervix is spindle shaped and measure
is bounded above by internal os and below by external os. In nulliparous wom
external os is circular whereas parous cervix is characterized by a transverse s

(Q.7) Fibroid causes all the following except


(a) Infertility
(b) Amenorrhea
(c) Pelvic mass
(d) Menorrhagia
Your Response :
Correct Answer : b
Exp: Symptoms of fibroid are:
Menorrhagia, polymenorrhea and metrorrhagia
Infertility, recurrent abortions
Pain
Abdominal lump
Pressure symptoms
Vaginal discharge
Around 50% women are asymptomatic.
Important points:
These are benign neoplasms occurring in 5-20% of women in reproductive age
Oestrogen dependence on growth is most imp etiological factor, so rare befor
and after menopause.
Calcification begins at periphery where as degeneration begins in central port
Distribution of myoma in body of uterus is intramural (75%), submucous (15%
subserous (10%).
Red degeneration is seen in 2nd half of pregnancy and needs conservative ma
Sarcomatous change is seen in 0.5% of all myomas. It occurs in post menopau
and characterized by sudden growth, pain and post menopausal bleeding.
Endometrial ca is associated with fibromyomas in women over 40 yrs of age in
(Q.8) An ovarian TUMOUR was identified in immediate post partum period, the timing of surgery will be
(a) Immediately after delivery
(b) 6 wks
(c) 1 wk
(d) 3 mths
Your Response :
Correct Answer : a
Exp: Treatment of ovarian tumour in pregnancy depending on time of pregnancy:
During pregnancy
In an uncomplicated case, the best time of elective operation is b/w 14-18wks
of abortion are less and access to the pedicle is easy. But if the tumour is diag
beyond 36 weeks, tumour is removed early in puerperium.
During labour
If the tumour is above the presenting part watchful expectancy is followed bu
if itsimpacted in pelvis, caesarian section should be done followed by removal
in same setting.
During puerperium
The tumour should be removed as early in puerperium as possible.

(Q.9) Treponema pallidum crosses placenta:


(a) After 36 weeks
(b) After 28 weeks
(c) After 2nd trimester
(d) At any stage of pregnancy
Your Response :
Correct Answer : d
Exp: Transmission of Treponema pallidum from an infected woman to her fetus acr
placenta may occur at any stage of pregnancy, but the lesions of congenital sy
usually develop after-the 4th month of gestation, when "fetal immunologic" c
begins to develop.
(Q.10) Contraindication of vaginal delivery in female with previous cesarean section include all EXCEPT.
(a) Previous classical cesarean section
(b) Breech presentation
(c) No history of vaginal delivery in the past
(d) Puerperal infection in previous pregnancy
Your Response :
Correct Answer : b
Exp: Absolute indications for cesarean section are:
(Where vaginal delivery is not possible cesarean section is needed even with
fetus)
a. Central placenta previa
b. Contracted pelvis or cephalopelvic disproportion absolute
c. Pelvic mass causing obstruction (cervical or broad ligament fibroid)
d. Advanced carcinoma cervix
e. Vaginal obstruction (atresia, stenosis)
Common indications are
Primigravida
a. Cephalopelvic disproportion
b. Fetal distress
c. Dystocia
Multigravida
a. Previous cesarean delivery, particularly previous classical cesarean section
b. Antepartum hemorrhage (placenta previa, placental abruption)
c. Malpresentation (breech)
Relative indications for cesarean section:
(Vaginal delivery may be possible with or without aids. But risks to the mothe
the baby are high. More often multiple factors may be responsible. Indication
common than the absolute ones)
a. Cephalo-pelvic disproportion (relative)
b. Previous cesarean delivery
When primary C.S. was due to recurrent indication (contracted pelvis)
Previous two C.S.
Features of scar dehiscence
Previous classical C.S.
c. Non reassuring FHR (fetal distress)
d. Dystocia may be due to three Ps relatively large fetus (Passenger), small pe
(Passage) or inefficient uterine contractions (Power).
e. Antepartum hemorrhage
Placenta previa and
Abruptio placenta
f. Malpresentation
Breech, Shoulder (transverse lie), Brow
g. Failed surgical induction of labor, Failure to progress in labor
h. Bad obstetric history with recurrent fetal wastage
i. Hypertensive disorders
Severe pre-eclampsia
Eclampsia uncontrolled fits even with anti seizure therapy
j. MedicalGynecological disorders
Diabetes (uncontrolled), heart disease (coarctation of aorta),
Marfans syndrome
Mechanical obstruction (due to benign or malignant pelvic tumors (carcino
or following repair of vesico-vaginal fistula
Note
a. Previous Classical Cesarean section presents with increased risk of scar rup
subsequent labor and one can permit vaginal delivery with greater confidenc
lower segment section than after classical section
b. No History of Vaginal Delivery in the past points towards a persisting indic
contracted pelvis) and it is better to do Elective Cesarean rather than try for V
Delivery
c. Breech presentation per se in a patient with previous LSCS is not an indicat
elective repeat cesarean section, if the baby is average in size and pelvis adeq
d. Puerperal infection in previous pregnancy may result in bad healing and he
scar.

(Q.11) Most common site for endometriosis is:


(a) Ovary
(b) Peritoneum
(c) Appendix
(d) Urinary bladder
Your Response :
Correct Answer : a
Exp: Endometriosis-is characterized by the presence and proliferation of endomet
(stroma and glands) outside the endometrial cavity. It occurs most commonly
the ages of 30-40 and is found incidentally at the time of surgery. The most c
involved sites are the ovary, fallopian tubes, uterine ligaments, and pelvic pe
The major symptom is pelvic pain, characteristically dysmenorrhea. The fertil
reduced.
(Q.12) Carcinoma cervix presents with
(a) Postcoital bleed
(b) Abnormal vaginal bleed
(c) Purulent discharge per vaginum
(d) All of the above
Your Response :
Correct Answer : d
Exp: Carcinoma cervix has a bimodal peak one at 30-35 yrs and a second peak at 5
.Presentation is typically abnormal vaginal bleeding --- in reproductive age gr
metrorrhagia and in postmenopausal women with postmenopausal bleeding
be a cervical growth which can lead to postcoital bleeding . It it undergoes ne
there can be purulent discharge per vaginum. Sometimes cancer cervix can p
VVF and with also with features of uraemia.

(Q.13) Pregnancy is contraindicated in all except:


(a) Eisenmenger syndrome
(b) Primary pulmonary hypertension
(c) Marfan syndrome with aortic root dilation
(d) WPW syndrome
Your Response :
Correct Answer : d
Exp: Eisenmengers syndrome carries a very high mortality in pregnancy(upto 50%
pregnancy and cesarean section are both contraindicated in this condition.
Per se in heart disease patients cesarean section is done only for obstetric ind
Heart disease in which elective cesarean section should be done is:
Marfans syndrome with aortic root dilatation > 4cm(due to risk of aortic dis
during labour) coarctation of aorta and aortic aneurysm should also be prefe
delivered by cesarean section due to risk of rupture during labour
Clarkes Classification Of Mortality In Heart Disease In Pregnancy
Low Risk (mortality< 1%)
Atrial septal defect
Ventricular septal defect
Patent ductus arteriosus
Pulmonary/tricuspid disease
MS(NYHA 1&2)
Corrected TOF
Intermediate Risk(5-15%)
Un corrected TOF
UN complicated Coarctation of the aorta
Marfan syndrome with a normal aortic root
MS( NYHA 3 & 4)
AS
High Risk( 25-50%) (CONTRA INDICATIONS TO PREGNANCY)
Eisenmenger's syndrome
Primary pulmonary hypertension
Complicated coarctation of aorta
Marfan syndrome with aortic root or valve involvement
(Q.14) Puerperal fever from breast engorgement
(a) Appears in less than 5% of postpartum women
(b) Appears 3 to 4 days after the development of lacteal secretion
(c) Is almost painless
(d) Is less severe and less common if lactation is suppressed
Your Response :
Correct Answer : d
Exp: Puerperal fever from breast engorgement is relatively uncommon, affecting 1
post partum women. It appears 24 to 48 h following initiation of lacteal secre
ranges from 38 to 39C (100.4 to 102.2F). Pain is an early and common symp
Treatment consists of breast support, ice packs, and pain relievers. The incide
severity of breast engorgement are lower if treatment is given for suppressio
lactation.

(Q.15) Maximum increase in cardiac output during pregnancy is seen in:


(a) 32 weeks
(b) 36 weeks
(c) During labour
(d) Just after delivery
Your Response :
Correct Answer : d
Exp: The cardiac output starts to increase from 5th wk of pregnancy and reaches i
50%) at about 30-34 wks. Thereafter it remains static till labour (+ 50%) and i
following delivery(+ 70%) over the pre-labour values. This increase in cardiac
due to squeezing out of blood from the uterus into the maternal circulation (
transfusion) during labour and in the immediate post partum.
Cardiac output returns to the pre labour values by 1hr following delivery and
pregnant levels by 4 weeks time.
(Q.16) Gartner's duct cyst is seen in:
(a) Vagina
(b) Cervix
(c) Hymen
(d) Labia minora
Your Response :
Correct Answer : a
Exp: The most Gammon type of vaginal cysts is Gartner's duct Cyst. These cysts ar
caudal remnants of the mesonephric or Wolffian duct system. They are usual
along the anterolateral aspect of the vaginal walls and are frequently bilatera
Gartner's ducts are remnants of the mesonephric (Wolffian) duct in the fema
consist of a series of tiny vestigial cysts extending along the lateral aspect of t
to the vaginal vestibule.

(Q.17) Fetal lung maturity is assessed by all the following except


(a) Lecithin sphingomyelin ratio
(b) Nile blue sulfate test
(c) Nitrazine paper test
(d) Creatinine estimation of amniotic fluid
Your Response :
Correct Answer : c
Exp: Nitrazine paper is used to detect small quantities of 'amniotic fluid' in vaginal
It is used in conjunction with the fern test to help detect ruptured membrane
positive results may occur from specimen contamination due to heavy vagina
blood, cervical mucus, semen, alkaline urine, and soap. Fa/se negative result
produced by prolonged rupture of membranes (> 24 hours) or when a small v
fluid has leaked. Specimen contamination will result in erroneous pH results.
(Q.18) The only non hormonal treatment of hot flushes approved byFDA ?
(a) Fluoxetine
(b) Paroxetin
(c) Estriol
(d) None of theabove
Your Response :
Correct Answer : b
Exp: There are a variety of FDA-approved treatments for hot flashes, but all conta
estrogen alone or estrogen plus a progestin.Hot flashes associated with men
occur in up to 75 percent of women and can persist for up to five years, or ev
somewomen . Brisdelle, which contains the selective serotonin reuptake inhi
paroxetine mesylate, is currently the only non-hormonal treatment for hot fl
approved by the FDA. The mechanism by which Brisdelle reduces hot flashes
unknown.The most common side effects in patients treated with Brisdelle we
headache, fatigue, and nausea/vomiting.

(Q.19) Clomiphene citrate is used in:


(a) Carcinoma endometrium
(b) Asherman's syndrome
(c) Stein Leventhal syndrome
(d) Adrenogenital syndrome
Your Response :
Correct Answer : c
Exp: Clomiphene is the drug of choice in patients suffering form Stein Leventhal sy
is also used in the treatment of:
Anovulation or oligo-ovulation in patients desiring pregnancy, whose sexual p
have adequate sperm, and who have potentially functional hypothalamic-hyp
ovarian systems and adequate endogenous estrogen;
Corpus luteum dysfunction;
Infertility in males with oligospermia.
(Q.20) Which of the following is the mechanism of abortion sticks?
(a) Stimulation of uterine contraction
(b) Menstrual bleeding
(c) Irritation of the GIT
(d) Irritation of the genito-urinary tract
Your Response :
Correct Answer : a
Exp: The main mechanism of the use of abortion sticks is to produce an irritant ac
uterus so as to induce uterine contractions and expulsion of the fetus.
Abortion stick is an object used to procure illegal abortion by unskilled interfe
either self or by someone else (a dai/midwife).
It usually consists of a specially made wooden or bamboo stick about 1520 c
twig of similar length with some irritant plant such as Madar (Calotropis), Chi
(Plumbago zeylanica) or Kaner (Nerium odorum) applied at one of its ends.
The irritant substance is applied to some cotton wool or a piece of rag at one
stick.
The abortion sick is introduced into the os of the uterus thus causing abortion
without rupture of the membranes.
Excoriation, bruising or perforation of the vagina or uterus may occur as a res
irritant action of the substance.

(Q.21) All the following drugs are used for postcoital contraception except

(a) Danazol

(b) Ethinyl estradiol

(c) Levonorgestrel

(d) Mifepristone

Your Response :
Correct Answer : a

Exp:
Postcoital contraceptive agents interfere with postovulatory events leading t
and are known as interceptive. They are chiefly used following rape, unprote
intercourse or accidental rupture of a condom during coitus which takes plac
ovulation. These agents are:
1. 2 tablets of relatively high doses of combined pill containing
100g ethinyl estradioland 1mg norethisterone are taken within 72 hours of
followed by 2 tablets taken 12 hours later.
2. Ethinyl estradiol 5 mg daily for 5 days starting within 72 hours of exposu
Stilboestrol 50 mg daily for 5 days.
3. IV equine conjugated estrogen twice daily
4. Levonorgestrel 600 g and d norgestrel .5-4 mg for 5 days.
5. RU 486 (Mifepristone)
(Q.22) Treatment of choice for carcinoma cervix stage IB is:
(a) Radiotherapy
(b) External hysterectomy
(c) Abdominal hysterectomy
(d) Pan-hysterectomy
Your Response :
Correct Answer : a
Exp: Carcinoma cervix in situ (stage 0) can generally be successfully managed by c
or by abdominal hysterectomy. For stage I disease, results appear equivalent
radical hysterectomy or radiation therapy. Patients with stages II to IV diseas
primarily managed with radical radiation therapy.

(Q.23) Karyopyknotic index is highest on


(a) Proliferative phase
(b) Day of ovulation
(c) Secretory phase
(d) Menstruation
Your Response :
Correct Answer : b
Exp: An index that is calculated as the percentage of epithelial cells with karyopyk
exfoliated from the vagina and is used in the hormonal evaluation of a patien
influence of estrogen karyopyknotic index increases . As estrogen peaks at ov
just prior the index is highest at ovulation. The smear for the same is taken fr
lateral vaginal wall as cells here respond to cyclical changes in hormone that
the menstrual cycle.
(Q.24) Low maternal serum alpha-fetoprotein is not seen in:
(a) Trophoblastic diseases
(b) Overestimated gestational age
(c) Trisomy 21
(d) Neural tube defect
Your Response :
Correct Answer : d
Exp: Alpha-fetoprotein is an antigen present in the human fetus and in certain
pathological conditions in the adult. The maternal serum level should be
evaluated at 16 to 18 weeks of pregnancy to detect fetal abnormalities. It is
increased in the following conditions:
. Anencephaly . Meningomyelocele (NT D)
. Encephalocele . Omphalocele (Exomphalos)
. Meckel syndrome . Congenital nephrosis
. Impending spontaneous .
abortion
. Rh sensitization . Presence of intestinal atresia
Decreased levels indicate an increased risk of having a baby with Down
syndrome (Trisomy 21).
If an abnormal level of AFP is found, further tests such as ultrasound or
amniocentesis should be done to establish the final diagnosis. Elevated
serum levels are also found in adults with certain hepatic carcinomas or
chemical injuries.

(Q.25) Supports of uterus are allexcept:


(a) Uterosacral ligaments
(b) Mackenrodt's ligament
(c) Broad ligament
(d) Levator ani
Your Response :
Correct Answer : c
Exp: CLASSIFICATION
Primary supports
a. Muscular or active
Pelvic diaphragm
Perineal body
Urogenital diaphragm
b. Fibromuscular or mechanical
Uterine axis
Pub cervical ligament
Transverse cervical ligament
Uterosacral ligament
Round ligament of uterus
Secondary supports
Broad ligaments
Uterovesical fold of peritoneum
Rectovaginal fold of peritoneum Broad Ligaments:
These are folds of peritoneum which attach the uterus to the lateral pelvic w
anterior and posterior surfaces and upper, lower, medial and lateral borders.
the following structures: . Uterine tube
Round lig of uterus
Lig of ovary
Uterine and ovarian plexus and nerve plexus
Epoophoron and paroophoron
Lymphatics
Connective tissue.
(Q.26) Not an absolute contraindication of oral contraceptive pill:
(a) Uterine bleeding
(b) Breast carcinoma
(c) Hydatidiform mole
(d) Genital cancer
Your
Response :
Correct
c
Answer :
Exp: Absolute contraindications of oral
contraceptive pills are:
Cancer of breast and genitals;
Liver disease;
Previous or present history of
thromboembolism;
Cardiac abnormalities;
Congenital hyperlipidemia;
Undiagnosed abnormal uterine bleeding.

(Q.27) Drug not helpful in induction of ovulation is:


(a) Progesterone
(b) Clomiphene
(c) Gonadotropin
(d) Tamoxifen
Your Response :
Correct Answer : a
Exp: Chorionic gonadotropin with menotropins or urofollitropin is the treatment o
induction of ovulation in patients who do not respond to clomiphene.
Clomiphene is indicated in the treatment of anovulation or oligo-ovulation in
desiring pregnancy.
Tamoxifen may induce ovulation in anovulatory women, stimulating release
gonadotropin-releasing hormone from the hypothalamus, which in turn stim
release of pituitary gonadotropins.
(Q.28) Not a feature of pseudocyesis:
(a) Weight gain
(b) Morning sickness
(c) Enlargement of uterus
(d) Amenorrhea
Your Response :
Correct Answer : c
Exp: Pseudocyesis or phantom pregnancy or pseudopregnancy is a condition in wh
patient has nearly all of the usual signs and symptoms of pregnancy (such as
enlargement of the abdomen, weight gain, cessation of menses, and morning
but is not pregnant. This condition is usually seen in women who either are v
of having children or wishes to avoid pregnancy. Treatment usually is done b
means.

(Q.29) The presentation of Asherman syndrome typically involves:


(a) Hypomenorrhea
(b) Oligomenorrhea
(c) Menorrhagia
(d) Metrorrhagia
Your Response :
Correct Answer : a
Exp: Ovulation is not affected in Asherman syndrome. Because of the decreased a
functional endometrium, progressive hypomenorrhea (lighter menstrual flow
amenorrhea is common. The best diagnostic study is the hysterogram under
Hysteroscopy with lysis of adhesions is the treatment of choice. Prophylactic
may improve success rates.
(Q.30) Endosalpingitis is best diagnosed by
(a) Laparoscopy
(b) X-Ray abdomen
(c) Hysterosalpingography
(d) Sono salpingography
Your Response :
Correct Answer : a
Exp: Laparoscopy also called laparo-hysteroscopy is a procedure in which the tube
examined under direct vision with an endoscope and if the pelvis looks inflam
best place to take a biopsy for culture is the tubal end. Same holds true for th
of endo-salpingitis.
X-Ray abdomen is too gross a procedure and informs mostly anatomical prob
pelvis.
HSG is a procedure which will inform regarding the tubal patency and outline
uterine anatomy along with uterine polyps and fibroids.
There may be some filling defects in the tube recognized with a good HSG if t
Endosalpingitis but it does not prove the diagnosis as is done by a direct tuba
Sono salpingography is an assessment of the tubal patency by perfusing the u
therefore the tubes while observing the uterus and the pouch of Douglas by
ultrasound. This procedure is good for the diagnosis of tubal patency but not
infections.

(Q.31) The source of HCG is:


(a) Syncytiotrophoblasts
(b) Cytotrophoblasts
(c) Langhanss layer
(d) Chorionic villi
Your Response :
Correct Answer : a
Exp: (Ref. Textbook of Obstetrics D.C. Dutta 6th Ed. 58)
The Syncytiotrophoblast of the placenta produces HCG. The half-life of HCG i
24hours. By radioimmunoassay, it can be detected in the maternal serum or
early as 8-9 days following ovulation.
REF : WILLIAMS OBSTETRICS 23rd Ed Pg 63 states :
Before 5 weeks, hCG is expressed in both Syncytiotrophoblast and cytotroph
Later when the maternal serum levels peak, it is produced almost solely in
theSyncytiotrophoblast. At this time hCG mRNAs for both alpha and beta su
inSyncytiotrophoblast are greater than at term. This may be an important co
when hCG is used as a screening procedure to identify abnormal fetuses.
For all practical purposes the answer to this question will be A. Syncytiotrop
(Q.32) LH is required for:
(a) Fertilization
(b) Follicular growth
(c) Tubular motility
(d) Menstruation
Your Response :
Correct Answer : b
Exp: In both males and females, LH is essential for reproduction.
In females, at the time of menstruation, FSH initiates follicular growth, specif
affecting granulosa cells. With the rise in oestrogens, LH receptors are also ex
the maturing follicle that produces an increasing amount of estradiol.
Eventually at the time of the maturation of the follicle, the oestrogen rise lea
hypothalamic interface to the positive feed-back effect, a release of LH ove
48-hour period. This 'LH surge' triggers ovulation, thereby not only releasing
also initiating the conversion of the residual follicle into a corpus luteum that
produces progesterone to prepare the endometrium for a possible implantat
LH is necessary to maintain luteal function for the first two weeks. In case of
pregnancy, luteal function will be further maintained by the action of hCG (a
very similar to LH) from the newly established pregnancy. LH supports theca
ovary that provide androgens and hormonal precursors for estradiol product
In the male, LH acts upon the Leydig cells of the testis and is responsible for t
production of testosterone, an androgen that exerts both endocrine activity
intratesticular activity on spermatogenesis.

(Q.33) All of the following ovarian tumours arise from surface epithelium EXCEPT:
(a) Mucinous cystadenoma
(b) Endometrial carcinoma
(c) Brenner tumour
(d) Benign cystic teratoma
Your Response
:
Correct
d
Answer :
Exp: NOTE: Teratoma is a germ cell tumour. Other ovarian germ
cell tumours are:
1. Dysgerminoma
2. Endo dermal sinus (yolk sac) tumour
3. Chorio carcinoma
4. Embryonal cell carcinoma
Serous cystadenoma arises from surface epithelium,
Sex - cord stromal tumours of ovary
1. Granulosa cell tumour
2. Theca cell tumour
3. Arrhenoblastoma
4. Gynandroblastoma
5. Hilus cell tumour
Most common connective tissue tumour of ovary is ovarian
fibroma.
(Q.34) Which of the following type of anesthesia is frequently associated with fetalbradycardia:
(a) Paracervical block
(b) Pudendal block
(c) Spinal block
(d) Epidural block
Your Response :
Correct Answer : a
Exp: Pudendal block is perhaps the most common form of anesthesia used for vag
delivery. It provides adequate pain relief for episiotomy, spontaneous deliver
delivery, or vacuum extraction. The success of a pudendal block depends on
understanding of the anatomy of the pudendal nerve and its surroundings.
Complications (vaginal hematomas, retropsoas, or pelvic abscesses) are quite
Paracervical block was a popular form of anesthesia for the first stage of labo
was implicated in several fetal deaths. It has been shown that paracervical bl
associated with fetal bradycardia in 25 to 35% of cases, probably the respons
uptake of the drug from the highly vascular paracervical space with a resultan
of uteroplacental blood flow. Death in some cases was related to direct injec
local anesthetic into the fetus.

(Q.35) Which of the following is the most common site of tuballigation:


(a) Ampulla
(b) Isthmus
(c) Interstitial
(d) Fimbria
Your Response :
Correct Answer : b
Exp: The most popular technique of tubal ligation is the Pomeroy operation in this
made by holding the tube by an all is forceps in such a way that the major pa
loop consists of isthmus and part of ampullary part of tube i.e. at the junction
proximal and middle third.
The total length of tube is 10 cm junction of proximal and middle third is app
Length of various parts of fallopian tube:
Intramural = 1.25 cm
Ampulla = 5 cm
Isthmus = 2.5 cm
Infundibulum = 1.25 cm
Other methods of sterilization:
Pomeroy method
Madlener method
Irving method
Aldridge method
Corneal resection
Uchida method
Fimbriectomy
(Q.36) Carcinoma endometrium with superficial inguinal lymph node involvement is of:
(a) Stage IV
(b) Stage III
(c) Stage II
(d) Stage I
Your Response :
Correct Answer : a
Exp: The International Federation of Gynecology and Obstetrics(FIGO) staging syst
carcinoma of corpus uteri is as follows:
Stage IA-Tumor limited to endometrium <8 cm)
Stage IB-Invasion to less than one half the myometrium (>8 cm)
Stage IC-Invasion to more than one half the myometrium
Stage IIA-Endocervical glandular involvement only
Stage lIB-Cervical stromal invasion
Stage IlIA-Tumor invades serosa and/or adnexa and/or positive peritoneal cy
Stage IIIB-Vaginal metastasis
Stage IUC-Metastases to pelvic and/or para-aortic lymph nodes
Stage IVA-Tumor invasion of bladder and/or bowel mucosa
Stage IVB-Distant metastases including intra-abdominal and/or inguinal ly
Cases of carcinoma of the corpus should be classified (or graded) accordin
degree of histologic differentiation. The histopathology and degree of differe
as follows:
Class G1-Non squamous or non morular solid growth pattern of 5% or less
Class G2-Non squamous or non morular solid growth pattern of 6-50%
Class G3-Non squamous or non morular solid growth pattern of more than

(Q.37) Which of the following statements concerning appendicitis in pregnancy is true?


(a) Diagnosis is similar to that in the non pregnant patient
(b) The maternal death rate is highest in the first trimester
(c) Surgical treatment should be delayed until the diagnosis is firmly establis
(d) The incidence is unchanged by pregnancy
Your Response :
Correct Answer : d
Exp: The incidence of appendicitis in pregnancy is 1 in 2000, the same as that in th
nonpregnant population. The diagnosis is very difficult in pregnancy because
leukocytosis, nausea, and vomiting are common in pregnancy and the upwar
displacement of the appendix by the uterus may cause appendicitis to simula
cholecystitis, pyelonephritis, gastritis, or degenerating myomas. Surgery is ne
even if the diagnosis is not certain. Delays in surgery due to difficulty in diagn
appendix moves up are probably the cause of increasing maternal mortality w
increasing gestational age. Pre mature birth and abortion account for a rate o
close to 15%.
(Q.38) Intrauterine diagnosis of anencephaly can earliestbe done at what gestational age?
(a) 10 to 12 weeks
(b) 12 to 14 weeks
(c) 20 to 26 weeks
(d) 22 to 26 weeks
Your Response :
Correct Answer : b
Exp: (Ref. Sutton Radiology 7th ed. 1050)
The main sonographic feature of anencephaly is symmetric absence of the sk
and the cerebral hemispheres but relative preservation of brainstem and por
midbrain. Although on ultrasonographically the diagnosis can be suspected b
weeks of gestation, it is more reliable by around 15-16 weeks, when the ossif
normal calvarial bones is more obvious.

(Q.39) Not caused by DMPA:


(a) Thromboembolism
(b) Amenorrhea
(c) Irregular cycle
(d) Weight gain
Your Response :
Correct Answer : a
Exp: Depomedroxyprogesterone acetate (DMPA) is a suspension of microcrystals
synthetic progestin which is injected intramuscularly. Pharmacological active
achieved within 24 hours after injection, and serum concentrations of 1 ng/m
maintained for 3 months. During the fifth or sixth month after injection, the l
decrease to 0.2 ng/ mL, and they become undetectable by 7-9 months after i
acts by the inhibition of ovulation with the suppression of FSH and LH levels a
eliminates the LH surge. This results in a relative hypoestrogenic state. Single
150 mg will suppress ovulation in most women for as long as 14 weeks. The
contraceptive regimen consists of 1 dose every 3 months. It is an extremely e
contraceptive option. Neither varying weight nor use of concurrent medicatio
been noted to alter efficacy. Within the first year of use, the failure rate is 0.3
not produce the serious adverse effects of estrogen, such as thromboemboli
Dysmenorrhea is decreased. Risk of endometrial and ovarian cancer is decrea
Disadvantages are disruption of the menstrual cycle to eventual amenorrhea
50% of women within the first year. Persistent irregular bleeding can be trea
administering the subsequent dose earlier or by adding a low-dose estrogen
Because DMPA persists in the body for several months in women who have u
long-term basis, it can delay the return to fertility. Other adverse effects, suc
gain, depression, and menstrual irregularities, may continue for as long as on
the last injection.
(Q.40) Chorionic villus biopsy is done in all of the following EXCEPT
(a) Neural tube defects
(b) Sickle cell disease
(c) Myotonic dystrophy
(d) Down syndrome
Your Response :
Correct Answer : a
Exp: Indications of Chorionic villus biopsy :
Karyotyping is the most common indicationDown syndrome
Hemoglobinopathies Sickle cell disease, Thalassemia, etc.
Biochemical studiesGauchers disease, Niemann-Pick disease, Tay-Sachs d
Single gene defectsMyotonic dystrophy, Cystic fibrosis, Huntingtons diseas
Phenylketonuria, etc.
Neural tube defects are diagnosed by ultrasound and estimation of alpha-fet
maternal serum/ amniotic fluid.
Chorionic villous biopsy can be done after 10th week (earlier than amniocent
Earlier results reduce maternal stress by going for a possible therapeutic abo
indicated) at an early stage.
Initially a trans-cervical route is acceptable to take the sample.
A trans-abdominal route is taken if the pregnancy exceeds 12th week.

(Q.41) Not a side effect of oral contraceptives:


(a) Dysmenorrhea
(b) Mastalgia
(c) Chloasma
(d) Breakthrough bleeding
Your Response :
Correct Answer : a
Exp: Adverse effects of oral contraceptive pills include nausea, breast pain and ten
weight gain, breakthrough bleeding, amenorrhea, headaches, depression, an
decreased libido. OCs do not provide protection from STDs. A few months of
normal ovulatory cycles after discontinuation of OCs may occur. Women taki
contraceptive pills may also develop chloasma, which is augmented by expos
sun.
(Q.42) LARC (Long acting reversible contraceptives)include?
(a) Implants
(b) Injections
(c) IUCD
(d) All of the above
Your Response :
Correct Answer : d
Exp: Long-acting reversible contraception (LARC) are methods of birth control tha
effective contraception for an extended period of time without requiring use
They include injections, intrauterine devices (IUDs) and subdermal implants.
most effective reversible methods of contraception because they do not dep
patient compliance.In addition to being long-lasting, convenient, they are ve
effective.

(Q.43) Ideal time for post partum ligation of fallopian tube in a patient of heart disease is:
(a) Immediately after delivery
(b) One day after delivery
(c) One week after delivery
(d) Six weeks after delivery
Your Response :
Correct Answer : c
Exp: The operation should be done under local anaesthesia by minilap technique
IMPORTANT: Oral contraceptive pill is contraindicated in heart disease as it m
precipitate thromboembolic phenomenon. lUCDs are contraindicated due to
infection. Barrier contraceptive is the best if the family is not complete.
(Q.44) A pelvis characterized by an anteroposterior diameter of the inlet greater than the transverse diam
classified as
(a) Gynecoid
(b) Android
(c) Anthropoid
(d) Platypelloid
Your Response :
Correct Answer : c
Exp: By tradition, pelvis are classified as belonging to one of four major groups. Th
pelvis is the classic female pelvis with a posterior sagittal diameter of the inle
slightly shorter than the anterior sagittal diameter. In the android pelvis, the
sagittal diameter at the inlet is much shorter than the anterior sagittal diame
the use of the posterior space by the fetal head. In the anthropoid pelvis, the
anteroposterior (AP) diameter of the inlet is greater than the transverse diam
resulting in an oval with large sacrosciatic notches and convergent side walls.
spines are likely to be prominent. The platypelloid pelvis is flattened with a sh
wide transverse diameter. Wide sacrosciatic notches are common. The pelve
women do not fall into a pure type and are blends of one or more of the abo

(Q.45) Antihormone used in infertility is:


(a) Tamoxifen
(b) Clomifene
(c) Danazol
(d) Finasteride
Your Response :
Correct Answer : a
Exp: Clomifene is a non steroidal compound related to diethylstilbestrol indicated
Anovulatory infertility
Polycystic ovarian disease associated with infertility
In invitro fertilization: GIFT and ART
25 mg orally for 25 days each month for 3 to 6 month to stimulate spermatog
Clomifene citrate stimulates hypothalamic gonadotropin secretion by blockin
negative feedback of estrogen by ovaries. Gonadotropin stimulates pituitary
secretion which induces ovulation.
Danazol is an isoxazole derivative of 17--ethinyl testosterone used for:
Endometriosis
Cyclical mastalgia
Gynecomastia
Fibrocystic disease of breast
Improves libido
Shrinks fibroid
Tamoxifen is a nonsteroidal antiestrogen drug used in palliative treatment of
breast cancer in postmenopausal women.
Finasteride is a competitive inhibitor of enzyme Sa reductase.
(Q.46) What is the rate of cervical dilatation is first stage of labour in multigravidae?
(a) 0.5 cm/hour
(b) 1 cm/hour
(c) 1.5 cm/hour
(d) 2 cm/hour.
Your Response :
Correct Answer : c
Exp: (Ref. Dutta Obst. 5th Ed. - 136)
Cervical dilatation related with dilatation of external OS.
Taking up of cervix is determined by obliteration of projection of cervix into v
The anterior lip of cervix is the last to be effected.
Cervical dilatation is expressed either in terms of fractions of full dilatation 1/
3/4th OR in terms of fingers 2,2, 3 or fully dilated; OR better in terms of cm
when fully dilated). One finger equals to 1.6 cm and taking up in terms of % f
In primi, latent phase is long (8hrs). Dilatation of cervix at rate of 1.2 cm/hou
and 1.5 cm in multi beyond 3cm dilatation is satisfactory.
Partogram (Freidmans Curve) is composite graphical record of so dilatation
of head against duration of labour in hrs.

(Q.47) During the delivery, it is necessary to do an episiotomy. The tear extends through sphincter of the
the rectal mucosa is intact. How would you classify this type of episiotomy?
(a) First- degree
(b) Second-degree
(c) Third-degree
(d) Fourth-degree
Your Response :
Correct Answer : c
Exp: A first-degree tear involves the vaginal mucosa or perineal skin, but not the u
tissue. In a second-degree episiotomy, the underlying subcutaneous tissue is
involved, but not the rectal sphincter or rectal mucosa. In a third-degree tear
sphincter is affected. A fourth-degree episiotomy involves a tear that extend
rectal mucosa.
(Q.48) Episiotomy is best done:
(a) Medially
(b) Laterally
(c) Mediolaterally
(d) J shaped
Your
Response :
Correct
c
Answer :
Exp: (Ref. Textbook of Obstetrics D.C. Dutta 6th Ed.
606)
Types of episiotomy:
Mediolateral (best and routinely done)
Median
Lateral
J shaped
Only mediolateral or median episiotomy is done
commonly.

(Q.49) In a patient who complains of urinary incontinence, cystometrogram is performed to:


(a) Determine urethral length
(b) Rule out an unstable trigone
(c) Diagnose stress urinary incontinence
(d) Determine if a patient has normal bladder sensation
Your Response :
Correct Answer : d
Exp: As a catheter is introduced for performing a cystometrogram, measurement
urine is obtained. During the cystometrogram, a normal first sensation is of f
at 100 mL. Urge is felt at approximately 350 mL, with maximum capacity at 4
primary reason to perform a cystometrogram is to rule out uninhibited detru
contractions. The cystometrogram is a urodynamic test, and it cannot determ
whether ureterovesical reflux exists. The degree of reflux can be evaluated w
voiding cystogram, a radio- logic test.
(Q.50) Which of the following is not included in Active management of III Stage of Labor?
(a) Uterotonic within 1 minute of delivery
(b) Immediate clamping, cutting and ligation of cord
(c) General Massage of the uterus
(d) Controlled cord traction
Your Response :
Correct Answer : b
Exp: Clamping and ligature of cord is done in the second stage of labor and not th
Active management III stage of labor involves active assistance in delivery of
as to reduce the chances of post partum hemorrhage.
It includes:
Administration of uterotonics like Oxytocin Controlled cord traction
General massage of the uterus
During active management III stage we have to rule out any incidence of mul
pregnancies and then administer intramuscular oxytocin within one minute o
delivery.
To deliver the placenta, gentle traction on the umbilical cord is put, while the
uterus is being held back.
Uterine massage is done to make the uterus go into contraction and facilitate
of any retained clots.

(Q.51) Which of the following abnormalities of labor is associated with a significantly increased incidence
neonatal morbidity?
(a) Prolonged latent phase
(b) Protracted descent
(c) Secondary arrest of dilation
(d) Protracted active-phase dilation
Your Response :
Correct Answer : c
Exp: Three significant advances in the treatment of uterine dysfunction have redu
of perinatal morbidity and mortality: (1) the avoidance of undue prolongation
(2) the use of intravenous oxytocin in the treatment of some patterns of uter
dysfunction; and (3) the liberal use of cesarean section (rather than midforce
to effect delivery when oxytocin fails. Prolonged latent phase is not associate
increased risk of perinatal morbidity (PNM) or low Apgar scores and should b
therapeutic rest. Pro traction disorders have a higher rate of PNM and low Ap
but not if spontaneous labor follows the abnormality Arrest disorders are ass
significantly higher rates of PNM following either spontaneous or instrument
delivery
(Q.52) Emergency contraception is contraception provided to:
(a) Contraceptive failure
(b) Couple not wanting to use other contraceptives
(c) Contraception provided to couple living in camps provided by governmen
(d) Unprotected intercourse
Your Response :
Correct Answer : d
Exp: Emergency postcoital contraception is defined as the use of a drug or device
pregnancy after unprotected sexual intercourse. Candidates for emergency
contraception include reproductive-aged women who have had unprotected
intercourse within 72 hours of presentation independent of the menstrual cy

(Q.53) Secondary amenorrhoea after abortion due to intrauterine adhesions is seen in:
(a) Uterine inertia
(b) Imperforate hymen
(c) Bicomuate uterus
(d) Asherman's syndrome
Your Response :
Correct Answer : d
Exp: Causes of Asherman 's syndrome are
i. Vigorous curettage ii. Abortion and MTP
iii. Puerperal infection iv. Endometrial tuberculosis
Diagnostic investigations for Asherman's syndrome - hysterosalpingography
and hysteroscopy.
Treatment of choice - Breaking of intrauterine adhesions by uterine sound
under anaesthesia.
(Q.54) False regarding dermoid cyst of ovary is:
(a) Bilateral in 35% of cases
(b) > 10 cm
(c) Lined by epithelial cells
(d) Has sebaceous material
Your Response :
Correct Answer : a
Exp: Of all cystic tumors of ovary 5-1 0% are dermoids. Dermoid cysts are usually u
swelling with smooth surfaces, seldom attaining more than 15 cm in diamete
contain sebaceous material and hair and the wall is lined in part by squamou
which contains hair follicles and sebaceous glands. Teeth, bone, cartilage, thy
and bronchial mucous membrane are often found in the wall. Dermoid cysts
arise in association with pseudomucinous cystadenomas to form a combined
They are not infrequently bilateral, 12%. Dermoid cysts are innocent ovarian
epidermoid carcinoma occurs in 1.7% of all dermoids and sarcomatous chang
also been described.

(Q.55) Shape of cervical canal predicting preterm labour is:


(a) U shaped
(b) V shaped
(c) Y shaped
(d) O shaped
Your Response :
Correct Answer : c
Exp: Diagnosis of preterm labour includes:
1. Regular uterine contractions with or without pain with a frequency of at le
every 10 minutes
2. Dilatation> 2 cm and effacement> 80% of cervix
3. Length of cervix on transvaginal ultrasound < 2.5 cm and funneling of inter
(funnel shape = Y shape)
4. Pelvic pressure, backache, vaginal discharge or bleed.
(Q.56) Macrosomia is not seen in:
(a) Postmaturity
(b) Hydrocephalus
(c) Obesity
(d) Diabetes mellitus
Your Response :
Correct Answer : b
Exp: Risk Factors for Macrosomia
. Maternal diabetes mellitus or glucose . Multiparity
intolerance
. Prior history of macrosomic infant . Post-dates gestation
. Maternal obesity or excessive weight . Male fetus
gain
. Parental stature . Labor dystocia

(Q.57) Vaginal cytology for detecting hormonal changes is taken from:


(a) Fornices
(b) Lateral wall
(c) Posterior wall
(d) Anterior wall
Your Response :
Correct Answer : b
Exp: For hormonal cytology, the lateral wall of the upper third of the vagina is ligh
This part of the vagina is most sensitive to hormonal influence. Smear is stain
Shorr's stain.
(Q.58) All the following are given in postpartum hemorrhage except
(a) Methylergonovine
(b) Carboprost
(c) Oxytocin
(d) Misoprostol
Your Response :
Correct Answer : d
Exp: Misoprostol is indicated for the prevention of gastric ulcer associated with th
nonsteroidal anti-inflammatory drugs (NSAIDs), including aspirin, in patients
of complications from gastric ulcer, such as the elderly, and in patients with c
disease or patients at high-risk of developing gastric ulceration, such as those

(Q.59) Point of distinction between partialmole to complete mole is:

(a) Partial mole show trophoblastic proliferation with absent villi

(b) Typical of partial mole is cellular atypia

(c) Partial mole is more prone to tumor malignancy

(d) Partial mole is triploid

Your Response :
Correct Answer : d

Exp:

Complete mole Partial mole


A complete mole contains no fetal Fetal tissue is often present in a partial
tissue mole
90% are 46, XX, and 10% are 46, XV. 69, XXX or 69, XXY
All chromosomes are of paternal
origin
An enucleate egg is fertilized by a This results from fertilization of a
haploid sperm haploid ovum and duplication
(which then duplicates its of the paternal haploid chromosomes or
chromosomes), or the from dispermy.
Egg is fertilized by 2 sperm
Chorionic villi have grapelike Swelling of the chorionic villi
(hydatidiform) swelling
There is trophoblastic hyperplasia There is hyperplastic trophoblastictissue
After a complete mole develops, No cases of choriocarcinoma have
uterine invasion beenreported after a partial mole,
occurs in 15% of patients, and although 4% of patients with partial
metastasis occurs moles develop persistent non-
in 4%. metastatictrophoblastic disease
(Q.60) Mullerian duct anomaly may include the absence of any of the following EXCEPT
(a) Uterus
(b) Vagina
(c) Ovary
(d) Uterine tube
Your Response :
Correct Answer : c
Exp: Mullerian duct is a mesodermal derivative and gives rise to uterus and uterin
also forms mesodermal part of the vagina, when it meets with the sino-vagin
Sino-vaginal bulbs are induced by the mullerian duct and they form the lowe
(endodermal) part of vagina.
Now if mullerian ducts are absent, there will be no uterus and uterine tubes.
also not form because it is the mullerian duct which induces the formation of
Ovaries will be present, since they arise from a separate source-the genital ri
Note
1. A patient with Mullerian agenesis will have amenorrhea due to the absenc
And this amenorrhea cannot be corrected.
2. The lady can become a mother by using a surrogate to whom she contribu
for in vitro fertilization.

(Q.61) Amnion is seen on?


(a) Fetal surface
(b) Maternal surface
(c) Decidua
(d) Placenta
Your Response :
Correct Answer : a
Exp: Amnion is the innermost fetal membrane. It is a thin, transparent sac that ho
suspended in the liquor amnii. The amnion grows rapidly at the expense of th
extraembryonic coelom, and by the end of the 3rd month it fuses with the ch
forming the amnio chorionic sac.
(Q.62) All of the following are known risk factors for development of endometrial carcinoma except:
(a) Obesity
(b) Family history
(c) Use of hormone replacement therapy
(d) Early menopause
Your Response :
Correct Answer : d
Exp: Etiology of endometrial carcinoma
Estrogen (most common): Persistent stimulation of endometrium with unopp
estrogen;
Age: 75% of patients are postmenopausal women;
Nulliparity;
Late menopause;
Corpus cancer syndrome (obesity, hypertension and diabetes);
Obesity;
Polycystic ovarian syndrome and functioning ovarian tumors (granulosa cell t
to unopposed estrogen stimulation
Long term use of tamoxifen;
Family history or person history of cancers of colon, ovary or breast;
Fibroid; and
Endometrial hyperplasia

(Q.63) Commonest chromosomal anomaly leading to spontaneous abortions is:


(a) Trisomy 16
(b) Trisomy 21
(c) Tetraploidy
(d) Turner's syndrome
Your Response :
Correct Answer : a
Exp: Cytogenetic study of the abortus can reveal chromosomal abnormality - mo
is trisomy
The most common type of parental chromosomal abnormality is balancer
translocation,
(Q.64) Micronized progesterone can be given by which of the following routes:
(a) Oral and vaginal routes
(b) I/V
(c) Vaginal route only
(d) Oral only
Your Response :
Correct Answer : a
Exp: Natural progesterone Progestogen content Route of
administration
Progesterone 25mg Intramuscular
Micronized progesterone 100 mg Oral/vaginal

(Q.65) IUCD lasting for 10 years is:


(a) Progestasert
(b) CuT - 380A
(c) CuT 220
(d) Nova T
Your Response :
Correct Answer : b
Exp: Cu T 380A is a second generation IUD. It carries 80 mm surface area of coppe
frame contains barium sulphate and is radio opaque. Replacement is done ev
years.
The time period for replacement for various IUDs are:
Copper T 200 3 years
Copper T 200B 4 years
Copper T 380 A (Paraguard) 10 years
Multiload Cu 250 3 years
Multiload 375 5 years
LNG- IUS 5 years
Progestasert 1 years
Nova T 5 years
Levonova 5 years
Mirena 5 years
(Q.66) In a case of grade II endometrial carcinoma with myometrial involvement up to 1 0 cm in the uter
staging is
(a) IA
(b) IB
(c) IIA
(d) lIB
Your Response :
Correct Answer : b
Exp: Carcinoma endometrium staging - FIGO
Stage 0 - carcinoma in situ
Stage I - carcinoma confined to endometrium, myometrium intact
Stage IA - Length of uterine cavity 8 em, or less
Stage IB - Length uterine cavity more than 8 cm
Stage II - carcinoma has involved corpus and cervix

(Q.67) A patient treated for infertility presents with 8 weeks' amenorrhoea, retention of urine and abdom
Diagnosis is
(a) Impacted cervical fibroid
(b) Retroverted gravid fibroid
(c) Pelvic hematocele
(d) Uterine hematoma
Your Response :
Correct Answer : c
Exp: Pelvic hematocele - may follow an instrumental delivery, a paracervical or pu
block anaesthesia, vigorous ironing of the lower vagina and perineum during
even rarely after a normal delivery.
Retroverted gravid uterus - causes urinary retention at 12 weeks of gestation
The patients treated with infertility most likely the incidence of ectopic pregn
much higher presents with amenorrhoea (75%), Abdominal pain (95%), vagin
retention of urine
Retention of urine - In a subacute variety of ectopic pregnancy, the blood col
pouch of Douglas to form a pelvic haemotocele, this haematocele forms an ir
mass of differing consistency due to a mixture of clot and blood, and bulges f
displacing the cervix against the bladder and leading to retention of urine
Cervical fibroid in the cervix or lower uterine segment may obstruct labour an
confused with the fetal head.
(Q.68) Puerperium is the period:
(a) 6 wks following delivery
(b) 3 wks following delivery
(c) 1 wk following delivery
(d) None of the above
Your Response :
Correct Answer : a
Exp: (Ref. Textbook of Obstetrics D.C. Dutta 6th Ed. 153)
Purperium begins as soon as the placenta is expelled and lasts for approxima
when the uterus becomes regressed almost to the nonpregnant size.

(Q.69) A 30-year-old female has a history of two previous anencephalic pregnancies. The chances of the n
pregnancy being anencephalic are
(a) 10%
(b) 40%
(c) 20%
(d) 25%
Your Response :
Correct Answer : a
Exp: Anencephali - most severe defect in which the forebrain meninges, vault of t
scalp all fail to form
Recurrence rate
7.8% with high spina bifida
0.7% with low spina bifida
2.2% with anencephaly and no increased risk of r craniorachischisis, encepha
multiple defects
Neural tube defects are classical examples of multi factorial inheritance
The empirical recurrence risk of first degree relatives is usually quoted as 2-3
factorial inheritance
(Q.70) Best reversal after tubectomy is in which of the following types?
(a) Isthemo-isthemic
(b) Isthemo-ampullary
(c) Ampullo-ampullary
(d) Cornual implantation
Your Response :
Correct Answer : a
Exp: Tubal ligation reversal utilizes the techniques of microsurgery to open and re
fallopian tube segments that are remaining after a tubal sterilization procedu
Microsurgery minimizes tissue damage and bleeding during surgery.
Essential elements of microsurgical technique include gentle tissue handling,
the operating field, keeping body tissues in their normal state with warmed i
fluids, and using the smallest sutures with the thinnest needles capable of ho
tubal ends together to promote proper healing of the rejoined tubal segmen
An Isthemo-isthemic anastomosis has the best outcome with live birth rates
provided that the reconstructed tube is longer than 4cm and the ampullary p
more than 1cm.

(Q.71) In IUCD, the organism causing infection is:


(a) Cryptococcus
(b) Herpes
(c) Chlamydia
(d) Actinomycosis
Your Response :
Correct Answer : d
Exp: Actinomycotic involvement of the pelvis occurs most commonly in associatio
IUD. The disease rarely develops unless the IUD has been in place for at least
it can present months after the removal of the device.
(Q.72) Lochia serosapersists upto:
(a) 1-3days
(b) 5-10days
(c) 10-15days
(d) Upto 21days
Your Response :
Correct Answer : b
Exp: (Ref. Textbook of Obstetrics D.C. Dutta 6th Ed. 155)
Lochia is the vaginal discharge for the first fortnight during puerperium. It ha
peculiar offensive fishy smell. Its reaction is alkaline tending to become acid t
end. It is named as:
Lochia Rubra (red) 1 to 4days
Lochia serosal 5-9days
Lochia alba (Pale white) 10-15days

(Q.73) Most common complication of laparoscopic hysterectomy is:


(a) Urinary tract injury
(b) Bowel injury
(c) Vaginal cuff abscess
(d) Uncontrolled haemorrhage
Your Response :
Correct Answer : b
Exp: Most common complication of laparoscopic hysterectomy is bowel injury.
Laparoscope assisted vaginal hysterectomy(LAVH) converts an abdominal hys
into vaginal hysterectomy and thus avoids an abdominal scar and decreases t
morbidity
(Q.74) Cause of repeated 2nd trimester foetal loss:
(a) Chromosomal anomaly
(b) Intrauterine infection
(c) Abnormality of cervix and uterus
(d) Hormonal imbalance
Your Response :
Correct Answer : c
Exp: (Ref. Textbook of Obstetrics D.C. Dutta 6th Ed. 172)
Common known causes of abortion:
First trimester: defective germ plasm, hormonal deficiency, trauma, acute in
Mid trimester: Cervical incompetence, uterine malformations, uterine fibroid
implantation of placenta, twins and hydramnios

(Q.75) Technique used for aspiration of sperm directly from testes for in vitro fertilization is:
(a) TESA
(b) MESA
(c) GIFT
(d) IVF
Your Response :
Correct Answer : b
Exp: Sperm aspiration technique involves the use of minor surgical procedures to
sperms from organs within the genital tract. In men in whom transport of spe
possible.
This may involve aspiration of sperms from the vas deferens, epididymis, or t
In vitro fertilization is required to achieve pregnancy with the majority of the
extraction procedures.
Procedure Source IVF
Vasal aspiration Vas deferens May be
Epididymal aspiration Epididymis Yes
Testicular aspiration Testicle Yes
Epididymal aspiration: is of two types:
- MESA: Microsurgical Epididymal Sperm
Aspiration
- PESA: Percutaneous Epididymal Sperm Aspiration
This is performed in situations in which vas is either not present or is scarred
surgery, trauma, or infection.
Testicular sperm extraction (TESE, TESA)
The newest of the three aspiration techniques, testicular sperm retrieval is a
breakthrough as it demonstrates that sperms donot have to mature and pass
epididymis in order to fertilize an egg. Because of their immaturity, however,
sperms need to be injected directly into the egg with ICSI (intra cytoplasmic s
injection) for fertilization to occur. Testicular sperm extraction is indicated fo
whom there is blockade in the epididymis very close to where it attaches to t
a blockade with in ducts that conduct sperm out of the testis.
IVF( in vitro fertilization)
In this induction of ovulation is done followed by aspiration of mature oocyte
ultrasonic guidance. 50000 selected sperms are used for insemination and at
stage, three emryo transfer into the uterine cavity 1 cm below the fundus is p
GIFT(gamete intra fallopian transfer technique)
It involves aspiration of oocytes following ovulation induction. These are mix
50,000 prepared sperms and transferred to ampullary portions of both the fa
tubes 4 cm from the fimbrial end. Thus MESA and TESA are both techniques o
aspiration but since the question specifically asks about aspiration directly fro
testes, TESA is the most appropriate ans here.
Of all the choices mentioned, MESA is a choice which can be answered Gene
ba choice between MESA and PESA is given, then PESA is the answer as PESA
simpler.

(Q.76) The following is used only extra amniotically:


(a) Ethacrydine lactate (0.1%)
(b) 20% mannitol
(c) 50% Urea
(d) Misoprostol
Your Response :
Correct Answer : a
Exp: Ethacrydine lactate (0.1%) is used only extra amniotically.
Ethacridine lactate (ethacridine monolactate monohydrate, acrinol, trade na
is an aromatic organic compound based on acridine. Its primary use is as an a
solutions of 0.1%. Ethacridine is also used as an agent for second trimester ab
Extra-amniotic ethacridine lactate plus intramuscular prostaglandin has beco
popular method for terminating second trimester pregnancies.
(Q.77) The treatment of the lutein cyst in a hydatiform mole is
(a) Ovarian cystectomy
(b) Ovariectomy
(c) Suction evacuation
(d) Ovariotomy
Your Response :
Correct Answer : c
Exp: Theca-lutein cyst - The incidence of obvious cysts in association with a mole i
to be 25-60% these cysts are thought to result form overstimulation of lutein
by large amounts of chorionic gonadotrophin elements secreted by prolifera
trophoblasts
Cysts are not limited to case of H. mole and are associated with placental hyp
with fetal hydrops or multifetal pregnancy
Because of the cysts regress after delivery oophorectomy should not be perfo
unless the ovary is extensively infarcted.
Suction evacuation is best line of treatment for H. mole.
(Q.78) Retention of urine in a pregnant woman with a retroverted uterus is seen at
(a) 8-10 weeks
(b) 12-16 weeks
(c) 20-24 weeks
(d) 28-32 weeks
Your Response :
Correct Answer : b
Exp: Retroverted gravid uterus - As a result of retension the bladder gets distende
becomes an abdominal organ reaching even up to umbilicus.
As pressure from the full bladder increases, small amounts of urine are passe
involuntarily, but the bladder never empties entirely - paradoxical incontinen
After incarceration (12-16 weeks) the initial symptoms is frequency of urinati
by difficulty in micturition which ultimately culminates into retension of urine
In majority of patients spontaneous rectification occurs by 12 weeks.

(Q.79) Precocious puberty is seen in all the following EXCEPT


(a) Granulosa cell tumour
(b) Head injury
(c) Corticosteroid intake
(d) Hyperthyroidism
Your Response :
Correct Answer : d
Exp: Causes of precocious puberty - Tumors in the brain (Hamartoma), post inflam
lesions such as meningitis, encephalitis, hypothyroidism (not hyperthyroidism
secreting ovarian tumors like granulosa and theca cell tumors and adrenal m
Mc cune - Al bright syndrome, Russel, Silver syndrome, Estrogen containing m
(Q.80) During pregnancy there is reduced risk of which of the following condition?
(a) Relapse of multiple sclerosis
(b) Bells palsy
(c) Meningoma
(d) Chorea
Your Response :
Correct Answer : a
Exp: The onset of a new movement disorder during pregnancy suggests chorea gr
variant of Sydenham's chorea associated with rheumatic fever and streptoco
infection; the chorea may recur with subsequent pregnancies. Patients with p
multiple sclerosis experience a gradual decrease in the risk of relapses as pre
progresses and, conversely, an increase in attack risk during the postpartum
interferons should not be administered to pregnant MS patients, but modera
relapses can be safely treated with pulse glucocorticoid therapy. Finally, certa
particularly pituitary adenoma and meningioma, may manifest during pregna
because of accelerated growth, possibly driven by hormonal factors.
Peripheral nerve disorders associated with pregnancy include Bell's palsy (idi
facial paralysis), which is approximately threefold more likely to occur during
trimester and immediate postpartum period than in the general population.

(Q.81) The commonest prostaglandin used for the termination of 2 nd trimester


pregnancy:

(a) PGE1

(b) PGI2

(c) PGA2

(d) 15 methyl PGF2

Your Response :
Correct Answer : d

Exp:
(Ref. Textbook of Obstetrics D.C. Dutta 6th Ed. 186)
Methods of termination of pregnancy:
Second Trimester (13-20 weeks):
1. Intrauterine instillation of hypertonic solutions
1. Intra amniotic 20% saline, 40% urea mannitol
2. Extra amniotic Ethacrydine lactate
2. Prostaglandins F2a and E2
3. Oxytocin infusion
4. Hysterotomy
(Q.82) A case of gestational trophoblastic neoplasia is detected to have lung metastasis. She should be st
as:
(a) Stage I
(b) Stage II
(c) Stage III
(d) Stage IV
Your Response :
Correct Answer : c
Exp: Anatomic staging for gestational trophoblastic tumors (GTT)-a FIGO
guideline
Stage I The lesion is confined to the uterus
Stage II The lesion spreads outside the uterus but is confined to the genital
organs
Stage III The lesion metastasizes to the lungs
Stage IV The lesion metastasizes to sites such as brain, liver and GI tract

(Q.83) Criteria for fetal growth are allEXCEPT:


(a) Height of the uterus
(b) Maternal weight gain
(c) Ultras onographic measurement of biparietal diameter
(d) Ultrasonographic measurement of fetal abdominal circumference
Your Response :
Correct Answer : b
Exp: Maternal weight gain is not a sensitive criteria for fetal growth because contr
fetal weight to maternal weight gain during pregnancy is less significant than
water retention and fat accumulation during pregnancy.
The total weight gain during the course of a singleton pregnancy is 11kg., out
fetus contributes only about 3kg.
(Q.84) Which one of the following biochemical parameters is most sensitive to detect open spina bifida?
(a) Maternal serum alpha-fetoprotein
(b) Amniotic fluid alpha-fetoprotein
(c) Amniotic fluid acetylcholinesterase
(d) Amniotic fluid glucohexaminase
Your Response :
Correct Answer : a
Exp: It has been found that a high concentration of alpha-fetoprotein in the amnio
strongly suggestive of a severe neural tube defect in the fetus. Furthermore,
of alpha-fetoprotein in mother's serum gives an indication that the AFP in the
fluid is likely to be raised.

(Q.85) Which of the following is the most common cause of persistent trophoblastic disease after H. Mol
evacuation?
(a) Choriocarcinoma
(b) Invasive mole
(c) Placental site trophoblastic tumor
(d) Residual Mole
Your Response :
Correct Answer : d
Exp: Upto 20% of women with H. Mole show persistence of tumor in uterus follow
evacuation.
15% persist as persistant or residual mole.
5% develop choriocarcinoma.
Trophoblastic tumors diagnosed for upto 6 months following an abortion or m
often
An invasive mole, but the one diagnosed after 6 months is usually choriocarc
It is to be noted that trophoblastic tumor developing after a full term pregna
always a choriocarcinoma.
(Q.86) Misoprostol has been found to be effective in all of the following, except
(a) Missed abortion
(b) Induction of labor
(c) Menorrhagia
(d) Prevention of postpartum hemorrhage (PPH)
Your Response :
Correct Answer : c
Exp: Studies in humans have shown that Misoprostol causes an increase in the fre
intensity of uterine contractions. Misoprostol administration has also been a
with a higher incidence of uterine bleeding and expulsion of uterine contents

(Q.87) Gonococcal infection spreads by


(a) Ascending route
(b) Haematogenous route
(c) Lymphatics
(d) Involvement of adjacent structures
Your Response :
Correct Answer : d
Exp: Gonococcal infections spreading along the mucous membranes to affect the
endometrium and the fallopian tubes
In rare cases spreads by blood stream may occur causing endocarditis, arthri
(Q.88) Stress incontinence is best corrected by
(a) Colposuspension
(b) Hysterectomy
(c) Bladder neck repair
(d) Bladder exercise
Your Response :
Correct Answer : c
Exp: Stress incontinence - Involuntary loss of urine during stress (coughing, sneezi
physical train) is a common complaint of post menopausal women
Digital pressure applied to the paraurethral tissues is an anterior direction th
vagina will reestablish the urethrovesical angle and prevent stress incontinen
[Marshall's test]
Treatment - in patients with normal bladder function and low residual urine
with behavioral therapy and perineal exercise.
If unsuccessfully - give oxybutyric and ephedrine
Definitive management is surgical
Marshall - Marchetti - Krantz procedure
Stamey modification of the Pereyra procedure
*Currently - The most effective surgical approach is sling procedure with a pi
autologous or synthetic fascia attached to the rectus muscle or os pubis and
the urethra at the bladder neck
Newer approaches - includes the use of collagen injection into the periurethr
resulting in increased urethral outflow resistance. .
The menstrual cycle is reduced from the normal of 28 days to cycle of 2-3 we
remains constant at that frequency
Menorrhagia (Hypermenorrhea) excessive or prolonged bleeding at the norm
menstruation.

(Q.89) Forceps are applied in all the following EXCEPT


(a) After-coming head in breech presentation
(b) Face presentation
(c) Occipitoposterior presentation
(d) Brow presentation
Your Response :
Correct Answer : d
Exp: Fetal Maternal
. Fetal distress . Maternal distress/exhaustion
. Preterm baby - LBW (preventing . Prolonged IInd stage of labour
intracranial damage) . Medical disorders like cardiac, sev
. After coming head in breach TB PIH, eclampsia or debilitating il
. Cord prolapse . Failure of descent or internal rotat
. Post maturity hours in primigravida and 1 hour in
multipara.
Situations which predispose to arrest of progress include
Poor uterine contractions
Malpositions like right or left OPP
Deflexed head
Prominent spines or subtle disproportion in lower pelvic strait
Rigid perineum
Divarication of recti
Lax pelvic floor
Heavy sedation or analgesia
Brow presentation - Attempts to convert the brow manually to either a face
for delivery with forceps or the vacuum are not practiced any more, if the pa
been long in labour these manipulation should be tried as the risk of uterine
high
Caesarean section is the safest method of delivery
Forceps delivery is only reserved for mentoanterior.
Contraindication to use of forces
Absence of a proper indication.
Absence of full dilatation of cervix
Cephalopelvic disproportion cannot be ruled out
High station of fetal head
Uterine contraction cease.
(Q.90) Placenta with umbilical cord attached to its margin is called:
(a) Battledore placenta
(b) Circumvallate placenta
(c) Succenturiate placenta
(d) Velamentous placenta
Your Response :
Correct Answer : a
Exp: (Ref. Textbook of obstetrics D C Dutta 6th ed. 233)
In Battledore placenta, the umbilical cord is attached to the margin of placen
associated with low implantation of the placenta; there is chance of the cord
compression in vaginal delivery leading to foetal anoxia and even death.
Succenturiate placenta: An extra placenta separate from the main placenta
Velamentous cord insertion is an abnormal condition during pregnancy. Nor
umbilical cord inserts into the middle of the placenta as it develops. In velam
insertion, the umbilical cord inserts into the fetal membranes (choriamniotic
membranes), then travels within the membranes to the placenta (between th
and the chorion). The exposed vessels are not protected by Wharton's jelly a
are vulnerable to rupture. Rupture is especially likely if the vessels are near th
which case they may rupture in early labor, likely resulting in a stillbirth. Early
can reduce the need for emergency cesarean sections

(Q.91) The most common complication of IUCD is


(a) Ectopic pregnancy
(b) Bleeding
(c) Backache
(d) Cervical stenosis
Your Response :
Correct Answer : b
Exp: The commonest complaint of a woman fitted with an IUCD (inert or medicate
increased vaginal bleeding)
Complication of IUCD
Immediate Early Late
1. Difficulty in insertion 1. Expulsion 1. PID
2. Vasovagal attack 2. Perforation 2. Ectopic pregn
3. Uterine cramps 3. Spotting, menorrhagia 3. Perforation
4. Dysmenorrhea 4. Menorrhagia
5. Vaginal infection 5. Dysmenorrhe
6. Actinomycosis* 6. Pregnancy
(Q.92) The most common type of pelvis associated with occipita-posterior position is
(a) Gynecoid
(b) Platypelloid
(c) Anthropoid
(d) Android
Your Response :
Correct Answer : c
Exp: In more than 50%, the occipito-posterior position is associated with either
anthropoid or android pelvis
Deep transverse arrest is common in android pelvis.
Face to pubis delivery occurs in - anthropoid pelvis

(Q.93) Mediolateral episiotomy is preferred over midline episiotomy because:


(a) Blood loss is less
(b) Less chance of third degree extension
(c) Easier to repair
(d) None of the above
Your
Response :
Correct
b
Answer :
Exp: (93). Ans: b.
Exp. Less chance of third degree extension
Merits ofmidline emsiotomv over medic-lateral episiotomv
Bloodless is less.
Repair is easy.
Healing is superior.
Dyspareunia is rare.
Wound disruption is rare.
Post operative comfort is more.
Merits of media-lateral eptswtomv over midline episiotomv
Lesser risk of rectal third degree extension.
Suitable for manipulative delivery or in abnormal
presentation or position.
(Q.94) Chorionic villous sampling is useful in all the following EXCEPT

(a) Tay-Sachs disease

(b) Spina bifida

(c) Thalassanemia

(d) Down syndrome


Your Response :
Correct Answer : b

Exp:
Chorionic villous sampling (CVS) is generally performed at 10 to 13 weeks.
1. Second trimester amniocentesis - for genetic diagnosis is usually performed betw
20 weeks.
2. Early amniocentesis - this is performed between 11 and 14 weeks CVS fortropho
(derived embryologically from the same fertilized eggs as the fetus is usually done du
gestational weeks 11-13
The advantage of CVS is that the results are available early in pregnancy
Indications - Advanced maternal age (> 35 years), previous child with chromosomal a
abnormal parenteral karyotype, inversion, balanced or reciprocal
translocation, Robertsonian, foetal karyotyping for X-linked disorder, maternal anxiet
inborn error of metabolism.
Lipid storage disease
- Gaucher's disease, Fabry disease. Krabbe disease metachromaticleukodystrophy, Nie
syndrome mucolipidosis II sialidosis
Mucopolysaccharide disorder- Hunter, Hurler and Sanfilippo syndrome
Carbohydrate disorder - galactosemia, glyco eft storage (type n & IV) Aminoacidopath
Homocystinuria, cystinosis, tyrosinemia, maple syrup urine disease. Autosomal domin
disorders - adult polycystic kidney disease. myotonic dystrophy, Huntington's
chorea,osteogenesis imperfecta, retinoblastoma
Autosomal recessive disorders - a and thalassanemia, al antitrypsin deficiency,
CAH,phenylketonuria, cystic fibrosis sickle cell disease
Sex linked disorders - Duchenne's A & B, chronic granulomatous disease, Norrie's dise
retinitis pigmentosa
Relative contraindications
1. Vaginal bleeding or spotting
2. Extreme ante-or retroversion of the uterus
3. Patient body habitus precluding easy access to the uterus
4. Clear visualization of its content with ultrasound
5. Active infection.

(Q.95) Most common cause for postmenopausal bleeding in Indian woman


is,
(a) Ca cervix
(b) Ca endometrium
(c) DUB
(d) Ca ovary
Your
Response :
Correct
a
Answer :
Exp: Post menopausal bleeding:
As many as 1/3 rd of cases are due to malignancy
Genital malignancy
Ca cervix, endometrium, vagina, vulva, fallopian tube
Sarcoma of uterus
Granulosa cell tumout of ovary
Common causes of postment) pausal bleeding
Genital malignancy
DUB
Senile endometritis
Decubitus ulcer
Urethral caruncle
Commonest cause in India is Ca cervix.
(Q.96) All the following are TRUE about Krukenbergs tumour EXCEPT
(a) Large cystic spaces
(b) Maintains ovarian shape
(c) Bilateral
(d) Usually from stomach carcinoma
Your Response :
Correct Answer : a
Exp: Krukenberg! tumor - are almost invariably bilateral
There is no tendency to form adhesions with neighboring viscera and there is
infiltration through the capsule
The tumor retains the shape of the normal ovary and has a peculiar solid wax
consistence, although cystic spaces due to degeneration of the growth are co
Histologically the tumor has a cellular or myxomatous stroma amongst which
large signet ring cells
The tumors are secondary growths in the ovary and most often arise from a p
carcinoma of the stomach (70%), Large bowel (15%) and breast (6%)
The tumor almost certainly arise by retrograde lymphatic spread

(Q.97) Which one of the following is the best drug of choice for treatment of bacterial vaginosis during
pregnancy?
(a) Clindamycin
(b) Metronidazole
(c) Erythromycin
(d) Rovamycine
Your Response :
Correct Answer : b
Exp: Treatment of bacterial vaginosis:
Sensitive to metronidazole / clindamycin
Metronidazole use in the first trimester - no human evidence of terratogenesi
Topical therapy using vaginal gel is effective but may not reduce the risk of pr
delivery.
(Q.98) IUCD is absolutely contraindicated in all the following EXCEPT
(a) Undiagnosed vaginal bleeding
(b) Suspected pregnancy
(c) Congenital malformation of uterus
(d) PID
Your Response :
Correct Answer : c
Exp: Contraindication of IUCD
Absolute Relative
1. Suspected pregnancy 1. Anaemia
2. PID 2. Menorrhagia
3. Vaginal bleeding of 3. H/o PID since last pregnancy
undiagnosed
Aetiology 4. Pure lent cervical discharge
4. Cancer of the cervix, uterus or
adnexa and other pelvic 5. Distortions of the uterine cavity due t
tumours
5. Previous ectopic pregnancy congenital malformations, fibroids
6. Unmotivated person

(Q.99) Antenatal diagnosis can be made from all the following EXCEPT
(a) Foetal blood
(b) Maternal blood
(c) Amniotic fluid
(d) Decidua
Your Response :
Correct Answer : d
Exp: Foetal blood - for anemia, bleeding disorders, Rh disease Non-Immune hydro
fetalisMaternal blood - for alfa feto protein estimation for congenital disease
Amniotic fluid - assessing the fetal maturity, diagnosis of various malformatio
fetus and monitoring well being of the fetus in high risk, pregnancies.
Deciduae - the endometria lining of the uterus is called decidua during pregn
is shed after delivery, and no role for antenatal diagnosis.
(Q.100) Drug not used in the management of PPH?
(a) Mifepristone
(b) Misoprostol
(c) Oxytocin
(d) Ergotamine
Your Response :
Correct Answer : a
Exp: Drugs used in the management of Postpartum Hemorrhage
Oxytocin can be administered as a 5-U intravenous bolus, as 20 U in 1 L of N
intravenously run as fast as possible, or as 10 U intramyometrially with a spi
no immediate intravenous access is available.
The traditional second-line agent for uterine atony has been ergonovine (or
given as an initial dose of 100 or 125 mcg intravenously or intramyometriall
250 mcg intramuscularly.
The maximum total dose is 1.25 mg. Hypertension is a relative contraindicat
regions, the availability of ergot preparations has become problematic. Ever
should be made to secure supplies of this inexpensive and useful agent.
Many authorities now recommend the use of intramuscular carboprost as th
line agent when it is available. The recommended dose is 250 mcg intramus
intramyometrially, not to exceed 2 mg (8 doses). Asthma is a relative contra
Carboprost has been shown to be 80-90% effective in stopping PPH in cases
to oxytocin and ergonovine. Intramuscular administration of these agents is
recommended if the patient demonstrates evidence of shock because absor
be compromised.
Misoprostol may also become a valuable agent in the treatment of PPH. The
the drug and its heat stability (does not require refrigeration) makes it espec
appealing for use in the developing world. More trials are pending.

(Q.101) Carcinoma cervix extends to the lateral pelvic wall in which stage
(a) Stage I
(b) Stage II
(c) Stage III
(d) Stage IV
Your Response :
Correct Answer : c
Exp: Stage III B more precisely.
FIGO staging of cancer of cervix
Preinvasive carcinoma
Stage 0 carcinoma in situ
Invasive carcinoma
Stage 1
IA Carcinoma strictly extend to the cervix
Invasive cancer diagnosed only by microscopy.
All gross lesions, even with superficial invasions are stage m
IA1 Measured invasion of stroma no greater then 3 mm in depth and no wid
mm
IA2 Measured invasion of stroma greater than 3 nun in depth and no longer
in depth
and no wider than 7 mm
IB Clinical lesion confined to the cervix or preclinical lesions greater tha
IB1 Clinical lesion on greater than 4 cm
IB2 Clinical lesions greater than 4 cm
Stage II Carcinoma extends beyond the cervix but has not extended to the
the carcinoma involves of vagina but not as far as lower third
IIA No obvious parametrial involvement
IIB Obvious parametrial involvement
Stage III Carcinoma has extended either to the lower third of the vagina o
pelvic sidewall, all cases of hydronephrosis
IIIA Involvement of lower third of vagina, no extension to pelvic side wa
IIIB Extension on the pelvic wall and/or hydronephrosis or non function
Stage IV Carcinoma extended beyond the true pelvis or clinically involving
of the bladder or rectum
IVA Spread of growth ot adjacent organs
IVB Smead of growth to distant organs
(Q.102) The weight of the uterus at 8 weeks postpartum is
(a) 100 g
(b) 500 g
(c) 700 g
(d) 900 g
Your Response :
Correct Answer : a
Exp: The uterus weight about 1000 to 1200 gm immediately after delivery, As a r
involution, it decreases in weight to about 500 gm by one week, and to abou
the end of second week
At the end of 6 weeks its measurement is almost similar to that of the non-p
state and weight about 60 gm.

(Q.103) Following predispose to the high risk of vulvovaginal candidiasis except:


(a) HIV
(b) Hypertension
(c) Pregnancy
(d) DM
Your Response
:
Correct
b
Answer :
Exp: (Ref. Shaws Text book of Gynaecology 14th/ pg.98; H-
th
17 /pg. 1254)
Candidiasis (monilial vaginitis)
Candidiasis is due to gram- positive fungus candida albicans.
Predisposing factors:
Pregnancy
Diabetes
Hormonal contraceptives pills.
Predisposing factors for hematogenously disseminated
candidiasis:
Anti-bacterial agents,
Indwelling intravascular catheters,
Hyperalimentation fluids,
Indwelling urinary catheters,
Parenteral glucocorticoids,
Respirators,
Neutropenia,
Abdominal and tho- racic surgery,
Cytotoxic chemotherapy, and
Immunosuppressive agents for organ transplantation.
Severe burns
Low-birth- weight neonates, and
Illicit IV drug abusers
HIV-infected patients.
(Q.104) Which of the following is not caused by Oxytocin:
(a) Milk ejection
(b) Lactogenesis
(c) Contraction of uterine muscles
(d) Myoepithelial cell contraction
Your Response :
Correct Answer : b
Exp: OXYTOCIN causes contraction of the myoepithelial cells that line the ducts o
This squeezes the milk out of the alveoli of the lactating breast into the larg
then Out of nipple (milk ejection) Oxytocin also causes contraction of smoot
uterus.

(Q.105) Caudal regression syndrome is seen in babies of mother


having
(a) Gestational diabetes
(b) PIH
(c) Cardiac disease
(d) Anaemia
Your
Response :
Correct
a
Answer :
Exp: Diabetogenic effects of pregnancy
Insulin resistance
Production of placental Somatostatin
Production of cortisol, estriol and progesterone
Increased insulin destruction by kidney
andplacenta
Increased lipolysis (by HPL)
Changes in gluconeogenesis
Effect of DM on foetus
Hypoglycemia
Congo Abnormality - Hyper viscosity
Macrosomia
-HMO
Hypocalcemia
Apnea and bradycardia
Traumatic delivery
Effects of DM on mothers
Pre eclampsia
Infection
-PPH
High incidence of LSCS
IUD
Effects of pregnancy on diabetes
More insulin necessary for metabolic control
Progression of diabetic retinopathy
Worsening of diabetic nephropathy
Worsening of diabetic cardiomyopathy
Risk factors requiring diabetic screening
Age >30yrs
Family history of DM
Prior macrosomia
Malformed or still born infant
Obesity
HT
Glucosuria
(Q.106) Endometrial hyperplasia is seen in
(a) Endodermal sinus tumour
(b) Dysgerminoma
(c) Polycystic ovarian disease
(d) Carcinoma of cervix
Your Response :
Correct Answer : c
Exp: Polycystic ovarian disease - endometrial hyperplasia due to high content of
largely estrone, by extraglandular aromatization of circulating androstenedi
Endometrial hyperplasia is common in carcinoma endometrium
Treatment - simple endometrial hyperplasia calls for cyclic progestin therap
(medroxyprogesterone or norethindrone)
If endometrial hyperplasia with atypical cells or carcinoma of the endometri
hysterectomy is necessary

(Q.107) Oral anticoagulants given to pregnant women can cause?


(a) Long bones limb defect
(b) Craniofacial malformation
(c) CVS malformation
(d) Costo chondrodysplasia
Your Response :
Correct Answer : b
Exp: Oral anticoagulants (Warfarin) causes
Conradis syndrome (Skeletal and facial anomalies optic atrophy, microceph
chondrodysplasia punctata
Warfarin given in early pregnancy increases birth defects, specially skeletal a
foetal warfarin syndrome - hypoplasia of nose, eye socket hand bones, grow
retardation
Given later in pregnancy - it can cause CNS defects, foetal haemorrhage, foe
and accentuates neonatal hypoprothrombinemia.
(Q.108) Condyloma acuminata in pregnancy is treated by
(a) Podophyllin
(b) Podophyllin toxin
(c) Trichloroacetic acid
(d) 5-FU cream
Your Response :
Correct Answer : c
Exp: Treatment of condyloma -acuminata - (verruca-genital is)
Podophyllum is effective only on the genital warts
Podophyllum is potentially toxic and must be avoided in during pregnancy
Chemical cauterization is usually done with liquified phenol or concentrated
trichloroacetic acid

(Q.109) Placenta praevia is seen in all the following EXCEPT


(a) Large placenta
(b) Previous CS scar
(c) Primigravida
(d) Previous placenta praevia
Your
Response :
Correct
c
Answer :
Exp: Etiology of the placenta praevia - mainly
Advanced maternal age
Multiparity
Prior cesarean delivery
Smoking
Placental abnormalities eg, Big surface
succenturiate lobes
(Q.110) Risk of HIV transmission is highest during?
(a) During Caesarian section
(b) During antepartum period
(c) During vaginal delivery
(d) Breast feeding
Your Response :
Correct Answer : c
Exp: (Ref. Textbook of Gynaecology by D C Dutta 5th/pg. 538)
Mothers who are HIV infected can pass the virus on to their fetuses in utero
infants via breast milk. Vaginal delivery is the most common and most effec
transmission.
Perinatal transmissionThe vertical transmission to neonates of the infected
2535 %. The baby may be affected in utero (30%) through transplacental tr
during delivery (7075 %) by contaminated secretions and blood of the birth
through breast milk in neonatal period (1020%).
It is clear from recent European studies that Cesarean section alone, or in co
with antiretroviral therapy can effect an additional significant reduction in p
transmission. These results support the conclusion that some HIV infection m
acquired during passage through the vaginal canal. In nonbreast feeding pop
is estimated that 50 to 70 % of HIV infection is acquired during labor and de
the remainder in utero. To date, studies directed at reducing exposure to HI
birth other than Cesarean section, have not demonstrated a benefit in redu
transmission.

(Q.111) X-ray pelvimetry is indicated in all the following EXCEPT


(a) Severe CPD
(b) Breech presentation in vaginal delivery
(c) Outlet obstruction
(d) Osteomalacia
Your Response :
Correct Answer : a
Exp: Radiographic pelvimetry is the most accurate method we have to measuring
and determining the shape of he pelvis
Radiographic pelvimetry in pregnancy is always restricted to a single standin
film
Currently majority of x-ray pelvimetry are limited to a single film taken in th
lateral position.
Indication of caesarean section in CPD
Severely contracted pelvis, true conjugate is less than 9 cm
Elderly primigravida
Breech presentation
Previous caesarian section
Failed trial of labour
No indication of x-ray pelvimeter is done because PN examination detects se
CPD and advise for caesarian section
(Q.112) Late deceleration indicates
(a) Head compression
(b) Cord compression
(c) Foetal hypoxia
(d) Breech presentation
Your Response :
Correct Answer : c
Exp: Deceleration pattern - Three type
Early deceleration - due to head compression
Late deceleration - chronic placental insufficiency
Variable deceleration - cord compression
Sinusoidal pattern - It is stable baseline FHR with fixed base line variability w
acceleration. If is often associated with fetal anaemia, feto-maternal haemo
hypoxia, and when narcotics are given to mothers
In situations of uteroplacental insufficiency, the uterine contractions may de
placental perfusion sufficiently to cause fetal hypoxia which outlasts the dur
contraction.
The FHR tracing shows decelerations which begin with the peak of the uterin
contractions and persists even after the conclusion of the contraction (late
deceleration), late decelerations are indicative of fetal compromise

(Q.113) All of the following autoimmune disorder are more common in females except?
(a) ITP
(b) Multiple sclerosis
(c) Type I diabetes
(d) Scleroderma
Your Response :
Correct Answer : c
Exp: Most autoimmune disorders occur more commonly in women than in men;
include autoimmune thyroid and liver diseases, lupus, rheumatoid arthritis (
scleroderma, multiple sclerosis (MS), and idiopathic thrombocytopenic purp
However, there is no sex difference in the incidence of type 1 DM
(Q.114) Paramesonephric duct develop into:
(a) Vas deferens
(b) Seminal vesicle
(c) Ureter
(d) Uterus
Your Response :
Correct Answer : d
Exp: Male Female
Mesonephric duct Duct of epididymis Duct of epoophoron
(Wolffian duct) Ductus deferens Part of bladder and
urethra
Ejaculatory duct
Part of bladder and
prostatic urethra
Female Appendix of testis Uterine tube
(Paramesonephric or Prostatic utricle Uterus
Mullerian duct)
Vagina (?)
(Q.115) HRT is helpful in all of the following except:
(a) Vaginal atrophy
(b) Flushing
(c) Osteoporosis
(d) Coronary heart disease
Your Response :
Correct Answer : d
Exp: Consequence of HRT
1. Benefits
Decreased incidence of
a. Hip# b. Vertebral # c. Wrist # d. Colon cancer
Harms:
Increased incidence of
a. Coronary heart disease b. Thromboembolic events c. Breast
cancer d. Cholecystitis
2. Uncertain benefits
Prevention of dementia
(Q.116) Advantages of ultrasound nuchal translucency over biochemical screening for Downsyndrome in
(a) Uses transvaginal approach
(b) More consistent measurements than lab tests
(c) Better in multiple gestation
(d) Wide gestational age range
Your Response :
Correct Answer : c
Exp: The ultrasound nuchal translucency (NT) is now appreciated as a sensitive m
Down syndrome and other aneuploidies between 10 and 13 weeks. Outside
the NT disappears. Although some centers have had superb results, others h
done well. Blood free -hCG and PAPP-A in the first trimester, and double (AF
or triple (AFP, hCG, and estriol at 15 to 20 weeks) evaluations are statisticall
comparable. The combination of NT and first-trimester biochemistry will like
optimal approach. Biochemistry does not work well for multiple gestations.
can also detect structural anomalies, but often high-quality ultrasound servi
patients to travel long distances, whereas blood can be shipped from essent
anywhere to a competent lab.

(Q.117) Instrumental delivery system is


(a) Forceps only used in full cervical dilation
(b) Forceps used when ventouse fails
Ventouse cannot be used in rotational deliveries of occipito-posterior and
(c)
presentations
(d) Forceps can be used in breech deliveries
Your Response :
Correct Answer : b
Exp: Ventouse - is an instrumental device designed to assist delivery by creating a
between it and the fetal scalp.
In case of failed ventouse, forceps may be used if
1. Head is almost completely rotated
2. Cervix is fully dilated
3. There is some degree of descent
(Q.118) A patient with positive antiphospholipid antibodies would have all the following EXCEPT
(a) Recurrent foetal loss
(b) Venous thrombosis
(c) Thrombocytosis
(d) Neurological complications
Your Response :
Correct Answer : c
Exp: Common obstetric complications associated with antiphosphoid syndrome
1. Recurrent fetal loss 4. Placental abruption (2nd trimester
2. IUGR 5. Recurrent thrombotic events (Arterial
3. PIH venous thrombosis)
6. Thrombocytopenia 7. Auto immune or connective tissue disease
Many of these patients have SLE like symptoms, but do not meet specific dia
criteria for that disease
Livedo reticularis, skin ulcers, mental status changes and mitral regurgitation
noted
Treatment - may require immunosuppressive or anticoagulant medications
aspirin, heparin IGIV) azathioprine and cyclosporine
Diagnosis
False positive serological test for syphilis
Prolonged coagulation studies (PT, PTT)
Positive autoantibody test
Current data suggests the most efficacious therapy to low dose heparin, alo
dose aspirin, If active lupus is present then prednisone is usually also given

(Q.119) Ramkali, a 37-year-old, is a case of polycystic ovary disease. LH/FSH estimation has to be done in
of the menstruation cycle
(a) 1-4 days
(b) 8-10 days
(c) 13-15 days
(d) 24-26 days
Your Response :
Correct Answer : b
Exp: Pre-ovulatory, LH, FSH level will need to be estimated
Menstrual 1-3 days
Preovulatory 8-10 days
Ovulatory -13-15 days
Post ovulatory or premenstrual- 26-28 days
(Q.120) 18year old girl presents with amenorrhoea, milk discharge, weight loss. Diagnosis is
(a) Pitutary Cancer
(b) Anorexia nervosa
(c) Hypothyroidism
(d) Hypohalamic cause
Your Response :
Correct Answer : a
Exp: Its tempting here to go for Hypothyroidism but that is usually associated wit
gain. However, pituitary tumors and craniopharyngiomas can cause increase
consequent galactorrheaweight loss attributed then to the malignant proc

(Q.121) Which of the following is the biochemical marker of choice from cholestatic jaundice of pregnanc
(a) Serum Bilirubin
(b) Bile acids
(c) Bile salts
(d) Alkaline phosphatase levels
Your Response :
Correct Answer : b
Exp: (Ref. Dutta Obstetrics 6th/ 291; H-17th/ 265)
Cholestasis of pregnancy occurs in the second and third trimesters and reso
delivery. Its cause is unknown, but the condition is probably inherited and c
can be triggered by estrogen administration. In cases of intrahepatic cholest
pregnancy,bile acids are cleared incompletely and accumulate in the dermis
causes intense itching. These patients develop pruritus in late pregnancy; th
characteristic skin changes or rashes except in women who develop excoria
scratching. Cholestyramine is often used in cases of cholestasis of pregnancy
serum bile salts and decrease pruritus. 16 weeks. In some cases, there can b
transient hepatic dysfunction. Intrahepatic cholestasis of pregnancy is chara
pruritus and/or icterus. Some women develop cholestasis in the third trimes
secondary to estrogen-induced changes. There is an accumulation of serum
which causes the pruritus. Liver enzymes are seldom elevated above 250 U/
(Q.122) In postmenopausal women, estrogen is metabolized mostly into:
(a) Estriol
(b) Estrone
(c) Estradiol
(d) Androstenedione
Your Response :
Correct Answer : b
Exp: Circulating estrogens In the ovulating woman are derived from two source
Sixty percent of mean estrogen formation during the menstrual cycle is in th
estradiol Q, formed primarily by ovaries.
Remainder is estrone formed mainly in extraglandular tissues from androste
After menopause extraglandular estrogen formation is the major pathway fo
synthesis Q Estrogen production by the menopausal ovary is minimal Plasma
estradiol are lower in postmenopausal women than levels of estrone The ra
peripheral formation of estrone increases in menopausal women so that est
production is only slightly less than it was prior to the menopause despite th
plasma androstenedione. The predominant estrogen formed is ESTRONE rat
estradiol. Q

(Q.123) Adenoacanthoma is which type of uterine cancer?


(a) Poorly differentiated adenocarcinoma
(b) Well differentiation Adenocarcinoma
(c) Mucinous carcinoma
(d) Papillary serous carcinoma
Your Response :
Correct Answer : b
Exp: Between 75 and 80% of all endometrial carcinomas are adenocarcinomas, a
prognosis depends on stage, histologic grade, and extent of myometrial inva
I tumors are highly differentiated adenocarcinomas, grade II tumors contain
areas, and grade III tumors are largely solid or undifferentiated. Adenocarcin
squamous differentiation is seen in 10% of patients; the most differentiated
known as adenoacanthoma, and the poorly differentiated form is
called adenosquamous carcinoma. Other less common pathologies include m
carcinoma (5%) and papillary serous carcinoma (<10%).
(Q.124) Provisional short antiretroviral regime given in the peripartum period reduces the risk of vertical
transmission by
(a) 35%
(b) 50%
(c) 65%
(d) 75%
Your Response :
Correct Answer : b
Exp: Most transmission of HN occurs during pregnancy and birth, breast feeding
account for 5-15% of infants becoming infected after delivery
Studies have demonstrated that truncated regimens of zidovudine alone or
Lamivudine given to mother during last few weeks of pregnancy or even dur
& delivery & to the infant for a week or less reduced transmission to infant b
compared to placebo.

(Q.125) Which of the following neoplasms has been associated with the use of oral contraceptives?
(a) Breast cancer
(b) Ovarian cancer
(c) Endometrial cancer
(d) Hepatic adenoma
Your Response :
Correct Answer : d
Exp: Beginning with high dose combination contraceptive pills used over 20 year
have been studied extensively for a possible association with neoplasia.
There is only scant evidence from this experience that use of oral contracep
increases the risk of any type of cancer.
Actually, the progestational component of combination pills (or progestin-o
may confer a protective effect against carcinoma of the breast and endome
avoiding ovulation may decrease the risk of developing ovarian carcinoma.
A slightly higher risk of cervical carcinoma was observed in some studies of u
contraceptives. These studies were not controlled, however, for confoundin
such as multiple partners or age at onset of sexual intercourse, and it is gen
believed now that any increased risk in contraceptive pill users would be att
these other factors and not the steroids themselves. Although the risk of de
benign liver adenoma is there with with the use of oral contraceptives.
(Q.126) A case of Gestational trophoblastic neoplasia belongs to high risk group if disease develop after
(a) Hydatidiform mole
(b) Full term pregnancy
(c) Spontaneous
(d) Ectopic pregnancy abortion
Your Response :
Correct Answer : b
Exp: Classification of gestational trophoblastic disease
H. mole - 1. Complete 2. Partial
Gestational trophoblastic tumors
Non metastatic
Metastatic - Low risk - no risk factors
High risk - any risk factors
Pyretherapy hCG level> 40,000 miu/mL
Duration> 4 months
Brain or liver metastasis
Prior chemotherapy failure
Antecedent term pregnancy
Hydatidiform mole (181 H &B) - It is regarded as a benign neoplasm of the c
a high malignant potential, It may result in one of the most fatal malignancie
and is also potentially the most curable due to recent advances in diagnosis
management.

(Q.127) Mrs Shikha, 50-yrs-old woman is diagnosed with cervical cancer. Which lymph node group would
involved in metastatic spread of this disease beyond the cervix and uterus?
(a) Common iliac nodes
(b) Parametrial nodes
(c) External iliac nodes
(d) Paracervical or ureteral nodes
Your Response :
Correct Answer : d
Exp: The main routes of spread of cervical cancer include vaginal mucosa, myom
paracervical lymphatics, and direct extension into the parametrium. The pre
lymph node disease correlates with the stage of malignancy Primary node g
involved in the spread of cervical cancer include the paracervical, parametri
obturator, hypogastric, external iliac, and sacral nodes, essentially in that or
commonly, there is involvement in the common iliac, inguinal, and paraaort
stage I, the pelvic nodes are positive in approximately 15% of cases and the
nodes in 6%. In stage II, pelvic nodes are positive in 28% of cases and paraao
in 16%. In stage III, pelvic nodes are positive in 47% of cases and paraaortic n
28%.
(Q.128) Normal stature with minimal or absent pubertal development may be seen in
(a) Testicular feminization
(b) Kallmann syndrome
(c) Pure gonadal dysgenesis
(d) Turner syndrome
Your Response :
Correct Answer : b
Exp: Testicular feminization is a syndrome of androgen insensitivity in genetic ma
characterized by a normal 46,X genotype, normal female phenotype during
tall stature, and normal breast development with absence of axillary and p
Breast development (gynecomastia) occurs in these males because high leve
circulating testosterone (which cannot act at its receptor) are aromatized to
which then acts on the breast. The external genitalia develop as those of a f
because testosterone cannot masculinize them, while the Mullerian structu
absent because of testicular secretion of Mullerian-inhibiting factor in utero
dysgenesis (e.g., 45,X Turner syndrome) is characterized by short stature an
pubertal development; in these girls the ovaries are either absent or streak
are nonfunctional. In either case, estrogen production is possible, and there
isosexual pubertal development does not occur. Mailman syndrome (hypog
hypogonadism) should be suspected in patients of normal stature with dela
absent pubertal development, especially when associated with the classic fi
anosmia. These individuals have a structural defect of the CNS involving the
hypothalamus and the olfactory bulbs (located in close proximity to the hyp
such that the hypothalamus does not secrete GnRH in normal pulsatile fashi
Other causes of minimal or absent pubertal development with normal statu
malnutrition; anorexia nervosa; severe systemic disease; and intensive athle
particularly ballet and running.

(Q.129) Asherman's syndrome is due to:


(a) Trauma
(b) Cu T insertion
(c) Curettage
(d) Forceps application
Your Response :
Correct Answer : c
Exp: "Intrauterine synechiae" represent an important acquired uterine condition
seriously impair fertility. Adhesions usually result from vigorous curettage o
If the curettage has actually removed the endometrium, synechiae with asso
amenorrhea may result (Asherman's syndrome). Asherman's syndrome usua
vigorous curettage for postpartum hemorrhage or after therapeutic abortio
complicated by infection.
(Q.130) Which of the following causes increase in basal body temperature during ovulation:
(a) Progesterone
(b) Estrogen
(c) Luteinizing hormone
(d) Follicle stimulating hormone
Your Response :
Correct Answer : a
Exp: Biphasic changes in BASAL BODY TEMPERATURE are typical of the ovulatory
are mediated by alterations in PROGESTERONE levels. An increase in basal b
temperature by 0.3 to 0.5C begins after ovulation, persists during the lutea
returns to the normal baseline (36.2 to 36.4C) after the onset of the subseq
menses.

(Q.131) Not a feminizing type at ovarian tumor:


(a) Arrhenoblastoma
(b) Luteoma
(c) Thecoma
(d) Granulosa cell tumor
Your Response :
Correct Answer : a
Exp: The most common virilizing ovarian tumor is the arrhenoblastoma, but othe
tumors, such as adrenal rest tumor, granulosa cell tumor, hilar cell tumor, a
tumor, may also cause virilization.
(Q.132) Imiquimod is used for treatment of
(a) Molluscum contagiosum
(b) Warts
(c) Skin cancer
(d) All of the above
Your Response :
Correct Answer : d
Exp: Imiquimod belongs to a group of drugs called immune response modifiers.
It works byhelping immune system to fight these abnormal skin growths -
precancerous growths (actinic keratoses), a certain type of skin cancer (supe
cell carcinoma), and warts on the outside of the genitals/anus, keloid.

(Q.133) Which of the following cancer is commonly predisposed in the Lynch syndrome?
(a) Nonpolyposis colon cancer
(b) Ovarian cancer
(c) Endometrial cancer
(d) All of the above
Your Response :
Correct Answer : d
Exp: The Lynch syndrome occurs in families with an autosomal dominant mutatio
mismatch repair genes MLH1, MSH2, MSH6, and PMS2, which predispose to
nonpolyposis colon cancer as well as endometrial and ovarian cancer.
(Q.134) The most common pure germ cell tumor of the ovary is:
(a) Choriocarcinoma
(b) Dysgerminoma
(c) Embryonal cell tumor
(d) Malignant teratoma
Your
Response :
Correct
b
Answer :
Exp: (Ref. Shaws textbook of Gynacology-13th Edn-361)
Dysgerminoma:
It is the most common pure germ cell tumor of ovary.
It corresponds to Seminoma.
It is common under the age of 20.
It is usually unilateral.
It is neutral.
It secrets placental alkaline phosphatase.
It may be associated is hypoplasia or aplasia of part of
genital tract.
It is very radiosensitive.
It is potentially malignant with malignancy rate of 30-50%.
Endodermal sinus/yolk sac tumor is 2nd most common
germ cell tumor.
Embryonal cell tumor is rare.

(Q.135) Uterine blood flow at term:


(a) 50-75 ml/min
(b) 150-200 ml/min
(c) 350-400 ml/min
(d) 500-700 ml/min
Your Response :
Correct Answer : d
Exp: (Ref. Dutta Obstetrics 4th ed. 53)
Uterine blood flow is increased from 50 ml per minute in non-pregnant state
about 750 ml near term.
(Q.136) Which of the following congential anomalies is seen with maternal use of cocaine
(a) Sacral agenesis
(b) Hydrops
(c) Cerebral infarction
(d) Hypertrichosis
Your Response :
Correct Answer : c
Exp: Cocaine - use has been associated with MI, arrhythmias aortic rupture ,strok
bowel ischemia, hyperthermia, and sudden death
Risk of vascular disruption within the embryo-fetus or placenta is highest aft
trimester and likely accounts for the increased incidence of stillbirths
A number of cocaine related congenital anomalies due to vascular based
disruption, they include skull defect, cutis aplasia, porencephaly, ileal atresia
anomalies and visceral infarcts
Drug of abuse during pregnancy -
Cocaine - LBW, preterm delivery, increased placental abrupt us, increased fe
fetal tachycardia and hypertension, CNS irritability, cerebral infarct
Caffeine - Increased miscarriage and still births
Marijuana - Thymic hypoplasia

(Q.137) Which of the following is TRUE about Mifepristone


(a) Used for inducing abortion in early pregnancy
(b) Used along with contraceptive pills
(c) Acts on the cytoplasmic receptors
(d) Used for preventing ectopic implantation
Your Response :
Correct Answer : a
Exp: Mifepristone
This is progesterone antagonist
Termination is effective upto 9 weeks of pregnancy
A single dose of 600 mg is given orally. If abortion fails to occur by 36 hours,
prostaglandins El methyl ester pessary (germ PROST) 1 mg is introduced vag
complete the abortion process
Misoprostol (another PGE1 analogue) is equally effective
Side effects - Nausea, vomiting and rarely haemorrhage
Contraindication - It should not used in women age over 35 years, heavy sm
those on the long term corticosteroid.
(Q.138) Best treatment of endometriosis in a young girl is
(a) Oestrogen
(b) Gonadotrophins
(c) Surgery
(d) Danazol
Your Response :
Correct Answer : d
Exp: Endometriosis - Treatment - Danazol causes marked improvements in 75% -
is the preferred drug by most, Androgenic side effects are the limiting factor
GnRH analog such as nafarelin nasal spray - suppress ovulation, side effects
vasomotor symptoms and bone demineralization may be relieved by "add-b
therapy with norethindrone
Surgical measure - for moderately extensive endometriosis for under the ag
years, resect the lesion, free adhesions, and suspend the uterus.
If patient is over 35 years old - salpingo-oophorectomy and hysterectomy w
be necessary
Foci of endometriosis can b~ treated at laparoscopy by bipolar coagulation o
vaporization.

(Q.139) The engaging diameter in the following presentation is?

(a) Submentobregmatic
(b) Submentovertical
(c) Mentovertical
(d) Occipitofrontal
Your Response :
Correct Answer : c
Exp: Various diameter:
Sub-occipitobregmatic diameter: Measured from below the occipital protub
the centre of the bregma or anterior fontanelle
Sub-occipitofrontal diameter: Measured from below the occipital protubera
centre of the frontal suture
Occipitofrontal diameter: Measure from the occipital protuberance to the g
root of the nose
Mentovertical diameter: Measured from the point of the chin to the highest
the vertex
Sub-mentovertical diameter: measured from the point where the chinjoints
the highest point of the vertex
Sub-mentobregmatic diameter: measured from the point where the chin joi
to the centre of the bregma
Biparietal diameter: The diameter between the two parietal eminences
Bitemporal diameter: The diameter between the furthest points of the coro
at the temples
(Q.140) In which of the following heart diseases is maternal mortality during pregnancy is found to be th
(a) Coarctation of aorta
(b) Eisenmengers complex
(c) Aortic stenosis
(d) Mitral Stenosis
Your Response :
Correct Answer : b
Exp: PULMONARY HYPERTENSION Maternal mortality in the setting of severe pu
hypertension is high, and primary pulmonary hypertension is a contraindica
pregnancy. Termination of pregnancy may be advisable in these circumstan
preserve the life of the mother. In the Eisenmenger syndrome, i.e., the com
pulmonary hypertension with right-to-left shunting due to congen-, ital abn
maternal and fetal death occur frequently. Systemic hypotension may occur
loss, prolonged Valsalva maneuver, or regional anesthesia; sudden death se
hypotension is a dreaded complication. Management of these patients, is ch
and invasive hemodynamic monitoring during labor and delivery is generally
recommended. In patients with pulmonary hypertension, vaginal delivery is
stressful hemodynamically than Cesarean section, which should be reserved
accepted obstetric indications.

(Q.141) Treatment of menorrhagia are all the following EXCEPT


(a) NSAIDS
(b) Tranexamic acid
(c) Norethisterone
(d) Clomiphene
Your Response :
Correct Answer : d
Exp: Treatment of menorrhagia
Conservative - Rest, sedatives, reassurance, oral iron, blood transfusions
Hormones therapy - oestrogen, progestational steroids (Norethynodrel, nor
Danazol, OCP, Testosterone
NSAIDS, - Mefenamic acid
Antifibrinolytic agents - Tranexamic acid
GnRH agonist, ethamsylate, radiotherapy
Clomiphene - may be advocated if pregnancy is desire and if cycles are anov
(Q.142) Pain in early labor is transmitted through:
(a) T11-T12
(b) L2 - L3
(c) L4 - L5
(d) S2 - S3
Your Response :
Correct Answer : a
Exp: (Ref. Williams Obstetrics22nd ed. Chapter 19. Obstetrical Anesthesia)
Uterine Innervation
Pain during the first stage of labor is generated largely from the uterus.
Visceral sensory fibers from the uterus, cervix, and upper vagina traverse th
theFrankenhuser ganglion, Q
The Frankenhuser ganglion lies just lateral to the cervix, into the pelvic plex
then to the middle and superior internal iliac plexuses.
From there, the fibers travel in the lumbar and lower thoracic sympathetic c
enter the spinal cord through the white rami communicantes associated wit
through T12 and L1 nerves.
Early in labor, the pain of uterine contractions is transmitted predominantly
T11 and T12 nerves.
The motor pathways to the uterus leave the spinal cord at the level of the T
vertebrae.
Theoretically, any method of sensory block that does not also block the mot
to the uterus can be used for analgesia during labor.

(Q.143) All of the following drugs are commonly use in treatment of malignant germ cell tumor except:
(a) Bleomycin
(b) Etoposide
(c) Cisplatin
(d) Doxorubicin
Your Response :
Correct Answer : d
Exp: Most of the malignant germ cell tumors are managed with chemotherapy af
Regimens similar to those used in testicular cancer, such as BEP (bleomycin,
and cisplatin), with three or four courses given at 21-day intervals, have pro
long-term survival in patients with disease stages IIII. This regimen is the tr
choice for all malignant germ cell tumors except grade I, stage I immature te
where surgery alone is adequate, and perhaps early-stage dysgerminoma, w
surgery and radiation therapy are used.
(Q.144) Most preferable contraceptive device for a female suffering from Rheumatic Heart Disease who
completed her family is:
(a) IUCD
(b) Tubal ligation
(c) Norplant
(d) Barrier method
Your Response :
Correct Answer : b
Exp: Tubal ligation under local anaesthesia by minilap technique is the procedure
for permanent sterilization in a female with RHD who has completed her fam
The best time for the procedure is at the end of the first week in the puerpe
the heart is well compensated.

(Q.145) True about Vasa previa.


(a) Risk factor is low Lying placenta in second trimester
(b) Management is Caesarian section at 38 weeks
(c) Incidence is 1 in 1500
(d) Undiagnosed vasa previa carries a perinatal mortality of 20%
Your Response :
Correct Answer : b
Exp: Vasa Previa
Vasa previa is a rarely (1:2500)(option c) reported condition in which fetal b
vessel(s) from the placenta or umbilical cord crosses the entrance to the bir
beneath the baby.
The condition has a high fetal mortality rate (50-95%)(opion d). This can be
rapid fetal exsanguination resulting from the vessels tearing.
Vasa previa might be present if any of the following conditions exist:
Velamentous cord insertion
Bilobed placenta
Succenturiate-lobed placenta
Low-lying placenta or placenta previa
Pregnancies resulting from in-vitro fertilization
Multiple pregnancies
Maternal history of D&C or uterine surgery
Management
When vasa previa is detected prior to labor, the baby has a much greater ch
surviving. Survival rates can range from 50 -95%,
Vasa previa can be detected during pregnancy with use of transvaginal sono
preferably in combination with color Doppler. Women with the above risk f
should have this test to rule out vasa previa.
When vasa previa is diagnosed, elective delivery by caesarean(37-38 weeks)
labor begins can save the baby's life. Ideally, it should be performed early e
avoid an emergency, but late enough to avoid problems associated with pre
(Q.146) Hyperemesis gravidarum in 1st trimester is seen with increased frequency in all of the following
(a) Hydatidiform mole
(b) Twins
(c) Pre-eclampsia
(d) Primigravida
Your Response :
Correct Answer : c
Exp: Hyperemesis gravidarum is a severe type of vomiting of pregnancy, which h
deleterious effect on the health of the mother and/or incapacitates her "in d
activities. It is mostly limited to the first trimester of pregnancy and more co
first pregnancy but has a tendency to recur again in subsequent pregnancy.
family history and is more common in Unplanned pregnancies. It is more pre
hydatidiform mole and multiple pregnancy.

(Q.147) The most sensitive method for detecting cervical Chlamydia trachomatis infection is:
(a) Direct fluorescent antibody test
(b) Enzyme immunoassay
(c) Culture on irradiated MacConkey cells
(d) Polymerase chain reaction
Your Response :
Correct Answer : d
Exp: Nucleic acid probes have been developed for use in amplification assays suc
chain reaction and polymerase chain reaction (PCR). These tests are now the
sensitive chlamydial diagnostic methods available, being the first non-cultur
actually to surpass culture itself in sensitivity.
(Q.148) Pressure of normal uterine contractions isbetween 190-300. It is measured in terms of?
(a) Montevideo units
(b) Mm of Hg
(c) Cm of water
(d) Joules/kg
Your Response :
Correct Answer : a
Exp: (Ref. Williams obstetrics 22nd ed., p 466; Danforths Obstetrics and Gyneco
ed., ch 9)
The Montevideo unit =
The Average intensity of the uterine contractions X Number of contractions
minute period (expressed as mm Hg/10 min).
UTERINE ACTIVITY - STRENGTH
The strength of the contractions can only be assessed with the direct Intrau
Pressure Catheters (IUPC).
The normal baseline uterine pressure between contractions is approximatel
Abnormally high baseline pressures in excess of 20 mm Hg may result from
hyperstimulation or occasionally from overdistention of the uterus by exces
amnioinfusion, polyhydramnios, or fetal macrosomia.
During contractions, normal uterine pressure ranges from 30 to 80 mm Hg,
pressures in excess of 80 mm Hg may be observed during the second stage o
Many different quantitative approaches to uterine contractility have been
using IUPC technology.

(Q.149) Which of the following is NOT an assisted reproduction technique


(a) ZIFT
(b) GIFT
(c) IVF and embryonal transfer
(d) Artificial insemination into the uterus
Your Response :
Correct Answer : d
Exp: Assisted reproductive technologies
Couples who have failed to respond to traditional in fertility treatments, inc
with tubal disease, severe endometriosis, oligospermia, immunologic or une
infertility, may benefit from in vitro fertilization (IVF), gamete intrafallopian
(GIFT) and zygote intrafallopian transfer (ZIFT)
All of the procedures involve ovarian stimulation to produce multiple oocyte
GIFT - involves the placement of sperm and eggs in the uterine tube by lapa
minilaparotomy and is more invasive than IVF
GIFT is not appropriate for women with severe tubal disease and is less succ
IVF ZIFT - fertilization occurs in vitro, and the early development of the emb
the uterine tube after transfer by laparoscopy or minilaparotomy
A recent development is intracytoplasmic sperm injection (ICSI) which allow
with a single sperm, this provides the opportunity for men with severe oligo
obstructive azoospermia to father children.
Artificial insemination in Azoospermia - if azoospermia is present, artificial in
by a donor usually result in pregnancy, assuming female function is normal
(Q.150) Amniotic fluid contains acetyl cholinesterase enzyme. What is thediagnosis.
(a) Open spina bifida
(b) Gastroschisis
(c) Omphalocele
(d) Osteogenesis imperfecta
Your Response :
Correct Answer : a
Exp: Amniotic fluid a fetoprotein levels are measured if a neural tube defect is su
if USG is nondiagnostic in the presence of elevated maternal serum AFP.
If elevated, presence or absence of acetyl cholinesterase is determined in am
Presence of this enzyme confirms that there is exposed neural tissue. Elevat
ofamniotic fluid AFP level without acetyl cholinesterase suggest another etio
Maternal AFP raised in
Neural tube defect
Umbilical hernia
Sacral teratoma
IUD
Congo nephrosis
- Multiple pregnancy

(Q.151) Which of the following findings characterizes a normal semen sample?


(a) Agglutination
(b) Sperm concentration of 35 million per ml
(c) 5% normal sperm morphology
(d) 10% progressive sperm motility
Your Response :
Correct Answer : b
Exp: Because of the variability in semen specimens from the same person, prefer
specimens should be evaluated over the course of an investigation for infert
normal semen analysis will demonstrate at least 20 million sperm per millilit
60% of the sperm with a normal shape, a volume of between 2 and 6 mL, an
50% of the sperm with progressive forward motility.
(Q.152) A pregnant 35-yrs-old patient is at highest risk for the concurrent development of which of the fo
malignancies?
(a) Cervix
(b) Ovary
(c) Breast
(d) Vagina
Your Response :
Correct Answer : a
Exp: Cervical cancer is a more common gynecologic malignancy in pregnancy tha
breast cancer due to the fact that it is a disease of younger women. Manage
cervical intraepithelial lesions is complicated in pregnancy because of increa
vascularity of the cervix and because of the concern that manipulation of an
the cervix can compromise continuation of the pregnancy A traditional cone
only indicated in the presence of apparent microinvasive disease on a colpo
directed cervical biopsy Otherwise, more limited procedures such as shallow
biopsies are more appropriate. If invasive cancer is diagnosed, the decision
immediately or wait until fetal viability depends in part on the gestational ag
the diagnosis is made and the severity o disorder. Survival is decreased for m
discovered later in pregnancy Radiation therapy almost always results in spo
abortion, in part because the fetus is particularly radiosensitive. Chemother
associated with higher than expected rates of fetal malformations consisten
theantimetabolite effects of agents used. Specific malformations depend on
used and the time in pregnancy at which the exposure occurs.

(Q.153) Bevicizumab is used in the treatment of?


(a) Ovarian cancer
(b) Uterine cancer
(c) Cervix cancer
(d) Carcinoma of the fallopian tube
Your Response :
Correct Answer : a
Exp: Bevicizumab is a monoclonal antibody that targets the vascular endothelial
factor. Initial trials produced a 17% overall response rate in heavily pretreat
However, hypertension, thrombosis, and bowel perforations have been rep
some trials.
(Q.154) In leiomyoma of uterus which of the following changes does not occur:
(a) Fatty degeneration
(b) Squamous metaplasia
(c) Hyaline degeneration
(d) Atrophy
Your Response :
Correct Answer : b
Exp: Secondary Changes (Degenerations) Associated with Leiomyomas
Atrophy. It occurs due to reduced vascularity
Hyaline A hyaline myoma is hard and firm and it causes no specific
degeneration clinical symptoms.
Cystic It is seen most frequently in large intramural tumors and is
degeneration best marked in the middle because the blood supply is less
plentiful here.
Fatty changes Most cases result from previous red degeneration
Red degeneration Develops most frequently during pregnancy although it is
not rare. in cases of painful myomas in women over the
age of 40. Myoma becomes tense and tender and causes
severe abdominal pain with constitutional upset and fever
Tumor itself assumes a peculiar purple red color and
develops a fishy odor. Careful examination shows that
some of the large veins of the capsule and the small
vessels in the substance of the tumor are thrombosed.
Discoloration is possibly caused by diffusion of blood
pigments from the thrombosed vessels.
Sarcomatous It is extremely rare in leiomyoma and the incidence is less
changes than 0.5% of all myomas.
Intramural and sub-mucous tumors have a higher
potential for sarcomatous change than a sub-serous
tumor.
Torsion Subserous myoma may undergo rotation at the site of its
attachment to the uterus. Veins are occluded and tumor
becomes engorged with blood. Very severe abdominal
pain is experienced and most patients are operated upon
immediately. Very rarely the rotated tumor may adhere to
adjacent viscera, obtain a fresh blood Supply from theses
adhesions and finally be detached completely from uterus
the so called wandering fibroid or parasitic fibroid.
Inversion Caused by submucous fundal myoma.
Capsule rupture Very rare.
Inflammatory Most frequently arise in submucous myomas and
changes myomatous polypi.

(Q.155) A lady diagnosed with sputum positive TB. Best management is


(a) Wait for 2nd trimester to start ATT
(b) Start Category I ATT in I trimester
(c) Start Category II ATT in I trimester
(d) Start Category III ATT I trimester
Your Response :
Correct Answer : b
Exp: Tuberculosis during pregnancy should be diagnosed promptly and as early a
Late diagnosis and care is associated with 4-fold increase in obstetric morbid
fold increase in pre-term labour.
Poor nutritional states, hypo-proteinemia, anaemia and associated medical
add to maternal morbidity and mortality.
A foetus can get TB infection either by hematogenous spread through umbil
foetal liver or by ingestion or aspiration of infected amniotic fluid True cong
believed to be rare.
The risk to neonate of getting TB infection shortly after the birth is greater
ATT should be started promptly as untreated disease presents a hazard to th
and foetus.
The same regimens are recommended for use in pregnancy as for the non-p
state except for withholding of Streptomycin. Doubts about the use of Pyraz
in pregnancy have since been set as rest. Currently, an intermittent regimen
weekly on alternate days) under the DOTS strategy of RNTCP is being increa
world-wide for the pregnant women having TB
None of the ATT drugs are teratogenic and ATT should be started as soon as
diagnosis is made sputum positive tuberculosis is category 1
(Q.156) The risk of thromboembolism increases in pregnancy because
(a) Viscosity of blood increases
(b) Increased hepatic production of clotting factor
(c) Increased antithrombin III levels
(d) Increased progesterone levels
Your Response :
Correct Answer : b
Exp: Thromboembolism - The postpartum period is the commonest time in pregn
thromboembolism as the puerperium fulfills all the criteria of Virchow's tria
1. Increased coagulation - The increases in clotting factor from pregnancy re
although
plasma volume returns to normal with a few hours of delivery 2. Stasis - Ma
are immobilized during labour or the immediate puerperium 3. Damage to v
endothelium
Uterine vein - when placenta separates
Deep leg veins - when weight of legs continues to compress veins if women
immobilized in bed.

(Q.157) Single pelvic ala is absent in


(a) Robert's pelvis
(b) Naegele's pelvis
(c) Osteomalacia pelvis
(d) Rickets pelvis
Your
Response :
Correct
b
Answer :
Exp: Naegele's pelvis - Ala on one side is absent
Robert's pelvis - Ala on both sides are absent
Osteomalacic pelvis - the shape of inlet is triradiate
Rachitic pelvis - shape of inlet is reniform
The expected date of delivery can be calculated by
Naegele's rule
(Q.158) In Sheehan's syndrome, the most effective drug is:
(a) Corticosteroid
(b) Prolactin
(c) Estrogen
(d) Gonadotropins
Your Response :
Correct Answer : a
Exp: If systemic hypotension develops, as with postpartum hemorrhage, the pitu
undergo infarction (Sheehan's syndrome or post partum pituitary necrosis).
management of the patients who have already developed Sheehan's syndro
be done by endocrinology department and consists of "maintenance regime
thyroxin, adrenal cortical hormone and possibly gonadotropin.

(Q.159) Which of the following is responsible for menopausal hot flashes:


(a) Decreased progesterone
(b) Decreased estrogen
(c) LH surge
(d) FSH surge
Your Response :
Correct Answer : c
Exp: The most common menopausal symptoms are vasomotor instability (hot
flashes), atrophy of the urogenital epithelium and skin, decreased size of the
and osteoporosis. HOT FLASHES (FLUSHES) may start with an aura preceding
discomfort quickly followed by a feeling of heat moving toward the head. Ne
becomes red, and then there is sweating followed by exhaustion. The patho
the hot flash is uncertain. There is a close. Relationship between the onset o
flash and pulses of LH secretion. LH is secreted in episodic bursts at intervals
minutes, and in the absence of gonadal hormones, these burst are large. EA
FLASH BEGINS WITH THE START OF A BURST.
(Q.160) Which of the following is true about Mayer Rokitansky Kster Hauser?
(a) Ovary uterus tubes absent
(b) Uterus absent, tubes and ovary present
(c) Uterus present, tubes and ovary absent
(d) Uterus, tubes and ovary present
Your Response :
Correct Answer : b
Exp: Mllerian agenesis is acongenital malformationin women characterised by a
theMllerian ductsto develop, resulting in absent uterusand variable malfor
thevagina.
It is the second most common cause of primaryamenorrhea.
The condition is also called MRKH or Mayer-Rokitansky-Kster-Hauser Syndr
named afterAugust Franz Joseph Karl Mayer,Carl Freiherr von Rokitansky,He
Kster, andG.A.Hauser.
Signs and symptoms
A woman with this condition is hormonally normal, that is she will enter pub
development of secondary sexual characteristics including thelarcheandadr
chromosome constellation will be 46,XX.
Typically, the vagina is shortened and intercourse is difficult and painful.
Gynecologic Ultrasonographydemonstrates a complete or partial absence o
thecervix,uterus, andvagina.
Thissyndrome is characterized by congenital absence of vagina, primary am
rudimentary cornua uteri or absent uterus and morphologically normal ova
rudimentary Fallopian tubes(or fallopian tubes may also be absent) situated
pelvic sidewall.
Normal ovulation; normal breast development; normal body and hair. The w
amenorrheic and infertile.
Frequently associated with urinary tract anomalies, skeletal abnormalities, c
heart conditions, and inguinal hernia.
Due Since there is no uterus, women with MRKH cannot carry a pregnancy.
Women with MRKH typically discover the condition when, during puberty ye
menstrual cycle does not start.
Treatment
Although there are treatments to increase the comfort in sexual intercourse
none to let the woman become pregnant. Since the women do have ovaries
with this condition can have genetic children throughIVFwithembryo transfe
agestational carrier. Some women also choose to adopt.

(Q.161) False regarding complete testicular feminization is:


(a) Pre lobular bulb
(b) Presence of feminine breast
(c) Primary amenorrhea
(d) Blind vagina
Your Response :
Correct Answer : a
Exp: Complete testicular feminization is a common form of male pseudohermaph
is the 3rd most common cause of primary amenorrhea after gonadal dysgen
congenital absence of the vagina. The features are characteristic. Namely, a
ascertained either because of inguinal hernia (prepubertal) or primary amen
(postpubertal). The development of the breasts, the habitus, and the distrib
body fat are female in character so that most have a ''truly" feminine appea
Axillary and pubic hairs are absent or scanty, but some vulval hairs are usua
Scalp hairs are that of a normal woman, and facial hairs are absent. The exte
genitalia are unambiguously female, and the clitoris is normal. The vagina is
blind-ending and may be absent or rudimentary. All internal genitalia are ab
for testes that contain normal Leydig cells and seminiferous tubules without
spermatogenesis. The testes may be located in the abdomen, along the cou
inguinal canal, or in the labia majora.
(Q.162) The most common serious liver disease encountered during pregnancyis :-
(a) Acute fatty liver of pregnancy
(b) Liver damage due to preeclampsia
(c) Viral hepatitis
(d) Intrahepatic cholestasis of pregnancy
Your Response :
Correct Answer : c
Exp: Viral hepatitis is the most common serious liver desease encountered in pre
women. The 5 distinct types of viral hepatitis are A,B,D (caused by hepatit
associated with agent), C & E.Accute fatty liver of pregnancy or acute mera
or acute yellow atrophy is a serious but uncommon complication. The liver
beinvolved in severe pre-eclampsia, this lesion is unique and consist of peri
haemorrahage, fibrin deposition and hepatocyte disruption with necrosis.

(Q.163) EMA-CO regime of chemotherapy is used in the treatment of?


(a) Gestational trophoblastic neoplasia
(b) Malignant Ovarian germ cell tumor
(c) Endometrial carcinoma
(d) CA cervix
Your Response :
Correct Answer : a
Exp: Patients with high-risk tumors (high beta-hCG levels, disease presenting 4 m
antecedent pregnancy, brain or liver metastasis, or failure of single-agent
methotrexate) are initially treated with combination chemotherapy. EMA-C
non-cross-resistant combination of etoposide, methotrexate, and dactinom
alternating with cyclophosphamide and vincristine); cisplatin, bleomycin, an
vinblastine; and cisplatin, etoposide, and bleomycin are effective regimens.
now the regimen of choice for patients with high-risk disease because of exc
survival rates (>80%) and less toxicity.
(Q.164) Velamentous insertion of the cord is associated with an increased risk for:
(a) Premature rupture of the membranes
(b) Fetal exsanguinations before labor
(c) Torsion of the umbilical cord
(d) Fetal malformations
Your Response :
Correct Answer : b
Exp: With velamentous insertion of the cord, the umbilical vessels separate in th
membranes at a distance from the placental margin, which they reach surro
by amnion. It occurs in about 1% of singleton gestations but is quite commo
multiple pregnancies. Feta malformations are more common with velament
insertion the umbilical cord. When fetal vessels cross the internal us (vasa p
rupture of membranes may be accompanied by rupture of a fetal vessel, lea
exsanguination. An increased risk of premature rupture of membranes and
the umbilical cord has not been described in association with velamentous i
the cord.

(Q.165) This instrument can be applied to all presentations except?

(a) Face
(b) Vertex
(c) Aftercoming head in breech
(d) Brow
Your Response :
Correct Answer : d
Exp: Simpson forcepsare the most commonly used among the types of forceps a
elongated cephalic curve. These are used when there is substantialmolding,
temporary elongation of the fetal head as it moves through the birth canal
There is the ample pelvic curve in the single blade above and the cephalic cu
in the articulated blades below. The fenestrated blade and the wide shank in
the English-style lock characterize the Simpson forceps.
(Q.166) Highest rate of transmission of toxoplasmosis in pregnancy is:
(a) Puerperium
(b) 3rd trimester
(c) 2nd trimester
(d) 1 st trimester.
Your Response :
Correct Answer : b
Exp: About 1/3rd of all women infected with Toxoplasma gondii during pregnanc
the parasite to the fetus; the remainder will give birth to normal, un infecte
the various factors that influence fetal outcome, gestational age at the time
is the most critical. In pregnancy, if the mother becomes infected during the
trimester, the incidence of transplacental infection is lowest (about 15 %), b
disease in the neonate is most severe. If maternal infection occurs during th
trimester, the incidence of transplacental infection is greatest (65 %), but th
usually asymptomatic at birth.

(Q.167) Ru 486 can be used in all of the following conditions except


(a) Endometriosis
(b) Emergency contraception
(c) To cause abortion
(d) Endometrial carcinoma
Your Response :
Correct Answer : d
Exp: Mifepristone (RU 486) is a recently developed 19-nonsteroid with potent co
anti progestational and significant antiglucocorticoid activity. Its uses are as
Termination of pregnancy of up to 9 weeks: 600 mg as single oral dose caus
abortion in 60-85%'1 cases. Mifepristone administered within 10 days of a m
period results in an apparent late heavy period (with dislodged blastocyst) in
of cases. Ontra gestational: Given once a month on the expected date of me
it can dislodge the embryo (if present) and thus ensure menstruation irrespe
contraception or otherwise. Administered as a single dose within 72 hours o
intercourse, it can serve as a postcoital contraceptive with fewer side effect
dose estrogen. Induction of labor: By blocking the relaxant action of progest
uterus of late pregnancy, mifepristone can induce labor.
Cushing's syndrome: For inoperative cases.
Other uses under evaluation: Endometriosis, uterine fibroid, certain breast c
andmeningioma
(Q.168) Excessive intake (hypervitaminosis) of which of the following vitamin is associated with increase
congenital malformations:
(a) Vitamin A
(b) Biotin
(c) Folic acid
(d) Vitamin K
Your Response :
Correct Answer : a
Exp: Fetal abnormalities (including urinary tract malformations), growth retardat
early epiphyseal closure have; been reported in children whose mothers too
amounts of Vitamin A during pregnancy.

(Q.169) All of the following are causes of intrauterine growth retardation, except
(a) Anemia
(b) Pregnancy induced hypertension
(c) Maternal heart disease
(d) Gestational diabetes
Your Response :
Correct Answer : d
Exp: Maternal causes of IUGR
Small women Maternal genetic and racial factors
Malnutrition during Anemia
pregnancy
Hypertension Antiphospholipid syndrome
Poor weight gain during Cyanotic heart disease
pregnancy
Malabsorption syndrome Alcohol drinking
Cigarette smoking Chronic renal failure
Chronic UTI
(Q.170) Which of the following cardiac disorder is very common in pregnancy?
(a) Supraventricular tachycardia
(b) Restrictive cardiomyopathy
(c) Ventricular ectopic
(d) Type II B block
Your Response :
Correct Answer : a
Exp: Supraventricular tachycardia is a common cardiac complication of pregnanc
Treatment is the same as in the nonpregnant patient, and fetal tolerance of
such as adenosine and calcium channel blockers is acceptable. When necess
electrocardioversion may be performed and is generally well tolerated by m
fetus.

(Q.171) Integrase inhibitors are used for the treatment of ?


(a) HPV
(b) HIV
(c) Tuberculosis
(d) Cancers
Your Response :
Correct Answer : b
Exp: Integrase is an enzyme that integrates HIV genetic material into the DNA of
cells making it possible for the infected cell to make new copies of HIV. By in
with integrase, the integrase inhibitors prevent HIV genetic material from in
the CD4 cell, thus stopping viral replication
(Q.172) The most unfavorable presentation for vaginal delivery is
(a) Mentoposterior
(b) Mentoanterior
(c) Occipitoposterior
(d) Deep transverse arrest
Your Response :
Correct Answer : a
Exp: There is no possibility of spontaneous delivery in persistent mento-posterio
because, the relatively short neck cannot clear off the total length of the sac
cm) As such the thorax is thrust in, resulting bregmatic sternal diameter (18
occupy the pelvis As a result the labour becomes inevitably obstructed.
Mento-anterior - vaginal delivery
First stage - In uncomplicated cases, a wait and watch policy is adopted, lab
conducted in the usual procedure and the special instructions, as laid down
posterior position, are to be followed.
Second stage - one should wait for spontaneous delivery to occur, perineum
protected with liberal mediolateral episiotomy, In case of delay, forceps del
doneDeep transverse arrest
A. Vaginal delivery is found safe - Any of the methods may be employed (1)
ventouseideal in these cases (2) Manual rotation and application of forceps
rotation with Kielland in the hands of an expert (4) craniotomy in dead baby
B. Vaginal delivery is not safe (with big baby and or inadequate pelvis) - Cesa
section is to be done.

(Q.173) Which of the following is the most common cause of postpartum hemorrhage mandating hystere
(a) Uterine atony
(b) Placenta previa
(c) Placenta accreta
(d) Genital tract laceration
Your Response :
Correct Answer : c
Exp: An abnormally adherent placenta, although an uncommon condition, assum
considerable significance clinically because of morbidity and, at times, mort
severe hemorrhage, uterine perforation, and infection. Abnormally adheren
placentation caused 65 percent of cases of intractable postpartum hemorrh
requiring emergency peripartum hysterectomy.
(Q.174) All the following are TRUE about Manning score EXCEPT
(a) Non-stress test
(b) Oxytocin challenge test
(c) Body movement
(d) Respiratory activity of a child
Your Response :
Correct Answer : b
Exp: Manning score - A method of biophysical scoring system based on multiple p
determined at real time ultrasound scanning forms the basis of the test.
It has been of great help in the management of high risk pregnancies includi
cases in determining the timing and mode of pregnancy termination, when
fetus seems to be in imminent jeopardy,
The parameter that constitute the biophysical profile include
(A) Non stress test (B) Fetal breathing (C) Fetal tone (D) Gross body moveme
Volume of amniotic fluid present.
Each parameter is given a score of 2 points, A score of 8 to 10 correlates we
good pregnancy outcome A score of less than 6 should be viewed with cauti
should be repeated within 24 to 48 hours
A score of less than 2 is associated with poor fetal outcome.

(Q.175) HIV transmission to the fetus from mother occurs most commonly at what gestational age?
(a) Ist trimester
(b) IInd trimester
(c) III trimester
(d) During birth
Your Response :
Correct Answer : d
Exp: Virologic analysis of aborted fetuses indicate that HIV can be transmitted to
early as the first and second trimester of pregnancy. However, maternal tra
the fetus occurs most commonly in the perinatal period. Studies indicate tha
relative proportions of mother-to-child transmissions were 2330% before b
65% during birth, and 1220% via breast-feeding.
(Q.176) A young girl presents with primary amenorrhoea, grade V thelarche, grade II pubarche and no ax
The most probable diagnosis is
(a) Testicular feminization
(b) Mullerian agenesis
(c) Turner syndrome
(d) Gonadal dysgenesis
Your Response :
Correct Answer : a
Exp: Testicular feminization syndrome - complete androgen resistance, presents
phenotypic young woman without sexual hair but with normal breast develo
primary amenorrhoea
Gonadal dysgenesis (Turner syndrome) 45X, 46XX characterized by primary
amenorrhea, sexual infantilism short stature, bilateral gonads streaks, in ph
woman
Mullerian agenesis - Congenital absence of the vagina, is the second to gona
dysgenesis as a cause of primary amenorrhoea, the height is normal and the
axillary and pubic hair, and habitus are feminine in character

(Q.177) A 15-year-old girl presents with a large unilateral adnexal mass and ascites. Which lesion is MOS
this patient?
(a) Immature teratoma
(b) Brenner tumor
(c) Granulosa cell tumor
(d) Serous adenocarcinoma
Your Response :
Correct Answer : a
Exp: Immature teratomas consist of fetal or embryonic tissue. These ovarian tum
most commonly in prepubertal adolescents. They grow rapidly and frequent
the peritoneum, resulting in ascites. Many may metastasize to distant sites.
(Q.178) All of the following may be observed in a normal pregnancy EXCEPT
(a) Fall in serum iron concentration falls
(b) Increase in serum iron binding capacity
(c) Increase in blood viscosity increases
(d) Increase in blood oxygen carrying capacity
Your Response :
Correct Answer : c
Exp: Physiological changes in termed physiological anemia of pregnancy, Iron
supplementation augments the RBC mass during pregnancy, There are distin
advantages, in the hemodilution, It lowers the viscosity of the maternal bloo
adequate gaseous exchange between the maternal and fetal blood, it also p
mother against the adverse effects of blood loss during the delivery Hemato
changes Increased
Plasma volume, RBC volume
Total haemoglobin
Plasma iron binding capacity
Iron absorption doubles
WBC counts
Factor I, VII, VIII, IX, X
Platelet survival time XI, XIII
Plasminogen levels fibrinogen
Albumin levels, ESR
Albumin: globulin ratio
Platelet count is generally normal, bleeding and clotting times remains norm
as factor II, V and XII remain unchanged

(Q.179) Which of the following organism can cause epidemics of puerperalsepsis ?

(a) Cytomegalovirus

(b) GroupA -hemolytic streptococci

(c) Group B- hemolytic streptococci

(d) Toxoplasma gondii

Your Response :
Correct Answer : c

Exp:
Group A -hemolytic streptococci can cause puerperal or postoperative pelv
Outbreaks of puerperal fever are still reported on obstetric services, though
anywhere near the frequency of 50 years ago. When the disease does occur
source among the hospital personnel should be suspected. Group B -hemo
streptococci, which can also cause puerperal fever, have recently been reco
major cause of severe neonatal infection. The organism can be isolated from
cervixes of about 5% of all pregnant women; infection of the infant, which c
sepsis, occurs as the infant passes through the vagina T gondii, a protozoan
transmitted by flies from cat feces to human food. Thus, humans can becom
by consuming infected meat that is inadequately cooked or by coming in dir
with feces of an infected cat. Acute toxoplasmosis in a pregnant woman ma
a fulminantfetal infection; infected neonates may be born with microcephal
intracranial calcification, or other symptoms. An effective attenuated virus v
available for immunization against rubella. However, its use is generally con
for pregnant women and commonly is associated with development of arth
adults. Rubella syndrome has not been seen in fetuses when mothers are va
and vaccination can be considered if a pregnant woman is exposed to the vi
(Q.180) Formation of one of the following is essential for success of this instrument?

(a) Caput
(b) Chignon
(c) Phlegmon
(d) None of the above.
Your Response :
Correct Answer : b
Exp: CMI Tender Touch extractor cup.
Comparisons to other forms of assisted delivery
A. Positive aspects
An episiotomy may not be required.
The mother still takes an active role in the birth.
No special anesthesia is required.
The force applied to the baby can be less than that of a forceps delivery, and
no marks on the face.
There is less potential for maternal trauma compared to forceps and caesare
section.
B. Negative aspects
The baby will be left with a temporary lump on its head, known as a chignon
There is a possibility of cephalohematoma formation, or subgaleal hemorrha

(Q.181) Bonney's test is used to demonstrate:


(a) Neurogenic incontinence
(b) True incontinence
(c) Urge incontinence
(d) Stress incontinence
Your Response :
Correct Answer : d
Exp: Most common cause of leakage of urine in women is stress incontinence. Th
secondary to laxity of the pelvic floor with incompetence of bladder neck an
mechanism. It most commonly occurs in late multiparous" women although
degree of stress incontinence is experienced by many young women. Patien
of loss of urine associated with coughing, laughing, or sneezing and this may
with changes in posture. The demonstration of incontinence associated with
and its subsequent control applying Bonney's test (a finger either side of the
pushed upwards to support the pelvic floor) is important.
(Q.182) Treatment of stage II of carcinoma endometrium is:
(a) Radiotherapy+surgery
(b) Radiotherapy
(c) Surgery
(d) Chemotherapy
Your Response :
Correct Answer : a
Exp: Treatment of stage II carcinoma of endometrium consists of preoperative in
radiotherapy, followed within a week, by abdominal hysterectomy, bilateral
oophorectomy and pelvic lymph node dissection. This is followed by externa
radiotherapy if glands are found affected.

(Q.183) The following surgical procedure is done in which phase of the menstrual cycle?

(a) Follicular phase


(b) Luteal phase
(c) Ovulatory phase
(d) Irrespective of phase
Your Response :
Correct Answer : a
Exp: The picture is that of laparascopic tubal sterilization which can be done wi
done in the interval phase it is done within first 7 days of the cycle to avoid t
luteal phase pregnancy. POstpsartum sterilization is done by minilaparotom
laparoscopically.
(Q.184) At birth, oocytes are in which stage of development:
(a) Prophase of 1st meiotic division
(b) Oogonia
(c) Telophase of 2nd meiotic division
(d) Resting phase between prophase and metaphase of 1st meiotic division
Your Response :
Correct Answer : d
Exp: (184). Ans: d.
Exp. Resting phase between prophase and metaphase of 1st meiotic divisi
In the fetus the oogonia are proliferating mitotically. Near birth, the mitotic
stop and the oogonia enter into prophase of first meiotic division to become
oocytes.
At birth, the primary oocytes have finished the prophase of first meiotic divi
remain in resting phase between prophase and metaphase.
The first meiotic division of the primary oocytes completes at puberty.

(Q.185) A patient underwent surgery for ovarian mass diagnosed on ultrasound. The tumor markers wer
At laparotomy peritoneal washings were taken and after thorough inspection of
abdomen Ipsilateral salpingo-oophorectomy was performed. The lateral end of the pedicle is for

(a) Round ligament


(b) Ovarian ligament
(c) Mesosalpinx
(d) Infundibulopelvic ligament
Your Response :
Correct Answer : d
Exp:
The medial end of the pedicle is formed of round ligament and medial end o
tube that is the interstitial end. The middle portion of the pedicle is formed
mesosalpinx.
(Q.186) Ventouse extraction is done in all EXCEPT:
(a) Deep transverse arrest
(b) After-coming head of breech
(c) Delay in first stage due to uterine inertia.
(d) Delay in descent of high head in case of second baby of twins.
Your Response
:
Correct Answer
b
:
Exp: (186). Ans: b.
Exp. After-coming head of breech
Absolute indications for use of forceps (or contraindications for
use of ventouse)
After-coming head of breech.
Premature baby (fetal head remains in a protective cage).
Fetal distress (ventouse is unsuitable because it takes longer time
to deliver).
Anterior face presentation.
Pelvic contraction (ventouse cannot generate adequate traction),

(Q.187) Which of the following doesnt prevent prolapse of uterus


(a) Pubo-coccygeus
(b) Broad ligament
(c) Uterosacral ligament
(d) Mackenrodts ligament
Your Response :
Correct Answer : b
Exp: Supports of uterus (pelvic viscera) are mainly: 1.Muscular, 2. Pelvic fascia co
3. others
Muscular Pelvic diaphragm, Urogenital diaphragm & Perineal body. Pubo-
a muscle of pelvic diaphragm.
Pelvic fascia condensation pubo-cervical, utero-sacral, lateral cervical ligam
ofMackenrodt etc.
Others peritoneal folds like broad ligament, round ligament of uterus etc.
Note: Broad ligament & round ligament are themselves lax structures and d
provide a strong pelvic support.
(Q.188) Non-hormonal drug to prevent post menopausal osteoporosis is:
(a) Alendronate
(b) Calcium gluconate
(c) Vitamin D
(d) Calcitonin
Your Response :
Correct Answer : a
Exp: (188). Ans: a.
Exp. Alendronate
Alendronate, Etidronate, Pamidronate are biposphonates which inhibit
bone resorption.
Uses: 1. Postmenopausal osteoporosis
2. Paget's disease
3. Osteolytic bone metastasis
Route: Oral or i.v. infusion.
Side Effects: Abdominal pain, bowel upset, nausea, headache,
bodyache.

(Q.189) MTP cannot be done after:


(a) 12weeks
(b) 20weeks
(c) 24weeks
(d) 28weeks
Your Response :
Correct Answer : b
Exp: MTP is permitted upto 20weeks of pregnancy. When the pregnancy exceeds
opinion of two medical practitioners is required.
Husband's consent is not necessary for MTP.
(Q.190) A 16-year-old girl presents with blind vaginal pouch with absence of uterus. Investigation to be d
(a) Prolactin levels
(b) Karyotyping
(c) IVP
(d) FSH levels
Your Response :
Correct Answer : b
Exp: Indications
Patients with malformations suggestive of one of the recognized syndromes
with a specific chromosome aberration
Patients of any age who are grossly retarded physically or mentally especiall
are associated anomalies
Any patients with ambiguous internal or external genitalia or suspected
hermaphroditisms
Girl with primary amenorrhoea and boys with delayed pubertal developmen
Males with learning or behavioral disorders who are taller than expected (B
parental height)
Certain malignant & pre malignant disease
Parents of a patients with chromosomal translocations
Couples with history of multiple spontaneous abortion of unknown cause
Couples who are infertile after more common obstetric and urologic causes
excluded
Prenatal diagnosis - advanced maternal age, previous child with chromosom
aberration, intrauterine growth delay

(Q.191) Sign of inoperability of ovarian tumour is:


(a) Excrescences over surface
(b) Stromal invasion
(c) Peritoneal involvement
(d) Pelvic metastasis
Your Response :
Correct Answer : c
Exp: Staeine of Carcinoma Ovary -
Stage I - Growth limited to ovaries.
Stage II- Growth involving ovaries with pelvic extension or pelvic metastasis
Stage III-Carcinoma involving ovaries with peritoneal implant outside the pe
positive retroperitoneal or
inguinal nodes.
Stage IV- Growth involving ovaries with distant metastases or pleural effusio
parenchyma liver metastasis.
Stage and II are operable and total abdominal hysterectomy with bilateral sa
ophorectomy with omentectomy should be done.
Stage III and IV are inoperable; if found at laparotomy, debulking surgery is d
(Q.192) Which of the following does not contain Fat on mammography?
(a) Post-traumatic cyst
(b) Hamartoma
(c) Seborrhic keratosis
(d) Galactoele
Your Response :
Correct Answer : c
Exp: (Ref: RRM 5th ed. 545; Sutton Radiology 7th ed 1462)
Seborrheic keratosis is a cutaneous disease that can occur anywhere ove the
inframammary clefts being one of the known-site affected and on mammog
may see air lucencies trapped in the lesions. Galactoceles are more frequen
mixed density lesions than radiolucent lesions. They may be managed by sim
aspiration but in some cases this is difficult due to the thick consistency of th
Radiographically, a galactocele is seen as single or multiple nodular lesions w
density equal to or less than that of the fibroepithelial tissue of the breast.
Note: Fat contained within a lesion proves benignity!
MAMMOGRAPHY
Differential diagnosis of fat containing Breast lesions
Lipoma
Oil cyst
Galactocele = fluid with high lipid content (last phase) Hamartoma
Traumatic fat necrosis (cyst)
Focal collection of normal breast fat

(Q.193) A patient with severe pregnancy induced hypertension presents with cardiac failure after 1 week
uneventful normal vaginal delivery. The chest roentgenogram demonstrates enlargement of the
silhouette, suggestive of Dilated cardiomyopathy. Themortality in this condition is ?
(a) 10%
(b) 30%
(c) 50%
(d) 70%
Your Response :
Correct Answer : a
Exp: Cardiac dilatation and CHF may develop during the last trimester of pregnan
6 months of delivery. The cause is unknown, although inflammatory myocar
immune activation, and gestational hypertension have all been incriminated
patient who develops peripartum cardiomyopathy typically is multiparous a
years, although the disease may be found in a wide spectrum of patients.
The mortality rate of this disorder is around 10%.
The prognosis is related to whether the heart size returns to normal after th
episode of CHF. If it does, subsequent pregnancies may sometimes be tolera
with an increased risk of recurrent CHF; if the heart remains enlarged, and/o
ejection fraction (EF) remains depressed after 6 months, the prognosis is po
further pregnancies frequently produce additional myocardial damage, ultim
leading to refractory CHF. Patients who recover from peripartum cardiomyo
should be encouraged to avoid further pregnancies, particularly if LV dysfun
persists.
(Q.194) Cause of post-menopausal bleeding is:
(a) Arrhenoblastoma
(b) Cystadenoma
(c) Granulosa cell tumour
(d) Hilus cell tumour
Your Response :
Correct Answer : c
Exp: Another frequently asked ovarian tumour to cause post-menopausal bleedi
theca cell tumour.
Both granulosa cell tumour and theca cell tumour are feminising sex-cord
tumours of the ovary.

(Q.195) Diagnosis of Asherman syndrome is done by all of the following EXCEPT?


(a) Endometrial culture
(b) Hysteroscopy
(c) Hysterosalpingography
(d) Sonosalpingography
Your Response :
Correct Answer : a
Exp: Destruction of the endometrium usually follows vigorous curettage for post
hemorrhage
After therapeutic abortion getting complicated by infection
Overzealous fractional curettage for gynecological diagnosis
Tuberculosis is a common cause of asherman syndrome in India which prim
this by endometrial scarring
This diagnosis is confirmed by hysterosalpingography or by direct visual exam
the endometrial scarring or synechiae using a hysteroscope.
(Q.196) Not a feature of Stein Leventhal syndrome is;
(a) Increased androgens
(b) Increased or normal oestrogens
(c) Galactorrhoea
(d) Increased LH
Your Response :
Correct Answer : c
Exp: Stein-Levent Hal syndrome is another term for polycystic ovarian disease (PC
In this condition, androgens are raised due to hyperthecosis leading to hirs
Oestrogen level is normal but LH level and LH / FSH ratio is raised.
It is an important cause of secondary amenorrhoea and infertility in young w
Clomiphene is the drug of choice for infertility in PCOD and cyperterone ace
drug of choice for hirsutism.

(Q.197) Which surgical procedure has the highest incidence of ureteric injury?
(a) Vaginal hysterectomy
(b) Abdominal hysterectomy
(c) Wertheims hysterectomy
(d) Anterior colporrhaphy
Your Response :
Correct Answer : c
Exp: Wertheims requires dissection of the periureteral tissues and removing the
surrounding the course of the ureter. This can devascularize the ureter caus
fistulas. The next most common cause of ureteric injury is abdominal hyster
Another common cause ( and generally missed ) is the entrapment of the u
repairing high tears in the vaginal vault during cervical or vaginal lacerations
(Q.198) Most common cause of acute cervicitis is:
(a) E. Coli
(b) Chlamydia
(c) Pseudomonas
(d) Gonococcus
Your Response :
Correct Answer : d
Exp: Causes of acute cervicitis are : 1. Gonorrhoea 2. Septic
abortion 3.Puerperal Sepsis.

(Q.199) Dysfunctional uterine bleeding (DUB) is seen in:


(a) Polycystic ovarian disease
(b) Endometrial tuberculosis
(c) Metropathia haemorrhagica
(d) Multiple fibroids
Your Response :
Correct Answer : c
Exp: (199). Ans: c.
Exp. Metropathia haemorrhagica
The term DUB is used for menorrhagia in the absence of any structural abno
pelvic pathology or evidence of extra-genital cause for bleeding or endocrin
Metropathia haemorrhagica is a specialized form of DUB. Continuous uterin
the most constant symptom and may be preceded by amenorrhoea of abou
duration.
Bleeding is always painless and anovulatory.
Thick polypoidal endometrium and a cystic follicle are present in one ovary.
(Q.200) All of the following are indications for post operative radiotherapy in a case of Carcinoma Endom
EXCEPT?
(a) Myometrial invasion >1/2 thickness
(b) Positive lymph nodes
(c) Endocervical involvement
(d) Tumor positive for estrogen receptors
Your Response :
Correct Answer : d
Exp: As far as receptor status is concerned, the best correlation is that of a good
with high progesterone receptors rather than a poor prognosis with high est
receptors
The best prognostic factor of cancer endometrium, amongst others, is the H
grading of the disease
Other important predictors are:
Depth of myometrial invasion Status of pelvic and para-aortic lymp
Malignant cells in peritoneal washings Lymph vascular invasion
Cervical invasion

S-ar putea să vă placă și